1. Trang chủ
  2. » Lịch sử

Tạp chí Epsilon số 3

237 21 0

Đang tải... (xem toàn văn)

Tài liệu hạn chế xem trước, để xem đầy đủ mời bạn chọn Tải xuống

THÔNG TIN TÀI LIỆU

Nội dung

Đặc biệt mảng hình học lần này sẽ giới thiệu một chùm hoa đẹp gồm ba bài viết do các thành viên của nhóm Bài toán hay – Lời giải đẹp – Đam mê toán học (Vũ Thanh Tùng, Nguyễn Chương Chí, [r]

(1)

90

90

90

x

x

x

45

x

1/3x

Nếu bạn muốn bạn thông minh,

hãy đọc cho chúng nghe truyện cổ

tích Nếu bạn muốn bạn thông

minh hơn, đọc cho chúng nhiều

truyện cổ tích hơn.

Albert Einstein

No

135

HENRYK MINC

ghi hồi ông nộp

một số báo vĩnh

thức kỷ 20

có bình duyệt viên

nói

“chế

tên lố bịch thế”?

V ĩnh

thứ c - Ng ô Q

u an g H ưn g

-Ma trận

ngẫu nhiên

Vũ Hà Văn

Thư

Kapitsa

về

khoa học

Đàm Thanh Sơn (dịch)

Cubic Rubik

Trần Nam Dũng (dịch tổng hợp)

thầy tốt hơn

Nguyễn Tiến Dũng

Định lí bướm kép

đối

với tứ giác.

Nguyễn Ngọc Giang Trịnh Huy Vũ

(2)

Tạp chí online

của cộng đồng

những người yêu Toán

ngày

13

tháng

06

năm

2015

Số

3 Chủ biên: TRẦN NAM DŨNG

Biên tập viên: VÕ QUỐC BÁ CẨN TRẦN QUANG HÙNG LÊ PHÚC LỮ

NGUYỄN TẤT THU

(3)

Ban biên tập Epsilon

Epsilon số mắt bạn đọc hạn vào ngày 13/4 làm cho Epsilon khơng thuộc vào đội ngũ Tạp chí số Tức tạp chí dồn cơng sức số đầu hay, tốt

Có số tức hy vọng có số Và bạn đọc đọc số tạp chí Epsilon – tạp chí online người yêu toán Qua số đầu tiên, vui mừng người đọc Epsilon nhiều lên, người biết đến ủng hộ Epsilon nhiều lên, người viết cho Epsilon nhiều lên Trong buổi uống bia trưa 10/6 quán Hải Xồm gần Viện Tốn học, chúng tơi cảm thấy vui vui người nhắc đến Epsilon “Hôm trước gặp hội anh Nguyễn Thành Nam, có nói vừa tờ Epsilon Trước Ngơ Bảo Châu có nói anh rảnh viết cho Epsilon” – GS Phạm Hữu Tiệp, người vừa trình bày chun đề Viện tốn nói với “Anh rảnh viết cho em Random walk anh”, “Chưa biết anh có viết không, sau trận bia anh với ramdon walk viện Toán” Thấy người bàn tán rơm rả Epsilon, PGS trẻ Phạm Hồng Hiệp, người vừa giải thưởng Tạ Quang Bửu, cịn nói đùa “Có anh Dũng phải đổi tên tạp chí thành 1/epsilon nên” Nói vui thế, cơng việc Epsilon thực công việc gom li ti để tạo nên sản phẩm nhỏ mà ý nghĩa, để người đọc ln tìm điều cho qua số tạp chí

(4)

với phong cách nghiên cứu trình bày báo khoa học Các thầy giáo có động lực việc tổng kết chuyên đề cách hệ thống hơn, thay viết nhỏ với ý tưởng đơn lẻ

Epsilon số lần quy tụ 14 viết với chủ đề phong phú: Các viết Vũ Hà Văn “Ma trận ngẫu nhiên” Ngô Quang Hưng “Vĩnh thức định thức Pfaff” giới thiệu với bạn đọc vấn đề nóng hổi toán học đại Mục lịch sử toán học dành trang viết cho nhà Vật lý qua “Những triết lý sống Einstein” BBT sưu tầm “Thư Kapitsa khoa học” GS Đàm Thanh Sơn dịch giới thiệu Mục Giảng dạy tốn học lần có GS Nguyễn Tiến Dũng “Tự học tốt có thầy tốt hơn” Nguyễn Quốc Khánh tiếp tục chuyên mục điểm sách đầy hấp dẫn qua “Đêm trước thảo in” Đặng Nguyễn Đức Tiến tạm gác chủ đề mũ để giới thiệu nhà truyền bá toán học tiếng giới – Martin Gardner Chuyên mục Các vấn đề cổ điển đại giới thiệu chuỗi toán đếm tam giác (dành cho học sinh tiểu học THCS) chuỗi tốn khối vng Rubik (từ trị chơi đến lý thuyết nhóm)

Mảng tốn sơ cấp, thường lệ rôm rả với viết công phu “Cực trị tập hợp” thầy Trần Minh Hiền, tiểu phẩm xinh xinh thầy Nguyễn Duy Liên xung quanh toán thi IMO 2001 Các học sinh chuyên toán chuẩn bị cho kỳ thi HSG năm học sau chắn tìm nhiều điều bổ ích qua bình luận lại kỳ thi chọn đội tuyển Việt Nam 2015 thầy Trần Nam Dũng

(5)

thành 10% nhiệm vụ người làm Epsilon đỗi vui mừng

(6)(7)

1 Lời ngỏ Ma trận ngẫu nhiên

Vũ Hà Văn

3 Vĩnh thức 29 Ngô Quang Hưng

4 Tự học tốt có thầy tốt 61 Nguyễn Tiến Dũng

5 Thư Kapitsa khoa học 71 Đàm Thanh Sơn

6 Những triết lý sống Einstein 81 Ban biên tập Epsilon

7 Lời giải bình luận đề thi TST 2015 89 Trần Nam Dũng

8 Martin Gardner - Người làm vườn toán học 105 Đặng Nguyễn Đức Tiến

9 Cực trị tập hợp 123 Trần Minh Hiền

(8)

11 Định lý Carnot ứng dụng 181 Vũ Thanh Tùng, Nguyễn Chương Chí

12 Về tốn hình học từ diễn đàn AoPS 193 Trần Quang Hùng, Nguyễn Bảo Ngọc

13 Định lý bướm kép tứ giác 205 Nguyễn Ngọc Giang (TP Hồ Chí Minh)

Trịnh Huy Vũ (THPT Chuyên KHTN Hà Nội)

14 Đêm trước thảo in 211 Nguyễn Quốc Khánh

(9)

VŨ HÀ VĂN

(Đại học Yale, Mỹ)

Lời giới thiệu

Lý thuyết ma trận ngẫu nhiên có mục tiêu đưa hiểu biết sâu sắc tính chất đa dạng ma trận mà thành phần chúng chọn ngẫu nhiên từ phân phối xác suất khác

Từ đời đến nay, lý thuyết ma trận ngẫu nhiên có phát triển mạnh mẽ, thúc đẩy ứng dụng Thống kê Giải tích số, Khoa học máy tính, Điều khiển tối ưu, đặc biệt ứng dụng Vật lý hạt nhân

Ở Việt Nam, lý thuyết ma trận ngẫu nhiên khái niệm tương đối Năm 2009, người viết lời giới thiệu này, dạy cho đội tuyển Olymic Toán Việt Nam chuẩn bị cho kỳ thi toán quốc tế IMO 2009, dẫn toàn đội tuyển đến dự nói chuyện GS Vũ Hà Văn Ma trận ngẫu nhiên Thú thực thầy trò hiểu lõm bõm điều GS Văn nói, tất ấn tượng hàng loạt giả thuyết Vũ Hà Văn Terence Tao chứng minh với tốc độ chóng mặt

Trong Epsilon số này, đồng ý tác giả, chúng tơi trích giới thiệu nội dung chương đầu báo cáo GS Vũ Hà Văn Đại hội Toán học Thế giới 2014 (ICM 2014) Để giúp độc giả nắm bắt nội dung chính, chúng tơi cố gắng giải chi tiết có thể, đồng thời đăng nguyên tiếng Anh để đối chiếu Vì lĩnh vực mới, có tài liệu tiếng Việt nên dịch thuật có chỗ chưa chuẩn, mong nhận ý kiến đóng góp bạn đọc để phần sau dịch tốt

(10)

Tóm tắt nội dung

Trong viết này, trao đổi số tốn lý thuyết ma trận ngẫu nhiên có chất tổ hợp

1 Mở đầu

Lý thuyết ma trận ngẫu nhiên mảnh đất màu mỡ toán học Bên cạnh vấn đề nội thú vị, ma trận ngẫu nhiên đóng vai trò quan trọng nhiều lĩnh vực Thống kê, Vật lý Tốn, Tổ hợp, Khoa học Máy tính

Trong khảo sát này, tập trung vào tốn có chất tổ hợp Các toán đặc biệt thú vị ma trận lấy mẫu từ phân phối rời rạc Các mơ hình thơng dụng là:

• (Bernoulli) Mn: ma trận ngẫu nhiên bậc n mà thành

phần biến ngẫu nhiên độc lập đồng theo phân phối Bernoulli (nhận giá trị ±1 với xác suất 1/2) Ma trận đơi cịn gọi ma trận dấu ngẫu nhiên1 Tổng cộng có tất N = 2n2

ma trận với tất thành phần là±1, ma trận có xác suất1/N

• (Bernoulli đối xứng2) Msym

n : ma trận đối xứng ngẫu nhiên

bậc n mà thành phần đường chéo

các biến ngẫu nhiên độc lập đồng theo phân phối Bernoulli Số ma trận đối xứng với thành phần ±1 M = 2n(n+1)/2 và ma trận có xác suất1/M.

• Ma trận kề đồ thị ngẫu nhiên Với đồ thị ta có ma trận kề định nghĩa sau: Giả sử đồ thị G

có n đỉnh {1,2 , n} Ma trận kề G ma trận đối

xứng vị tríij ta viết1nếu ij cạnh củaGvà0trong trường hợp ngược lại

Về mơ hình đồ thị ngẫu nhiên. Trong viết này, xột trờn hai mụ hỡnh: Erdăos-Rộnyi v th u ngẫu nhiên Chi tiết mơ hình này, xem [6, 35]

1Random sign matrix Toàn thích Ban

Biên tập

(11)

ã (Erdăos-Rộnyi) Ta ký hiu G(n, p) đồ thị ngẫu nhiên n đỉnh, sinh cách vẽ cạnh nối hai điểm với xác suấtp cách độc lập

• (Đồ thị ngẫu nhiên3) Đồ thị ngẫu nhiên cón đỉnh với bậcdthu cách chọn ngẫu nhiên với xác suất tập hợp tất đơn đồ thị bậcd4 tập đỉnh{1,2, , n} Ta ký hiệu đồ thị Gn,d

Có ý quan trọng cạnh củaGn,dkhơng độc lập5 Vì

vậy, mơ hình thường khó nghiên cứu mơ hình G(n, p) Ta ký hiệu A(n, p) ma trận kề th ngu nhiờn Erdă os-RộnyiG(n, p), v An,d l ma trận kề Gn,d tương ứng

Về ký hiệu: Trong suốt này, n giả sử lớn Các ký hiệu tiệm cận6 o, O,Θ hiểu n tiến tới vô Ta viết A B A = o(B) c ký hiệu cho số chung7 Tất logarit logarit tự nhiên khơng nói khác

2 Xác suất suy biến

Bài toán tổ hợp tiếng ma trận ngẫu nhiên có lẽ toán suy biến8 Gọip

n xác suất ma trậnMn suy biến (một

ma trận vuông suy biến định thức 0) Hiển nhiên là:

pn≥2−n

vì vế phải xác suất để hai dòng đầu ma trận nhau9.

Vì ta chọn hai dịng (cột) (thay chọn dịng đầu) thay dấu dấu nhân với

3Random regular graph.

4Đồ thị bậcd(d-regular graph) đồ thị mà đỉnh có bậc bằng d Ví dụ đồ thị bậc có đỉnh đỉnh cô lập, đồ thị đầy đủKnlà

đồ thị bậcn−1

5Nghĩa xác suất tồn cạnh củaG

n,d không độc lập với nhau,

trong xác suất cạnh củaG(n, p)là độc lập

6Asymptotic notation.

7Universal constant Đôi dịch số phổ dụng hay hằng

số độc lập

8Singularity problem.

(12)

±110, ta có cận tốt chút:

pn ≥(4−o(1))

n

2

2−n= (1

2 +o(1))

n (2.1)

Một giả thuyết đặt là:

Giả thuyết 2.1. [Giả thuyết suy biến] pn = (12 +o(1))n

Giả thuyết 2.1 tốn mở, ta phát biểu giả thuyết xác (xem [4]), dựa vào "niềm tin" sau:

Hiện tượng I.11 Lý chủ yếu để ma trận ngẫu nhiên suy

biến phụ thuộc số hàng/cột

Thực việc chứng minh pn =o(1) không đơn

giản Kết lần chứng minh Komlós [38] vào năm 1967 (trong phần này, đưa chứng minh ngắn gọn cho định lý Komlós) Sau Komlós (xem [6]) tìm chứng minh cho cận trên: pn =O(n−1/2)

Trong báo quan trọng, Kahn, Komlós Szemerédi [37] lần đưa cận theo hàm mũ:

Định lý 2.2. pn≤.999n

Lập luận Kahn, Komlós Szemerédi đơn giản hóa Tao Vũ báo [66] năm 2004, dẫn đến cận tốt chút O(.958n) Sau lâu, tác giả [67]

kết hợp cách tiếp cận [37] với ý tưởng định lý đảo (xem [71, chương 7] hay [53]) đạt kết quan trọng:

Định lý 2.3. pn≤(3/4 +o(1))n

Khơng dừng lại đó, Bourgain, Vũ Wood báo [9] dùng thêm ý tưởng khơng gian có chiều phân số để tiếp tục cải thiện cận trên:

Định lý 2.4. pn≤(√12 +o(1))n

10Nghĩa ta chọn dịng (hoặc cột) có trị tuyệt đối

thay

11Ở tác giả dùng "Phenomenon" nên đối dịch "Hiện tượng".

(13)

Hai phương pháp [67, 9] cho phép thu cận củapntrực tiếp từ ước tính lượng giác đơn giản Ví

dụ cận 3/4 có từ:

|cosx| ≤

4+

4cos 2x,

trong cận1/√2thu từ

|cosx|2 = +

1

2cos 2x

Định lý 2.2 [9] đưa mối liên hệ hình thức ước lượng suy biến ước lượng lượng giác Các liên hệ chưa thể dùng để giải toán suy biến tổng quát, sử dụng để ước tính xác cận số trường hợp, chẳng hạn xác suất suy biến ma trận ngẫu nhiên với thành phần(0,±1)

Để kết thúc mục này, đề cập đến công cụ hu ớch: nh lý Littlewood-Offord-Erdăos Gi v = {v1, , vn} tập

hợp gồm n số thực khác ξ1, , ξn biến ngẫu nhiên

Bernoulli độc lập phân bố đồng Định nghĩa S :=Pni=1ξivi

vàpv(a) =P(S =a) vàpv = supa∈Zpv(a)

Vào năm 1940, Littlewood Offord đưa cách ước tính pv (ở [45]) thành tố kỹ thuật nghiên cứu họ nghiệm thực đa thức ngu nhiờn Erdăos, bng cỏch ci thin kt qu ca Littlewood Offord, chứng minh định lý sau mà gọi bất đẳng thức bóng nhỏ Erdăos-Littlewood-Offord (xem [53] rừ hn v cỏi tờn ny)

Định lý 2.5. (Bất đẳng thức bóng nhỏ) Giả sử v1, ,

các số thực khác0vàξ1, , ξn biến ngẫu nhiên Bernoulli

độc lập phân bố đồng Khi đó:

pv

n

bn/2c

2n =O(n

−1/2).

(14)

Để độc giả cảm nhận bất đẳng thức bóng nhỏ có ích việc đánh giá pn, ta xếp dòng

của Mn dòng từ xuống Giả sử n −1

dòng đầu độc lập tạo thành siêu phẳng với véc-tơ pháp tuyếnv= (v1, , vn) Khi đó, xác suất để Mn suy biến là:

P(v= 0) =P(ξ1v1 +· · ·+ξnvn = 0),

trong X = (ξ1, , ξn) dòng cuối

Trong phần3, độc giả thấy ứng dụng định lý2.5dẫn đến kết gốc Komlós: pn = o(1) Để thu đánh

giá mạnh kết định lý 2.3 2.4, ta cần thiết lập định lý Littlewood-Offord đảo, dựa nguyên lý tổng quát sau:

Hiện tượng II. Nếu P(X ·v = 0) là tương đối lớn hệ số

v1, , có cấu trúc cộng tính mạnh.

Các định lý thúc đẩy định lý đảo kiểu Freiman tổ hợp cộng tính12 mà việc thảo luận nằm phạm vi khảo sát Độc giả quan tâm xem chi tiết [53]

3 Một chứng minh đơn giản định lý

Komlós

p

p

p

nnn

=

=

=

o

o

o

(1)

(1)

(1)

Ta tính chất đơn giản Từ sau véc-tơ Bernoulli hiểu véc-tơ với tọa độ±1

Tính chất 3.1. Cho H không gian ≤ d ≤ n chiều Khi H chứa nhiều 2d véc-tơ Bernoulli.

Để thấy điều này, ta ý không gian d chiều, tồn tập hợpd tọa độ xác định tọa độ lại13 Tính chất suy ra:

pn ≤

n−1

X

i=1

P(Xi+1 ∈Hi)≤ n−1

X

i=1

2i−n

≤1−

2n

12Additive Combinatorics.

13Trong khơng giand chiều, dùng d véc-tơ sở để biểu diễn mọi

(15)

Rất không điều đối nghịch với kết muốn chứng minh, đừng vội nản chí, hay cịn phần sau! Để thu cận o(1) mong muốn, ta cần chứng minh tổng số hạng tử cuối, chẳng hạn log logn, không

vượt (chẳng hạn)

log1/3n Để chứng minh điều này, ta sử

dụng tính chất Hi sinh véc-tơ ngẫu nhiên

Bổ để sau suy định lý Komlós thơng qua định lý cận hợp14:

Bổ đề 3.1. Cho H không gian sinh d véc-tơ ngẫu nhiên, đód≥n−log logn Khi với xác suất nhất1−1

n,

H chứa nhiều 2n

log1/3n véc-tơ Bernoulli

Ta nói tập hợp S gồm d véc-tơ k-phổ dụng với tập k số khác ≤ i1, , ik ≤ n tập dấu

1, , n (i =±1) nào, tồn véc-tơ v thuộc S cho dấu

của tọa độ thứij củav j, với mọi1≤j ≤k

Tính chất 3.2. Nếud≥n/2thì 1−

n xác suất thấp cho

tập hợp gồmd véc-tơ ngẫu nhiên k-phổ dụng với k := logn/10

Để chứng minh điều này, ta ý xác suất thất bại, theo định lý cận hợp, không vượt

n k

(1− 2k)

d

≤nk(1− 2k)

n/2 ≤n−1.

Nếu S k-phổ dụng, véc-tơ v khác phần bù trực giao không gian sinh S có nhiều k véc-tơ khác (nếu khơng, có véc-tơ S có tích trong15

14Union bound Cịn gọi bất đẳng thức Boole, phát biểu với mọi

tập hữu hạn đếm xác suất để kiện xảy nhỏ tổng xác suất tất kiện, nghĩa E1, E2, En,

các kiện thì:

P{∃i:Ei xảy ra}=P{

[

i=1

Ei} ≤

X

i=1

P{Ei}

Ở số tài liệu union bound gọi định lý tổng xác suất

(16)

dương với v) Nếu ta cố định véc-tơ v giả sử X véc-tơ ngẫu nhiên Bernoulli theo định lý 2.5

P(X ∈Span(S))≤P(X·v = 0) =O(

k1/2)≤

1 log1/3n, bổ đề3.1 định lý chứng minh

* * *

(17)

Keywords General mathematics, collection of articles

Abstract In this survey, we discuss several problems in Random Matrix theory of combinatorial nature

1 Introduction

The theory of random matrices is a very rich topic in mathematics Beside being interesting in its own right, random matrices play fundamental role in various areas such as statistics, mathematical physics, combinatorics, theoretical computer science, etc

In this survey, we focus on problems of combinatorial nature These problems are most interesting when the matrix is sampled from a discrete distribution The most popular models are:

• (Bernoulli) Mn: random matrix of size n whose entries are i.i.d Bernoulli random variables (taking values±1 with probability 1/2) This is sometimes referred to as the random sign matrix

• (Symmetric Bernoulli) Msym

n : random symmetric matrix of size n whose

(upper triangular) entries are i.i.d Bernoulli random variables

• Adjacency matrix of a random graph This matrix is symmetric and at

position ij we write if ij is an edge and zero otherwise

• Laplacian of a random graph

Model of random graphs We consider two models: Erdăos-Renyi and random reg-ular graphs For more information about these models, see [6, 35]

ã (Erdăos-Renyi) We denote byG(n, p) a random graph onnvertices, generated by drawing an edge between any two vertices with probability p, indepen-dently

(18)

• (Random regular graph) A random regular graph onnvertices with degreed is obtained by sampling uniformly over the set of all simple d-regular graphs on the vertex set {1, , n} We denote this graph byGn,d

It is important to notice that the edges of Gn,d are not independent Because of this, this model is usually harder to study, compared to G(n, p)

We denote byA(n, p) (L(n, p)) the adjacency (laplacian) matrix of the Erdăos-Renyi random graphG(n, p) and byAn,d(Ln,d) the adjacency (laplacian) matrix ofGn,d, respectively

Notation In the whole paper, we assume thatnis large The asymptotic notation

such as o, O,Θ is used under the assumption that n → ∞ We write A B if

A=o(B) cdenotes a universal constant All logarithms have natural base, if not specified otherwise

2 The singular probability

The most famous combinatorial problem concerning random matrices is perhaps the ”singularity” problem Letpnbe the probability thatMnis singular Trivially,

pn ≥2−n,

as the RHS is the probability that the first two rows are equal

By choosing any two rows (columns) and also replacing equal by equal up to sign, one can have a slightly better lower bound

pn≥(4−o(1))

n

2−n = (1

2 +o(1))

n. (1)

It has been conjectured, for quite sometime, that

Conjecture 2.1 [Singularity Conjecture] pn = (12+o(1))n

Conjecture 2.1 is still open, but one can formulate even more precise conjectures (see [4]), based on the following belief

Phenomenon I.The dominating reason for singularity is the dependency between

a few rows/columns

(19)

(see [6]) found a new proof which gave quantitative bound pn =O(n−1/2) In an important paper, Kahn, Koml´os and Szemr´edi [37] proved the first exponential bound

Theorem 2.2 p(n)≤.999n.

Their arguments were simplified by Tao and Vu in 2004 [66], resulting in a slightly better bound O(.958n) Shortly afterwards, these authors [67] combined the ap-proach from [37] with the idea of inverse theorems (see [71, Chapter 7] or [53] for surveys) to obtained a more significant improvement

Theorem 2.3 p(n)≤(3/4 +o(1))n.

With an additional twist, Bourgain, Vu and Wood [9] improved the bound further

Theorem 2.4 p(n)≤(√1

2+o(1))

n.

The method from [67, 9] enables one to deduce bounds on pn directly from simple trigonometrical estimates For instance, the 3/4-bound comes from the fact that

|cosx| ≤

4 +

1

4cos 2x, while the 1/√2 bound come from

|cosx|2=

2 +

1

2cos 2x

[9, Theorem 2.2] provides a formal connection between singularity estimates and trigonometrical estimates of this type, which, while not yet solve the Singularity Conjecture, does lead to sharp bounds in other situations, such as singularity of random matrices with (0,±1) entries)

To conclude this section, let us mention a very useful tool, the Littlewood-Offord-Erdăos theorem Let v = {v1, , vn} be a set of n non-zero real numbers and ξ1, , ξn be i.i.d random Bernoulli variables DefineS :=Pni=1ξivi and pv(a) =

P(S=a) and pv = supa∈Zpv(a)

The problem of estimatingpv came from a paper of Littlewood and Offord in the 1940s [45], as a key technical ingredient in their study of real roots of random polynomials Erdăos, improving a result of Littlewood and Offord, proved the following theorem, which we will refer to as the Erdăos-Littlewood-Offord small ball inequality; see [53] for an explanation of this name

Theorem 2.5 (Small ball inequality) Let v1, , be non-zero numbers and ξi

be i.i.d Bernoulli random variables Then

pv≤

n

bn/2c

2n =O(n−

(20)

Theorem 2.5 is a classical result in combinatorics and have many non-trivial ex-tensions with far reaching consequences (see [7, 34, 53], [71, Chapter 7] and the references therein)

To give the reader a feeling about how small ball estimates can be useful in esti-mating pn, let us expose the rows of Mn one by one from top to bottom Assume that the firstn−1 rows are independent and form a hyperplane with normal vector

v = (v1, , vn) Conditioned on these rows, the probability that Mn is singular is

P(X·v= 0) =P(ξ1v1+· · ·+ξnvn= 0), where X= (ξ1, , ξn) is the last row

In Section 3, the reader will see an application of Theorem 2.5 that leads to Koml´os’ original resultpn =o(1) In order to obtain the stronger estimates in Theorems 2.3 and 2.4, one needs to ebstablish Inverse (or structural) Littlewood-Offord theorems, based on the following general principle

Phenomenon II.IfP(X·v= 0)is relatively large, then the coefficientsv1, ,

posses a strong additive structure

These theorems are motivated by inverse theorems of Freiman type in Additive Combinatorics, the discussion of which is beyond the scope of this survey The interested reader is referred to [53] for a detailed discussion

3 A simple proof of Koml´

os’ Theorem

Let us start with a simple fact Here and later Bernoulli vectors mean vectors with coordinates ±1

Fact 3.1 Let H be a subspace of dimension1≤d≤n ThenH contains at most

2d Bernoulli vectors

To see this, notice that in a subspace of dimensiond, there is a set ofdcoordinates which determine the others This fact implies

pn ≤ nX−1

i=1

P(Xi+1∈Hi)≤

nX−1

i=1

2i−n ≤1− 2n

(21)

show that the sum of the last (say) log lognterms contribute at most (say) log1/3n To this, we will exploit the fact that the Hi are spanned by random vectors The following lemma implies the theorem via the union bound

Lemma 3.1 Let H be the subspace spanned by d random vectors, where d ≥

n−log logn Then with probability at least 1−

n, H contains at most

2n

log1/3n

Bernoulli vectors

We say that a set S of dvectors is k-universal if for any set of k different indices ≤i1, , ik ≤n and any set of signs 1, , n (i=±1), there is a vector v in S such that the sign of the ijth coordinate of v matchesj, for all 1≤j≤k

Fact 3.2 Ifd≥n/2, then with probability at least1−1

n, a set ofdrandom vectors

is k-universal, fork:= logn/10

To prove this, notice that the failure probability is, by the union bound, at most

n k

(1− 2k)

d

≤nk(1− 2k)

n/2

≤n−1

It S is k-universal, then any non-zero vector v in the orthogonal complement of the subspace spanned by S should have more than knon-zero vectors (otherwise, there would be a vector in S having positive inner product withv) If we fix such v and let X be a random Bernoulli vector, then by Theorem 2.5,

P(X ∈Span(S))≤P(X·v= 0) =O( k1/2)≤

(22)

Tài liệu tham khảo

[1] N Alon, Eigenvalues and expanders, Combinatorica

6(1986), no 2, 83-96

[2] N Alon and V Milman, λ1- isoperimetric inequalities for

graphs, and supercon- centrators, J Combin Theory Ser. B 38(1985), no 1, 73-88

[3] N Alon and J Spencer, The probabilistic method, 3rd ed.,

John Wiley & Sons Inc., Hoboken, NJ, 2008

[4] R Arratia and S DeSalvo, On the singularity of ran-dom Bernoulli matrices ˜Nnovel integer partitions and lower bound expansions, Ann Comb. 17 (2013), no 2, 251-274 [5] Z Bai and J Silverstein, Spectral analysis of large

dimen-sional random matrices Second edition Springer Series in Statistics Springer, New York, 2010

[6] B Bollobás, Random graphs Second edition, Cambridge Studies in Advanced Mathematics, 73 Cambridge Univer-sity Press, Cambridge, 2001

[7] B Bollobás, Combinatorics Set systems, hypergraphs, families of vectors and combinatorial probability Cam-bridge University Press, CamCam-bridge, 1986

[8] C Bordenave, M Lelarge, and J Salez, The rank of diluted random graphs,Ann Probab. 39 (2011), no 3, 1097-1121 [9] J Bourgain, V Vu and P M Wood, On the singularity probability of discrete random matrices,J Funct Anal.258 (2010), no 2, 559–603

[10] S Brooks and E Lindenstrauss, Non-localization of eigen-functions on large regular graphs, Israel J Math. 193 (2013), no 1, 1–14

[11] Chung, F R K.; Graham, R L.; Wilson, R M Quasi-random graphs.Combinatorica (1989), no 4, 345–362 [12] F Chung, Spectral graph theory, CBMS series, no 92

(23)

[13] K Costello, Bilinear and quadratic variants on the Littlewood-Offord problem,Israel J Math. 194 (2013), no 1, 359–394

[14] K Costello and V Vu, The ranks of random graphs Ran-dom Structures and Algorithm.33 (2008), 269-285

[15] K Costello and V Vu, The rank of sparse random matrices,

Combin Probab Comput. 19 (2010), no 3, 321–342

[16] K Costello, T Tao and V Vu, Random symmetric matrices are almost surely singular, Duke Math J. 135 (2006), no 2, 395–413

[17] Y Dekel, J Lee, and N Linial Eigenvectors of ran-dom graphs: Nodal ran-domains.Approx- imation, Randomiza-tion, and Combinatorial Optimization Algorithms and Tech-niques,pages 436-448, 2008

[18] I Dimitriu and S Pal, Sparse regular random graphs: spectral density and eigenvectors,Ann Probab. 40 (2012), no 5, 2197–2235

[19] A Edelman, E Kostlan and M Shub, How many eigen-values of a random matrix are real? J Amer Math Soc. (1994), no 1, 247–267

[20] A Edelman, Eigenvalues and condition numbers of ran-dom matrices, SIAM J Matrix Anal Appl. 9 (1988), 543 560

[21] P Erdăos, On a lemma of Littlewood and Offord, Bull Amer. Math Soc. 51(1945), 898902

[22] L Erdăos, A Knowles, H-T Yau and J Yin, Spectral statis-tics of Erd?s-RvZnyi graphs I: Local semicircle law, Ann. Probab. 41 (2013)

(24)

[24] L Erdăos, B Schlein and H-T Yau, Wegner estimate and level repulsion for Wigner random matrices,Int Math Res. Not IMRN 2010, no 3, 436479

[25] L Erdăos and H-T Yau, Universality of local spectral statis-tics of random matrices, Bull Amer Math Soc. (N.S.) 49 (2012), no 3, 377–414

[26] J Friedman On the second eigenvalue and random walks in random d-regular graphs Technical Report CX-TR-172-88, Princeton University, August 1988

[27] J Fiedman, A proof of Alon’s second eigenvalue conjec-ture and related problems (English summary)Mem Amer. Math Soc.195 (2008), no 910, viii+100 pp

[28] J Friedman and D-E Kohler, The Relativized Second Eigenvalue Conjecture of Alon,preprint

[29] Z F ăuredi and J Komlús, The eigenvalues of random sym-metric matrices,Combinatorica (1981), no 3, 233-241 [30] V L Girko, A refinement of the central limit theorem

for random determinants (Russian) Teor Veroyatnost i Primenen 42 (1997), no 1, 63–73; translation in Theory Probab Appl 42 (1997), no 1, 121–129 (1998)

[31] V L Girko, A central limit theorem for random determi-nants Teor Veroyatnost i Mat Statist. 21 (1979), 35–39, 164

[32] A Guionnet and O Zeitouni, Concentration of the spec-tral measure for large matrices, Electron Comm Probab.5 (2000), 119–136

[33] H Golstein and J von Neumman, Numerical inverting of matrices of high order, Bull Amer Math Soc. 53 (1947), 1021-1099

[34] G Halász, Estimates for the concentration function of com-binatorial number theory and probability, Period Math. Hungar. 8(1977), no 3-4, 197–211

[35] S Janson, T Luczak and A Rucinski, Random

(25)

[36] J Kahn and E Szemerédi, STOC 1989

[37] J Kahn, J Komlós, E Szemerédi, On the probability that a random±1matrix is singular,J Amer Math Soc.8(1995), 223–240

[38] J Komlós, On the determinant of (0,1) matrices, Studia Sci Math Hungar. 2(1967) 7-22

[39] J Komlós, On the determinant of random matrices,Studia Sci Math Hungar. 3(1968) 387–399

[40] M Krivelevich and B Sudakov, Pseudo-random

graphs.More sets, graphs and numbers, 199-262, Bolyai Soc Math Stud., 15, Springer, Berlin, 2006

[41] A Lubotzky, R Phillips, and P Sarnak Ramanujan graphs,Combinatorica, 8(3):261-277, 1988

[42] G.A Margulis , Explicit group-theoretical constructions of combinatorial schemes and their application to the design of expanders and superconcentrators [in Russian] Prob-lemy Peredachi Informatsii 24 (1988), pp 51-60

[43] B.D McKay The expected eigenvalue distribution of a large regular graph.Linear Algebra and its Applications, 40:203-216, 1981

[44] P Mitra, Entrywise bounds for eigenvectors of random graphs Electron J Combin. 16 (2009), no 1, Research Paper 131,

[45] J E Littlewood and A C Offord, On the number of real roots of a random algebraic equation III Rec Math [Mat. Sbornik] N.S. 12, (1943) 277–286

[46] A Litvak, A Pajor, M Rudelson, N Tomczak-Jaegermann, Smallest singular value of random matrices and geometry of random polytopes, Adv Math. 195 (2005), no 2, 491– 523

[47] A Marcus, D Spielman and N Srivastava, Interlacing Families I: Bipartite Ramanujan Graphs of All Degrees,

(26)

[48] K Maples, Symmetric random matrices over finite fields announcement, April 15, 2013, preprint

[49] A Nilli, On the second eigenvalue of a graph,Discrete Math-ematics91 (1991), 207-210

[50] A Nilli, Tight estimates for eigenvalues of regular graphs,

Electronic J Combinatorics11 (2004), N9, 4pp

[51] H Nguyen, On the least singular value of random symmet-ric matsymmet-rices, Electron J Probab. 17 (2012), no 53

[52] H Nguyen, Inverse Littlewood-Offord problems and the singularity of random symmetric matrices, Duke Math J.

161 (2012), no 4, 545–586

[53] H Nguyen and V Vu, Small probability, inverse theorems, and applications, Erdos’ 100th Anniversary Proceeding, Bolyai Society Mathematical Studies, Vol 25 (2013)

[54] H Nguyen and V Vu, Random matrices: Law of the deter-minant, Annals of Probability (2014), Vol 42, No 1, 146-167

[55] M Rudelson, Invertibility of random matrices: norm of the inverse, Ann of Math. (2) 168 (2008), no 2, 575–600 [56] M Rudelson, Lecture notes on non-aymptotic random

ma-trix theory, notes from the AMS Short Course on Random Matrices, 2013

[57] M Rudelson and R Vershynin, The Littlewood-Offord problem and invertibility of random matrices, Adv Math.

218 (2008), no 2, 600–633

[58] M Rudelson and R Vershynin, Delocalization of eigen-vectors of random matrices with independent entries,

preprint

[59] O N Feldheim and S Sodin, A universality result for the smallest eigenvalues of certain sample covariance matri-ces,Geom Funct Anal. 20 (2010), no 1, 88–123

(27)

[61] D Spielman and S-H Teng, D Spielman, S.-H Teng,

Smoothed analysis of algorithms, Proceedings of the Inter-national Congress of Mathematicians, Vol I (Beijing, 2002), 597–606, Higher Ed Press, Beijing, 2002

[62] B Sudakov and V Vu, Local resilience of graphs, Random Structures Algorithms 33 (2008), no 4, 409–433

[63] T Tao and V Vu, A central limit theorem for the deter-minant of a Wigner matrix, Adv Math. 231 (2012), no 1, 74–101

[64] T Tao and V Vu, Random matrices: universal properties of eigenvectors, Random Matrices Theory Appl. (2012), no

[65] T Tao and V Vu, Random matrices: Universality of the local eigenvalues statistics pdf file Acta Math 206 (2011), no 1, 127–204

[66] T Tao and V Vu, On random ±1matrices: Singularity De-terminant,Random Structures Algorithms 28 (2006), no 1, 1–23

[67] T Tao and V Vu, On the singularity probability of random Bernoulli matrices, J Amer Math Soc. 20 (2007), no 3, 603–628

[68] T Tao and V Vu, Inverse Littlewood-Offord theorems and the condition number of random matrices,Annals of Math.

169 (2009), 595-632

[69] T Tao and V Vu, On the permanent of random Bernoulli matrices,Advances in Mathematics 220 (2009), 657-669 [70] T Tao and V Vu, Random matrices: Universality of local

spectral statistics of non-Hermitian matrices, to appear in Annals of Probability

[71] T Tao and V Vu, Additive Combinatorics,Cambridge Univ. Press, 2006

(28)

[73] T Tao and V Vu, Random matrices: the distribution of the smallest singular values,Geom Funct Anal.20 (2010), no 1, 260–297

[74] T Tao and V Vu, Random matrices: universal properties of eigenvectors, Random Matrices Theory Appl. (2012), no

[75] L Tran, V Vu and K Wang, Sparse random graphs: Eigen-values and Eigenvectors,Random Structures Algorithms 42 (2013), no 1, 110–134

[76] R Vershynin, Invertibility of symmetric random matrices,

Random Structures and Algorithms 44 (2014), 135–182 [77] E.P Wigner On the distribution of the roots of certain

symmetric matrices.Annals of Mathematics,67(2):325-327, 1958

[78] N.C Wormald, Models of random regular graphs,In Sur-veys in Combinatorics, 1999, J.D Lamb and D.A Preece, eds, pp 239-298

[79] V Vu and K Wang, Random projection, random quadratic

forms, and random eigenvectors, to appear in Random

Structures and Algorithms

(29)

NGÔ QUANG HƯNG

(Đại học Buffalo, Mỹ)

Tóm tắt nội dung

Vĩnh Thức, định thức, định thức Pfaff đa thức đa biến ma trận Chúng có liên hệ mật thiết với nhau, có ứng dụng Vật Lý thống kê, Kinh tế học, toán Tổ hợp, độ phức tạp tính tốn, lý thuyết đồ thị, thuật tốn Bài viết điểm qua lịch sử chứng minh số kết liên kết đối tượng Tổ hợp kỳ thú

1 Vĩnh Thức

GọiA= (aij)là ma trận vngn×n.Vĩnh Thức1 củaAđược

định nghĩa sau:

Perm(A) = X

π∈Sn n

Y

i=1

aiπ(i),

trong đóSn tập tất hốn vị của[n] Như vậy, cơng thức

tính vĩnh thức giống cơng thức Leibniz để tính định thức A, khác điểm ta không nhân sgn(π) vào số hạng tổng

Hàm vĩnh thức có nhiều ứng dụng Ví dụ, vấn đề tốn Tổ hợp tìm số cách lát hình chữ nhật với qn đơ-mi-nơ kích thước 2×1 Mỗi cách lát hoàn hảo tương ứng với bắt cặp hồn hảo2 đồ thị lưới tương ứng (Xem hình 3.1.) Làm để ta đếm tổng số cách bắt cặp hoàn hảo đồ thị lưới? Dễ thấy đồ thị lưới đồ thị hai phần3 Giả

1Permanent Sau thảo luận với anh Phạm Hi Đức Phùng Hồ Hải,

tôi định chọn từ vĩnh thức để dịchpermanent

(30)

sử đồ thị hai phần có n đỉnh bên trái n đỉnh bên phải (Nếu bên có số đỉnh ta thêm vào cách đỉnh đơn lẻ cho hai bên nhau.) Sau đó, ta xây dựng ma trậnA = (aij)

trong đóaij = đỉnhibên trái có cạnh đến đỉnhj bên phải;

và aij = khơng có cạnh ij đồ thị Khi đó, Perm(A)

bằng tổng số cách bắt cặp hoàn hảo đồ thị – số cách lát đơ-mi-nơ mà ta cần tìm

Hình 3.1: Lợp hình chữ nhật8×5bằng hình đơ-mi-nơ2×1, bắt cặp hồn hảo tương ứng đồ thị lưới

Vấn đề tìm số cách lát đơ-mi-nơ khơng phải tốn giải trí Tổ hợp thông thường Đây vấn đề Vật lý thống kê Hoá học trạng thái rắn [17,16,18] Chúng ta quay lại với mơ hình dimer Vật lý

Vĩnh Thức khởi nguyên năm 1812, nhà Toán học người Pháp Jacques Philippe Marie Binet Augustin-Louis Cauchy định nghĩa Trong kỷ 19 có khoảng chục nhà tốn học nghiên cứu đẳng thức bất đẳng thức liên quan đến vĩnh thức, bao gồm Arthur Cayley Thomas Muir Vương Quốc Anh Cái tên “permanent” có lẽ bắt nguồn từ Cauchy (1812), người thật dùng làm “chết tên” Muir (1882) Tuy nhiên, nghiên cứu vĩnh thức thật “nóng” lên từ năm 1960 đóng góp toán học Hà Lan

1.1 Giả định van der Waerden

(31)

trận ngẫu nhiên kép4 là [35] Trực quan cho thấy ma trận Jn = n1J có lẽ ma trận có vĩnh thức cực tiểu (Ma trận J

là ma trận vngn×n gồm tồn số 1.) Từ đó, bất đẳng thức sau gọi làgiả định van der Waerden5:

Perm(A)≥ n!

nn (3.1)

với ma trận ngẫu nhiên kép A kích thướcn×n Van Lint [37] kể rằng, năm 1969 có lần van der Waerden đến dự buổi hội thảo toán Tổ hợp Ông vốn dân Đại số, làm toán tổ hợp Một diễn giả trẻ lên báo báo vấn đề liên quan đến giả định van der Waerden Van der Waerden giơ tay lên hỏi “giả định nói vậy?” Cuối buổi diễn giả xuống xem bảng tên người đặt câu hỏi

Van Lint, vốn nhà tốn học cừ khơi người Hà Lan, truy vấn van der Waerden xem giả định có xuất phát điểm từ đâu Van der Waerden nhớ lại hồi 1926, ơng nói chuyện với O Schreier, Schreier cho ông biết G A Miller có chứng minh có hệ đại diện chung6 cho các

lớp kề7 trái và các lớp kề phải của nhóm con H của một nhóm hữu hạn G

Van der Waerden quan sát lớp kề trái phân hoạch (R1, , Rn) G, R1 ∪ · · · ∪Rn = G, |Ri| =

|G|/n với i ∈ [n] Bộ lớp kề phải phân hoạch

(C1, , Cn) G, thoả |Cj| =|G|/n với j ∈[n] Hệ đại

diện chung cho hai phân hoạch phần tử

S ={g1, , gn} ⊆Gsao cho|Ri∩S|=|Cj∩S|= 1, với i, j ∈[n]

Bây ta xây dựng ma trậnA= (aij)trong đóaij =|Ri∩Cj|

thì tồn hệ đại diện chung tương đương với phát biểu

Perm(A) > Do tổng hàng cột A

|G|/n, ta “thường hoá”8 A cách đặt aij = | Ri∩Cj|

|G|/n Khi A

là ma trận ngẫu nhiên kép Ta chứng minh Perm(A) >

d dng bng nh lý Kăonig-Hall Nhng câu hỏi khác

4Doubly stochastic matrix Một ma trận vuông gọi “ngẫu nhiên kép”

nếu khơng âm, tổng hàng cột bằng1

5Van der Waerden conjecture

6Common system of representatives. 7Coset.

(32)

tự nhiên không giá trị nhỏ Perm(A) đạt tới Đây nguồn gốc giả định van der Waerden

Trong “Permanents” (1978 [23]), Henryk Minc ghi hồi ông nộp số báo vĩnh thức kỷ 20 có bình duyệt viên nói “chế tên lố bịch thế”? Giả thiết van der Waerden cuối chứng minh, năm 1981, độc lập hai báo khác G P Ego-rychev [8] D I Falikman [9] Cả hai dùng bất đẳng thức hình học gọi bất đẳng thức Alexandroff-Fenchel [1] Egorychev Falikman giải Fulkerson năm 1982 Năm 2006, Leonid Gurvits [11] chứng minh bất đẳng thức tổng quát với chứng minh ngắn gọn, bao gồm hệ Định lý van der Waerden

1.2 Giả định Minc

Giả định van der Waerden nói chặn vĩnh thức lớp ma trận định Về chặn năm 1963 Minc [22] có giả định sau Gọi A ma trận 01, nghĩa aij ∈ {0,1} với i, j ∈ [n] Gọi mi số số

hàngicủa ma trận, ta cóPerm(A)≤Qni=1(mi!)1/mi Năm 1973

Lev M Brègman [4] chứng minh giả định này, thường gọi Định lý Brègman.9 Năm 1977, nhà toán học lỗi lạc khác người Hà Lan, Alex Schrijver, trình bày chứng minh cực gọn lịch [28] Ngoài ra, Schrijver tổng quát hoá định lý cho ma trận nguyên không âm Dưới ghi lại chứng minh cho trường hợp ma trận

01của Schrijver

Định lý 1.1(Định lý Brègman). GọiA ma trận 01kích thước n×n Gọi mi số số1 hàng i củaA Ta có,

Perm(A)≤

n

Y

i=1

(mi!)1/mi (3.2)

Chứng minh. Cho cặp i, j ∈ [n], gọi Aij ma trận đạt

bằng cách bỏ hàngivà cột j khỏi A Từ định nghĩa (và từ giả

9Đây Brègman củaphân kỳ Brègman(Bregman divergence)

(33)

thiết A ma trận 01), dễ thấy

Perm(A) = X

j:aij=1

Perm(Aij), với i∈[n] (3.3)

(Khai triển giống khai triển Laplace tính định thức.) Do vế phải (3.2) tích, ta thử chặn Perm(A) cách chuyển vế phải (3.3) thành tích hàm củaPerm(Aij), sau áp dụng quy nạp Cách tự nhiên

để chặn tổng tích dùng bất đẳng thức Jensen Cụ thể hơn, từ tính lồi hàm xlnx miền x > 0, bất đẳng thức Jensen cho ta biết

t1+· · ·+tm

m

ln

t1 +· · ·+tm

m

≤ t1lnt1+m· · ·tmlntm,

miễn ti > với i ∈ [m] Từ đó, ta chặn tổng

tích:

(t1+· · ·+tm)

t1+···+tm

≤mt1+···+tmtt1

1 · · ·t

tm m

Và dễ thấy bất đẳng thức với ti ≥ 0,

chứ không cầnti >0như trước Áp dụng vào (3.3), ta

một chặn cho Perm(A) mà vế phải tích:

Perm(A)Perm(A) ≤mPermi (A) Y

j:aij=1

Perm(Aij)Perm(Aij) (3.4)

Bất đẳng thức (3.4) với i ∈ [n] Bước tự nhiên ta nhân chúng với để có vế phải đối xứng, vế phải “trung hoà” lẫn kích thước chúng khác Nói cách khác, ta dùng trung bình nhân vế phải:

Perm(A)nPerm(A)

n

Y

i=1

mPermi (A)

! ·   n Y i=1 Y

j:aij=1

Perm(Aij)Perm(Aij)

  (3.5) Phát triển tự nhiên áp dụng bất đẳng thức (3.2) cho

Perm(Aij) bên vế phải Tuy nhiên, điều phiền phức có Perm(Aij) số mũ – áp dụng vào làm loạn vế phải Cho

nên, ta tìm cách viết lại thừa số thứ hai bên vế phải (3.5) chút Định nghĩa,

(34)

Sij =

π ∈S | π(i) =j

Từ định nghĩa vĩnh thức, dễ thấy |S| = Perm(A)

|Sij| = Perm(Aij) aij = Đây “diễn giải tổ hợp”

hàm vĩnh thức Ngoài ra, aij = |Sij| = Do đó, ta có

thể viết lại thừa số thứ hai bên vế phải (3.5) sau:

n

Y

i=1

Y

j:aij=1

Perm(Aij)Perm(Aij) =

n

Y

i=1

Y

j:aij=1

Perm(Aij)|Sij|

=

n

Y

i=1

Y

j:aij=1

Perm(Aij)|Sij|

Y

j:aij=0

Perm(Aij)|Sij|

= n Y i=1 n Y j=1

Perm(Aij)|Sij|

Đến ta dùng mẹo “định trị đại lượng hai cách”

cực kỳ phổ dụng toán Tổ hợp Trong vế phải đẳng thức trên, với cặp (i, j) Perm(Aij) xuất |Sij| lần Bây

tưởng tượng ta xây dựng đồ thị hai phần mà đỉnh bên trái cặp (i, j), đỉnh bên phải hốn vị π ∈ S Sau đó, ta nối đỉnh (i, j) bên trái với đỉnh π bên phải π(i) = j Với cạnh ta cho “cân nặng”

Perm(Aij), tích

Qn i=1

Qn

j=1Perm(Aij)|Sij| tích

cân nặng cạnh tưởng tượng nhóm theo đỉnh bên trái đồ thị hai phần Thế thì, cách khác để nhóm tích cân nặng lại với nhóm theo đỉnh π∈S bên phải Với đỉnhπ ta lấyn cạnh kề với nó, cạnh nối π với cách đỉnh (i, π(i))bên trái Như ta suy

n Y i=1 n Y j=1

Perm(Aij)|Sij| =

Y

π∈S n

Y

i=1

Perm(Aiπ(i))

Còn thừa số vế phải (3.5) dễ viết lại thành

n

Y

i=1

mPermi (A) =

n

Y

i=1

m|iS|=Y

π∈S n

Y

i=1

mi

Đến đây, ta viết lại tồn bất đẳng thức (3.5) dạng dễ chịu nhiều:

Perm(A)nPerm(A)≤ Y

π∈S n

Y

i=1

(35)

Vế phải (3.6) cân đối, đến lúc ta áp dụng giả thiết quy nạp rồi:

n

Y

i=1

Perm(Aiπ(i))≤

n Y i=1     Y

k6=i akπ(i)=1

((mk−1)!)1/(mk−1)

Y

k6=i akπ(i)=0

(mk!)1/mk

    (3.7) Vế phải bất đẳng thức trơng kinh dị, lại có dạng đơn giản Nó tích “cân nặng” cặp i 6= k Cân nặng ((mk − 1)!)1/(mk−1) akπ(i) = 1,

(mk!)1/mk akπ(i)= Vế phải (3.7) nhóm tích theo

i Ta nhóm theok (nhớ dùng đồ thị hai phần tưởng tượng – bên trái lài, bên phải k, nối (i, k)nếu i6=k) Nhóm theok có lợi cân nặng cặpi6=k phụ thuộc vào k Lưu ý π ∈ S, akπ(k) = Do đó, số i

sao cho i 6= k akπ(i) = mk −1, số i cho

i6=k vàakπ(i) = n−mk Do đó, vế phải (3.7) n

Y

k=1

(mk−1)!·(mk!) n−mk

mk

Kết hợp với (3.6) ta có

Perm(A)nPerm(A) ≤ Y

π∈S

" n Y i=1 mi # · " n Y k=1

(mk−1)!·(mk!) n−mk

mk

#!

= Y

π∈S n

Y

k=1

(mk!)n/mk

!

=

n

Y

k=1

(mk!)1/mk

!nPerm(A)

Đó chứng minh tuyệt vời Alex Schrijver

1.3 Độ phức tạp tính tốn

(36)

tính Perm(A) ma trận 01 vấn đề #P-khó [33] Đây cơng trình cột mốc mang đến giải Turing năm 2010 cho Valiant Ơng có hai trai Greg Valiant (giáo sư khoa Máy tính Stanford), Paul Valiant (giáo sư khoa Máy tính Brown)

Ở mô tả rất nôm na kết Valiant Một

vấn đề định10 là vấn đề mà câu trả lời là hoặc khơng tồn lời giải LớpP lớp vấn đề định mà ta tìm lời giải trả lời không tồn lời giải thời gian đa thức Ví dụ, cho đồ thịG= (V, E), vấn đề bắt cặp lớn nhất11 hỏi G có tập cạnh khơng giao có kích thước k hay không Đây vấn đề định, thuật toán trổ hoa12 của Edmonds năm 1965 giải thời gian O(|V|4) [7] Lớp NP

là lớp vấn đề định mà việc xác minh lời giải có thật lời giải hay khơng làm nhanh thời gian đa thức, việc tìm lời giải chưa Ví dụ, tơ màu đồ thị G nói “đây tơ màu hợp lệ dùng màu” ta dễ dàng kiểm tra thời gian đa thức xem người có thành thật hay không Nhưng cho trước đồ thị G ta khơng biết cách để trả lời câu hỏi “có tồn cách tơ G dùng màu hay không?” thời gian đa thức Dễ thấy P⊆NP, chứng minh P6=NP

câu hỏi triệu đô Hiện nay, để chứng minh vấn đề định khó (nghĩa khó có khả tồn thời gian đa thức để giải nó) thường ta chứng minh khó tất vấn đề NP Cụ thể chút, ta chứng minh trả lời câu hỏi “Gcó tơ màu khơng?” cách hiệu thìtất cảcác vấn đề NP

đều có thuật tốn hiệu để giải Vì thế, vấn đề màu vấn đềNP-khó

Các vấn đề định vấn đề tính tốn bit thơng tin: bit (trả lời khơng) bit (trả lời ) Đây giải pháp kỹ thuật để chứng minh độ khó Nhưng tất nhiên vấn đề thực tế không vấn đề mà output có bit thơng tin Ví dụ ta cần tính xác suất mà mạng máy tính liên thơng, cho biết trước xác suất lỗi kết nối Bài toán gọi

10Decision problem.

(37)

vấn đề độ tin cậy13 của mạng Hoặc, ta cần tính Perm(A) ma trận A cho trước Rõ ràng thực tế có hàng tỉ vấn đề Lớp#P Valiant định nghĩa để nắm bắt độ khó việc tính số (thay bit thơng tin): #P lớp vấn đề mà câu trả lời số lời giải vấn đềNP Ví dụ, vấn đề tìm số cách tơ đồ thị dùng 3màu hay vấn đề tìm số tập cạnh khơng giao với kích thước n/2 vấn đề lớp#P

Rõ ràng ta đếm số lời giải, ta định có lời giải hay khơng Do đó, vấn đề trong#Pkhó vấn đề tương ứng trongNP Đếm số cách tơ3màu khó xác minh xem có cách tơ màu hay khơng Đếm số cách bắt cặp hồn hảo khó xác minh xem có cách bắt cặp hồn hảo hay khơng Tương tự NP-khó, #P-khó lớp vấn đề mà – ta giải vấn đề lớp cách hiệu quả, ta giải tất cả vấn đề lớp #P cách hiệu quả, ta giải đượctất cảcác vấn đề NP

một cách hiệu

Tóm lại, thuật tốn thời gian đa thức cho vấn đề

#P-khó dẫn đến hệ P=NP Trong ngữ cảnh viết điều có nghĩa khó có khả tồn thuật tốn tính Perm(A) thời gian đa thức – cho dù ma trận A ma trận 01 Khơng tính hiệu ta có hai chọn lựa tự nhiên: tìm thuật tốn xấp xỉ (trong thời gian đa thức), hai tìm lớp ma trận A mà tồn thuật toán hiệu Chọn lựa thứ hai dẫn ta đến câu hỏi Pólya khái niệm định thức Pfaff Tuy nhiên, trước thảo luận câu hỏi Pólya định thức Pfaff, ta cần chuẩn bị kiến thức hốn vị

2 Vài quan sát hoán vị

Cho hốn vị π ∈ Sn Ta biểu diễn π thành dạng đồ

thị có hướng, với tập đỉnh[n], cạnh (i, π(i)) Đồ thị tập hợp chu trình khơng giao nhau, chu trình phủ tồn bộ[n] (Xem hình 3.2.)

(38)

3

6

5

7

1

4

π

= (8 4)

Hình 3.2: Hốn vị cấu trúc chu trình Hốn vị có chu trình chẵn

Một nghịch thế14 π cặp (i, j) cho i < j π(i) > π(j) Gọi Inv(π) tổng số nghịch củaπ Dấu củaπ định nghĩa

sgn(π) = (−1)Inv(π)

Một chuyển vị kề15 là hoán vịπ = (1 · · ·i−1 i+ 1i i+ 2· · ·n) với

i∈ [n−1] đó; hốn vị thường viết (i i+ 1) Nếu ta hợp thành π chuyển vị kề (i i+ 1) ta hốn vị

σ=π◦(i i+ 1) = (π(1)· · ·π(i−1)π(i+ 1) π(i) π(i+ 2)· · ·π(n)) Mọi hoán vị σ tuỳ ý đạt đến từ hốn vị π tuỳ ý cách áp dụng nhiều chuyển vị kề Điều tương đương với thuật toán sắp xếp bong bóng16 Trong thuật tốn xếp bong bóng, ta dùng chuyển vị kề để phần tử nhỏ dãy số cho trước “nổi” lên đầu tiên, phần tử nhỏ nhì lên vị trí thứ nhì, vân vân Nghĩa hoán vị chuyển hoán vị đơn vị chuyển vị kề Và hốn vị chuyển hốn vị khác dùng chuyển vị kề

Dễ thấy sgn(σ) =−sgn(π) σ hợp thành π chuyển vị kề Còn chuyển vị kề thay đổi cấu trúc chu trình giao hốnπ nào? Nếu ivài+ 1nằm chu trình π, sau chuyển vị chu trình bị bẻ thành hai chu trình Nếui vài+ nằm hai chu trình khác hai chu trình trộn thành chu trình

14Inversion Còn dịch làcặp trật tự ngược(ban Biên tập). 15Adjacent transposition.

(39)

Các khẳng định vừa cho chuyển vị (i j), khơng thiết chuyển vị kề Từ đó, ta chứng minh tập sau đây, tập toán Tổ hợp đếm

Bài tập 2.1. Chứng minh sgn(π) = (−1)e(π), đó e(π)

là số chu trình chẵn cấu trúc chu trình π (Chu trình chẵn chu trình có số chẵn đỉnh.)

Một tính chất thú vị hốn vị hai hốn vị liên hợp có cấu trúc chu trình giống hệt Cụ thể hơn, hoán vị π σgọi haihoán vị liên hợp17nếu π=τ◦σ◦τ−1 với τ hốn vị Ví dụ, nếuπ = (i i+ 1)◦σ◦(i i+ 1)−1 thì cấu trúc chu

trình π cấu trúc chu trình σ giống y nhau, đổi chỗi i+ Hình 3.3minh hoạ điều

3

6

7

5

1

4

Hình 3.3: Cấu trúc chu trình củaσ = (5 7)◦π◦(5 7)−1,π là hốn

vị Hình3.2

3 Câu hỏi Pólya

Năm 1913, George Pólya [24] quan sát

det

a b c d

=Perm

a b

−c d

,

nghĩa với ma trận 2×2 có cách đổi dấu (vài) phần tử ma trận để biến vĩnh thức thành định thức Pólya thắc mắc điều có với ma trận hay không? Cụ thể hơn, câu hỏi Pólya sau Cho trước ma trận

A= (aij) Có tồn ma trận B= (bij) chobij =±aij, với

mọi i, j ∈[n], , với hốn vị π∈Sn ta có n

Y

i=1

aiπ(i) =sgn(π)

n

Y

i=1

biπ(i) (3.8)

(40)

Trong đó,sgn(π) ∈ {+1,−1} dấucủa hốn vị π, định nghĩa phần trước Theo khai triển Leibniz

det(B) = X

π∈Sn

sgn(π)

n

Y

i=1

bij

Do đó, tồn ma trận B Perm(A) = det(B) Từ nay, ma trậnB thoả tính chất gọi ma trận Pólya

của ma trậnA

Cũng năm đó, Gábor Szego˝18 [31] trả lời là khơng, với mọin ≥3ln tồn ma trậnn×n khơng có ma trận Pólya tương ứng

Bài tập 3.1. Gọi J3 ma trận 3×3 gồm tồn số Chứng

minh rằngJ3 khơng có ma trận Pólya Dựa vào đó, chứng minh

rằng với mọin≥3, tồn ma trậnn×nkhơng có ma trận Pólya tương ứng

Câu hỏi tự nhiên tiếp theo, câu hỏi phần viết, sau:

Câu hỏi 3.2 (Câu hỏi Pólya). Ma trận A phải có ma trận Pólya cho nó?

Giả sử A có ma trận PólyaB Thì ta viết B=A◦L làtích Hadamard A ma trận L = (`ij) Trong bij = aij`ij

với i, j ∈ [n], `ij ∈ {−1,0,1}, `ij 6= aij 6= Từ (3.8)

ta suy ma trận L phải thoả mãn tính chất sau Nếu Qn

i=1aiπ(i)6=

sgn(π)

n

Y

i=1

`iπ(i) = 1, (3.9)

nghĩa tất số hạng khác khai triển Leibniz định thức Lđều

Một ma trận L với phần tử tập {−1,0,1} gọi

ma trận không kỳ dị dấu19 nếu thoả điều kiện tất các số hạng khác khai triển định thức L có dấu, tồn số hạng khác0 Tập cặp(i, j)sao cho aij 6= gọi giàn tựa20 A Phân tích vừa cho

thấy câu hỏi Pólya tương đương với câu hỏi sau đây:

18Pólya khơng cần phải giới thiệu Bản thân Szego nhà˝

Toán học lớn người Hungary, kèm thêm cho John von Neumann

(41)

Câu hỏi 3.3. Cho trước ma trận A, tồn ma trậnLcó giàn tựa A, vàL ma trận không kỳ dị dấu

Một điều thú vị là câu hỏi có ứng dụng Kinh Tế học, Paul Samuelson (Nobel Kinh tế, 1970) đặt “Nền tảng phân tích Kinh Tế” năm 1947 [27] Để tìm cách trả lời câu hỏi 3.3, ta xếp lại thành tốn Tổ hợp Trước hết, xây dựng đồ thị hai phần G = (R∪C, E), R= [n] tập hàng củaA,C = [n]là tập cột A, có cạnh (i, j) ∈ E đồ thị G aij 6= Mỗi ma trận L có giàn tựa A tương

ứng với phép gán hệ số {−1,+1} vào cạnh đồ thị G Bất kể ta gán hệ số nào, khai triển định thức củaLcó số hạng khác0nếu tồn bắt cặp hồn hảo đồ thị G; số hạng khác khai triển định thức tương ứng với bắt cặp hồn hảo Như vậy, khơng tính tổng qt ta giả sử G có bắt cặp hồn hảo Bắt cặp hồn hảo (nếu có) đồ thị hai phần G tính nhiều thuật tốn, ví dụ thuật tốn nới dài đường dẫn21 hoặc thuật toán Hopcroft–Karp [12] với thời gian chạy O p|V(G)||E(G)|=O n5/2.

Gọi π ∈ Sn bắt cặp hoàn hảo G, nghĩa cạnh

(i, π(i))thuộc bắt cặp Nếu ta xáo trộn hàng cột ma trận L ta khơng thay đổi tính chất khơng kỳ dị dấu Do đó, khơng tính tổng quát ta giả sử π(i) =i cách đánh số lại đỉnh tập R Còn nữa, ta nhân hàng cột củaL với giá trị

−1thì ta khơng thay đổi tính chất khơng kỳ dị dấu

L Do đó, ta giả sử phép gán hệ số vào cạnh Ggán số 1cho tất cạnh (i, i) củaG

Tiếp tục với hành trình xếp lại tốn Bây ta xây dựng đồ thị có hướng D = ([n], E(D)) hai bước: (1) định hướng tất cạnh(i, j)∈E(G),i∈R,j ∈C, cách đổi thành mũi tên từ i đến j; (2) sau nhập đỉnh i∈R đỉnh i∈C củaG làm Sau làm điều đồ thị Dcó n đỉnh, đỉnhi có vịng22 mũi tên trỏ

21Augmenting path algorithm Cịn gọi làThuật tốn tìm đường tăng

(luồng)(ban Biên tập)

(42)

từ i đến i Tại vì, G có mũi tên từ i ∈ R vào i ∈ C Ngồi ra, hốn vị π ∈ Sn cho (i, π(i)) ∈ E(G),∀i ∈ [n],

tương ứng với chu trình khơng giao đồ thị D, chu trình phủ toàn đỉnh D Một tập chu trình (có hướng) D gọi phủ chu trình23 củaD

Tóm lại, tốn ta chuyển từ gán hệ số {−1,1} vào cạnh vô hướng G thành việc gán hệ sốw:E(D)˜o{−1,1} vào cạnh có hướng củaD Với phủ chu trìnhπ, định nghĩa “cân nặng”

w(π) = sgn(π)

n

Y

i=1

w(i, π(i))

(Ở ta lạm dụng ký hiệu, dùng luônwđể ký hiệu hàm cân nặng phủ chu trình.) Và ta muốn tìm phép gán hệ số cho w(π) = 1, với phủ chu trìnhπ củaD Thật tốn địi hỏi số hạng dấu, π(i) = i phủ chu trình, cân nặng bằng1, nên ta biết tất cân nặng phủ chu trình phải bằng1

Mặc dù chuyển vấn đề từ ma trận vấn đề đồ thị phần dễ hình dung hơn, cách để (bằng thuật tốn) kiểm tra xem có phép gán hệ số mong muốn hay không Vazirani Yannakakis [40] lại xếp tiếp toán

Bổ đề 3.4(Vazirani-Yannakakis, 1988). Cho đồ thị có hướng D = ([n], E) với n vòng (i, i) ∈ E Tồn phép gán hệ số w : Eo˜{−1,1} cho tất phủ chu trình D có cân nặng tồn phép gán hệ số w: Eo˜{−1,1} cho, với chu trình C D, có số lẻ cạnh C với hệ số

Chứng minh. Giả sử tồn phép gán hệ số cho chu trình C D có số lẻ cạnh với hệ số Như vậy, chu trình lẻ có số chẵn hệ số −1; chu trình chẵn có số lẻ hệ số−1 Gọi πlà phủ chu trình

(43)

bất kỳ củaD Từ Bài tập2.1, ta biết sgn(π) = (−1)c trong đó c là

số chu trình chẵn π Từ đó, dễ thấyw(π) =

Ngược lại, giả sử tồn phép gán hệ số cho tất phủ chu trình có cân nặng là1 Gọi C chu trình D Nếu ta lấy C, với vòng(i, i)với i /∈C, ta có phủ chu trìnhπ Nếu C chu trình lẻ, thìsgn(π) = – π khơng có chu trình chẵn Vì thế, phải có số chẵn hệ số −1 chu trình C Ngược lại, C chu trình chẵn thìsgn(π) = −1 Vì thế, phải có số lẻ hệ số−1trênC Đến vấn đề rõ ràng chút Tuy nhiên, kể cho ta phép gán hệ số ta cách để kiểm tra cách hiệu xem phép gán hệ số có tốt hay khơng Tại vì, tổng số chu trình đồ thị cho trước làm hàm mũ Ta khơng thể kiểm tra chu trình Đó chưa nói đến chuyện phải qua tất phép gán hệ số {−1,1}

Định nghĩa 3.5. Một đồ thị có hướng D gọi đồ thị chẵn

nếu, với phép gán hệ số {−1,1} vào cạnh D, tồn chu trình củaD có số chẵn hệ số

Bổ đề Vazirani-Yannakakis cho ta biết câu hỏi Pólya tương đương với câu hỏi kiểm tra xem đồ thị có hướng có phải đồ thị chẵn hay khơng Năm 1987, Paul Seymour Carsten Thomassen [29] nghiên cứu ma trận không kỳ dị dấu, chứng minh rằng, tồn đồ thị có hướng H cho Dlà đồ thị chẵn H có chu trình với số chẵn cạnh Và H xây dựng từ D thời gian đa thức Như vậy, ta có

Định lý 3.6. Câu hỏi Pólya tương đương mặt thuật toán với toán xác minh xem đồ thị có hướng H có chu trình với sỗ chẵn cạnh hay không

Chứng minh định lý khơng tầm thường, khơng phải q khó Xem thêm phần Chú Thích cuối tham khảo từ Câu hỏi Pólya trở thành câu hỏi rõ ràng mặt tổ hợp, ta hồn tồn khơng biết cách trả lời câu hỏi thời gian đa thức

(44)

thức Bài báo tờ Annals of Mathematics Như sau 86 năm, câu hỏi Pólya trả lời thoả đáng Khơng thế, đề cập phần tới bài, câu hỏi có liên quan mật thiết đến đối tượng đại số tuyến tính khác –định thức Pfaff – quyến rũ không định thức vĩnh thức

4 Định thức Pfaff

Cũng vĩnh thức định thức, định thức Pfaff đa thức đa biến, với biến phần tử ma trận vuông Trong phần viết, tơi giải thích Định thức Pfaff liên quan đến định thức, đến tổng số cách bắt cặp hoàn hảo đồ thị cho trước, đo liên đới đến vĩnh thức

4.1 Định thức Pfaff

Gọi n số nguyên dương chẵn Gọi Fn tập tất

phân hoạch của[n] thành n/2 cặp số Như vậy, thành viên F ∈ Fn tập n/2cặp(i, j),i < j, cho tất số

[n] điều thuộc cặp F Hai cặp(i, j),(i0, j0)∈ F gọi hai cặpcắt nhau24 nếu

i < i0 < j < j0

Đơn giản ta vẽ hai cung từiđếnj từ i0 đếnj0 chúng cắt nhau, hình sau

i i0 j j0

Gọiχ(F)là tổng số cặp cắt F

Định thức Pfaff định nghĩa sau Gọi A = (aij)

ma trậnphản xứng25, nghĩa A=−AT, thìđịnh thức Pfaff của A, ký hiệu Pf(A), định nghĩa

Pf(A) = X

F∈Fn

(−1)χ(F) Y (i,j)∈F

aij (3.10)

24Crossing.

(45)

Bài tập 4.1. Có cách khác để định nghĩa “dấu” thành viên F ∈ Fn Giả sử F =

{i1, j1},· · · {in/2, jn/2} ,

i` < j` với ` ∈[n/2] Gọi π hoán vị sau

π=

1 · · · n−1 n

i1 j1 i2 j2 · · · in/2 jn/2

Chứng minh (−1)χ(F) =sgn(π) (Trong số sách bạn đọc

sẽ thấy họ dùng sgn(π) thay (−1)χ(F) để định nghĩa định thức

Pfaff.)

Theo “Chứng minh khẳng định”26 của David M Bres-soud [5] tổng dạng định thức Pfaff xuất hiên từ báo Johann Friedrich Pfaff từ năm 1815 Ơng mơ tả phương pháp giải hệ 2n phương trình vi phân bậc cách dùng biến phương trình phụ trợ Các phương trình phụ trợ gồm tổng số tỉ lệ mà tử số định thức Pfaff Lúc Pfaff gần 50 tuổi, cơng trình ơng khơng đọc rộng rãi Carl Gustav Jacob Jacobi viết phương pháp Pfaff Định thức Pfaff liên hệ mật thiết đến ma trận phản xứng Các người khổng lồ Poisson, Lagrange, Laplace, Monge làm việc với ma trận nửa đầu kỷ 18 Nhưng phải đến Ja-cobi nhận liên hệ định thức Pfaff định thức ma trận Mà kể vậy, phải chờ đến Arthur Cayley (1847) người ta biết đến đẳng thức quan trọng det(A) = Pf(A)2 Trên Wikipedia nói đẳng thức do

Thomas Muir chứng minh sách ông định

thức năm 1882 Nhưng tin Bressoud đúng! Duyệt nhanh qua sách Muir khó khẳng định ơng khơng viết theo kiểu chứng minh

Chúng ta chứng minh đẳng thức dùng phương pháp tổ hợp John Stembridge [30]; báo thật tuyệt hảo toán tổ hợp đếm

Định lý 4.2 (Cayley, 1847). Gọi A ma trận phản xứng n×n với n chẵn Ta có,

det(A) =Pf(A)2

(46)

Chứng minh John Stembridge. Trước hết, viết lại vế trái đẳng thức dùng thêm kiến thức làAlà ma trận phản xứng Khi ta khai triển định thức

det(A) = X

π∈Sn

sgn(π)

n

Y

i=1

aiπ(i)

thì có nhiều cặp số hạng bù trừ lẫn A ma trận phản xứng Cụ thể hơn, giả sử có chu trìnhC củaπ chu trình lẻ (như chu trình (1 4) Hình 3.2) Gọi π0 hốn vị có chu trình giống hệt π, ngoại trừ chu trình lẻ C bị đảo chiều Khi đó, sgn(π) = sgn(π0), Qni=1aiπ(i) = −Qni=1aiπ0(i)

Như hai số hạng tương ứng với π π0 bù trừ lẫn Ta bắt cặp hốn vị trongSnbằng cách Nếuπ có

chu trình lẻ, ta lấy chu trình lẻ có chứa số bé chu trình lẻ, đảo chiều chu trình lẻ để lấyπ0 Đây bắt cặp hoàn hảo hốn vị có (ít một) chu trình lẻ Do đó, gọi En⊆Sn tập tất hốn vị của[n]với tồn chu

trình chẵn, ta có

det(A) = X

π∈En

(−1)e(π)

n

Y

i=1

aiπ(i) (3.11)

(Nhớ rằng, định nghĩa Bài tập 2.1, e(π) số chu trình chẵn củaπ.) Bây ta viết lại vế phải đẳng thức cần chứng minh

Pf(A)2 =

X

F∈Fn

(−1)χ(F) Y (i,j)∈F

aij

 ·

 X

F0∈Fn

(−1)χ(F0) Y (i0,j0)∈F0

ai0j0

 

= X

(F,F0)∈Fn×Fn

(−1)χ(F)+χ(F0)

 Y

(i,j)∈F

aij

 ·

 Y

(i0,j0)∈F0

ai0j0

 

= X

(F,F0)∈Fn×Fn

(−1)χ(F)+χ(F0) Y (i,j)∈F∪F0

aij

Kế hoạch tìm song ánh En Fn× Fn cho, với

mỗi π∈En có cặp (F, F0) tương ứng (và ngược lại)

để có đẳng thức sau xong:

(−1)e(π)

n

Y

i=1

aiπ(i) = (−1)χ(F)+χ(F

0) Y

(i,j)∈F∪F0

(47)

Tìm song ánh En Fn× Fn điều dễ hàng Một

cặp (F, F0) ∈ Fn× Fn cặp (có thứ tự) bắt cặp hồn hảo

các số [n] Ta gọi cạnh F cạnh xanh cạnh củaF0 cạnh đỏ Có tất cản/2cạnh xanh,n/2 cạnh đỏ, tập cạnh màu bắt cặp hoàn hảo Khi ta vẽ cạnh xanh đỏ vào đồ thị với[n]là tập đỉnh, đỉnh có bậc Cụ thể đỉnh kề với cạnh xanh cạnh đỏ Do đó, tập n cạnh tạo thành phủ chu trình [n] Mỗi chu trình phủ chu trình có chiều dài chẵn, ta vịng theo chu trình ta gặp màu xanh đỏ luân phiên

Cho đến ta chưa hốn vị phủ chu trình chưa có hướng Để định hướng cho chu trình ta đỉnh nhỏ chu trình, theo cạnh xanh trước Dễ thấy song ánh, cho trước phủ chu trình π∈En, ta làm ngược lại: từ đỉnh nhỏ chu trình,

và tơ màu xanh đỏ thay phiên cho cạnh Hình 3.4

minh họa song ánh Ta gọi song ánh làsong ánh Stem-bridge

1

Tương ứng với hoán vị

π

= (5 3)

Hình 3.4: Song ánh En Fn× Fn

Như vậy, điều cịn lại để chứng minh chứng minh sgn(π)có “dấu” với cặp ảnh (F, F0)của song ánh Stembridge Vớiπ ∈En, gọir(π) ={i|π(i)< i}thì đẳng thức

cần chứng minh (3.12) tương đương với

e(π) +r(π) = χ(F) +χ(F0) (mod 2) (3.13) Ví dụ, đẳng thức tất nhiên vớiπ= (2 n n−1)

vì đóe(π) =n/2 vàr(π) = n/2

(48)

• Nếuivài+1nằm kề chu trình củaπ, hốn chuyểni i+ ta thay đổir(π) mod 2, giữ ngun e(π) Khơng khó để thấy ta thay đổi χ(F) mod χ(F0) mod 2, khơng thay đổi hai Ví dụ, giữaivài+1có cạnh màu xanh, hốn chuyển vị trí i i+ khơng thay đổi số điểm cắt cạnh màu xanh Còn i i+ kề với hai cạnh màu đỏ khác Nếu chúng khơng cắt cắt sau hoán chuyển, chúng cắt khơng cắt sau hốn chuyển

• Nếuivài+1khơng nằm kề chu trình củaπ hốn chuyển ivài+ 1sẽ giữ r(π)(vàe(π)) nguyên trạng; lần χ(F) vàχ(F0) thay đổi

Như vậy, để chứng minh (3.13) cho π, ta cần chứng minh (3.13) cho σ – miễn ta chuyển π thành σ (nhiều lần) đổi chỗ hai số nguyên liên tiếp Mà cách đổi chỗ hai số nguyên liên tiếp ta chuyển π thành hốn vị mà chu trình thứ C1 chứa số nguyên nhỏ

nhất {1,2, ,|C1|}, chu trình thứ hai chứa số nguyên kế

tiếp, theo thứ tự, vân vân Dễ thấy (3.13) với σ thoả tính chất

4.2 Định hướng Pfaff

GọiA= (aij)là ma trận kề27của đồ thị vô hướng G= (V, E),

nghĩa làaij = nếuij cạnh Mỗi số hạng (−1)χ(F)

Q

ij∈Faij

của khai triển định thức Pfaff A khác F bắt cặp hoàn hảo G Tiếc A ma trận đối xứng ma trận phản xứng Năm 1961, nhà Vật lý Pieter Kasteleyn [14, 15] (người Hà Lan), Harold Neville Vazeille Temperley Michael Fisher [32] (người Anh) nhận ý tưởng đơn giản Giả sử ta định hướng cạnh ij G, gánaij = ioj˜ aij =−1 joi˜, ma trậnA định

hướng G ma trận phản xứng Khi đó, tồn định hướng G cho tất số hạng khác khai triển định thức Pfaff A có dấu, Định Lý 4.2 cho ta cách đếm nhanh số bắt cặp hoàn hảo đồ thị Gdùng định thức! Một định hướng G thoả tính chất

(49)

này gọi định hướng Pfaffian đồ thị Và, thuật toán ta gọi làthuật toán FKT

Hai câu hỏi ta cần trả lời là:

• Khi tồn định hướng Pfaff choG?

• Nếu biết định hướng Pfaff tồn tại, làm để tìm cách hiệu quả?

Để tìm cách trả lời câu hỏi này, ta tìm miêu tả mang tính tổ hợp định hướng Pfaff Một chu trình C G chu trình tốt có số chẵn đỉnh có bắt cặp hoàn hảo đồ thịG−C Cho định hướng −→G G chu trìnhC gọi chu trìnhđịnh hướng lẻ vịng quanh C theo chiều định số mũi tên xi chiều ta thấy số lẻ Tất nhiên, C chẵn nên ta theo chiều ngược lại số mũi tên xuôi chiều số lẻ

Bổ đề 4.3. Cho −→G định hướng G, −→G định hướng Pfaff tất chu trình tốt G được−→G định hướng lẻ

Chứng minh. Gọi A ma trận kề −→G, nghĩa aij =

(i, j)∈ E(−→G), aij =−1 (j, i) ∈ E(−→G), ngồi aij = Gọi

F F0 hai bắt cặp hồn hảo G, −→G định hướng Pfaff

(−1)χ(F) Y (i,j)∈F

aij = (−1)χ(F

0) Y

(i0,j0)∈F0

ai0j0,

với cặp(F, F0) Đẳng thức tương đương với

(−1)χ(F)+χ(F0) Y

(i,j)∈F∪F0

aij = (3.14)

Lưu ý rằngF, F0 ∈ Fn, (i, j)∈F∪F0 i < j Gọi πlà ảnh

của (F, F0) song ánh Stembridge Từ đẳng thức (3.13), ta suy (3.14) tương đương với

(−1)e(π)+r(π) Y (i,j)∈F∪F0

aij = (3.15)

(50)

tốt Gọi M bắt cặp hồn hảo G−C Ta tách C thành hai bắt cặp C1 C2 cách vòng theo C, bỏ

một cạnh vàoC1, cạnh kế vào C2, luân phiên Định nghĩa

F =M ∪C1 F0 =M ∪C2, F F0 hai bắt cặp hoàn hảo

của G Gọi π ảnh cặp (F, F0) qua song ánh Stembridge, phân tích đẳng thức (3.15) thoả mãn Gọi V(C)là tập đỉnh C, vàE(C)là tập cạnh Định nghĩa

r(C) = |{i∈V(C) | π(i)< i}| Dễ thấy

e(π) = |M|+

r(π) = |M|+r(C)

Định nghĩa

s(i, j) =

(

1 i < j

−1 j ≥i

Do Alà ma trận phản xứng, nếui < j ta có aij =s(i, j)aij

j < ita có aji =s(i, j)aij Vì thế,

Y

(i,j)∈F∪F0

= Y

ij∈E(C)

i<j

aij ·

Y

ij∈M

a2ij

= Y

i∈V(C)

s(i, π(i))·aiπ(i)

= (−1)r(C) Y

i∈V(C)

aiπ(i)

Các đẳng thức vừa rồi, cộng với đẳng thức (3.15) dẫn đếnQi∈V(C)aiπ(i) =

−1, nghĩa làC định hướng lẻ

Cho chiều ngược lại, giả sử chu trình tốt định hướng lẻ Ta chứng minh đẳng thức (3.15) với cặp bắt cặp hoàn hảoF vàF0 Dễ thấy rằngF∪F0 hội củak chu trình tốt C1∪ · · · ∪Ck Tương tự trên,

e(π) +r(π)≡

k

X

`=1

(51)

Y

(i,j)∈F∪F0

=

k

Y

`=1

(−1)r(C`) Y

i∈V(C)

aiπ(i)

 =

k

Y

`=1

(−1)r(C`)·(−1)

Do (3.15) chứng minh

Bạn đọc hẳn nhận tương đồng tốn tìm định hướng Pfaff đồ thị câu hỏi Pólya Điều khơng phải ngẫu nhiên Hai tốn tương đương với (về mặt thuật toán) G đồ thị hai phần; xem chứng minh Vazirani Yannakakis [40]

Tuy nhiên, kết FKT bước theo hướng khác, đẹp Họ chứng minh tồn định hướng Pfaff G làđồ thị phẳng,28 nghĩa ta vẽ Gtrên mặt phẳng sao cho khơng có cạnh giao cạnh

Định lý 4.4 (FKT, 1961). Nếu G đồ thị phẳng ln tồn định hướng Pfaff cho

Chứng minh. Ta xét cách vẽ G mặt phẳng tuỳ ý.29 Khơng tính tổng qt ta giả sử G đồ thị liên thông Đồ thị G chia mặt phẳng thành nhiều mặt30, phần vơ hạn ngồi G gọi mặt vơ hạn G Các mặt lại mặt hữu hạn Nếu G khơng phải có mặt hữu hạn, mặt vô hạn ráp gianh với số mặt hữu hạn Xem Hình3.5

Trước hết, ta chứng minh tồn cách định hướng cạnh G cho điều sau đúng: ta theo chiều kim đồng hồ vòng quanh mặt hữu hạn bất kỳ, tổng số mũi tên xuôi chiều ta gặp số lẻ Tạm gọi tính chất tính chất lẻ địa phương mặt Lưu ý rằng, ta làm việc với cách vẽ G cụ thể mặt phẳng, ta dùng “thuận chiều kim đồng hồ” mà không ngượng mồm Với đồ thị chưa vẽ mặt phẳng, khái niệm “thuận chiều kim đồng hồ” khơng có định nghĩa rõ ràng

(52)

e

F1

F2

F3

F4

Mặt vô hạn F5

Công thức Euler: 12−15 + =

Hình 3.5: Một đồ thị phẳng, 12đỉnh, 15cạnh, 5mặt Ta chứng minh khẳng định quy nạp Nếu Glà tất nhiên điều khơng có mặt hữu hạn Nếu G Gọi e cạnh nằm biên giới mặt vô hạn mặt hữu hạn (ví dụ cạnh e Hình3.5) Theo quy nạp ta định hướng tất cạnh G cho thoả tính chất “lẻ địa phương” Bây giờ, ta ráp cạnh e vào lại định hướng e cho mặt hữu hạn chứa e lẻ địa phương

Kế đến, ta chứng minh định hướng lẻ địa phương định hướng Pfaff Theo Bổ Đề 4.3, ta cần chứng minh chu trình tốt −→G định hướng lẻ Gọi C chu trình Giả sử C phủ f mặt hữu hạn bên Gọi wC số cạnh thuận chiều kim đồng hồ vòng theo C Với

mỗi mặt hữu hạnFi,i∈[f], bên trongC, gọiwi số cạnh thuận

chiều kim đồng hồ mặt Thì ta có wi lẻ với i∈[f];

do đóPfi=1wi ≡f (mod 2) Trong tổng

P

iwi cạnhbên trongC đếm lần, thuận chiều với mặt kề ngược chiều với mặt bên Do đó, ta gọieC

tổng số cạnh chu trình C, eI tổng số cạnh bên

C, ta có

f

X

i=1

wi ≡wC +eI ≡wC + (eI+eC) (mod 2)

(53)

vì G−C có bắt cặp hồn hảo; nghĩa đỉnh bên C bắt cặp hồn hảo với Theo cơng thức Euler v−(eC +eI) + (f + 1) = (Vì ta phải đếm mặt vô hạn

công thức Euler.) Do

wC ≡

f

X

i=1

wi−(eI+eC)≡f −(eI +eC)≡v−1≡1 (mod 2)

Công thức sau công thức tốn Tổ hợp đếm mà số ngun (mình cần đếm) lại biểu diễn thành hàm số vô tỉ Cơng thức Fibonacci ví dụ khác Nhưng cơng thức “ngầu” có lẽ cơng thức Ramanujan đếm tổng số phân hoạch số nguyên – bao gồm tích phân Cauchy miền số phức

Định lý 4.5. Gọi G đồ thị lưới m×n với mn số chẵn Thì, tổng số cách bắt cặp hoàn hảo củaG

2mn2

m Y k=1 n Y `=1 cos2 πk

m+

+ cos2

π`

n+

1

4

Chứng minh. Khơng tính tổng qt, ta giả sử n chẵn Định hướng Hình 3.6 thoả mãn tính chất mặt hữu hạn định hướng lẻ Do đó, định hướng Pfaff đồ thị lưới Ta đánh số đỉnh hình;

1 17 25 33 10 18 26 34 11 19 27 35 12 20 28 36 13 21 29 37 14 22 30 38 15 23 31 39 16 24 32 40

(54)

mỗi hàng có n = đỉnh cột có m = đỉnh Ma trận kề định hướng viết dạng sau

A=         

X In

−In −X In

−In X In

−In −X In

In

−In (−1)m−1X

         đó,X ma trận n×n có dạng:

X=         

−1

−1

−1

−1

        

Ma trậnXlà ma trận kề hàng đồ thị lưới Hàng thứ mũi tên trỏ từ nhỏ đến lớn, ta có X Hàng thứ hai có mũi tên trỏ ngược lại, ta có−X, luân phiên Ma trận I ma trận đơn vị, kề đỉnh hàng cạnh ma trận lưới

Chúng ta cần tính det(A) Để đơn giản hố tốn, ta viết lại

A chút Nếu ta nhân hàng A với −1 cột với −1 ta thay đổi dấu det(A) không thay đổi trị tuyệt đối Ta tìm cách nhân ma trận

A đối xứng Các ma trận đối xứng dễ tìm trị đặc trưng Và định thức tích trị đặc trưng

Để biến A thành ma trận đối xứng, ta đổi dấu khối n cột thứ A, đổi dấu khối n hàng thứ ba thứ tư, đổi dấu khối n cột thứ tư thứ năm, đổi dấu khối hàng thứ thứ 8, vân vân Ta đạt ma trận

M=         

−X In

In −X In

In −X In

In −X In

In

In −X

(55)

Định nghĩa Y ma trận m×m Y=          1

1 1

1         

Thì dễ thấy

M =−Im⊗X+Y⊗In

trong ⊗ tích Kronecker hai ma trận Ma trận M gọi tổng Kronecker hai ma trận −X Y Nhờ Bài tập4.6, tác vụ ta tìm trị đặc trưng củaX

Y

Cả X Y ma trận Toeplitz có ba đường chéo Theo tập4.7thì Y có trị đặc trưng cos kπ

m+1,k ∈[m], Xcó

bộ trị đặc trưng là2icos `π

n+1, `∈[n]

Như vậy, với mỗik ∈[m] và` ∈[n] ta có trị đặc trưng choM

là cos kπ

m+1 −icos

`π n+1

Do cos `π

n+1 = −cos

(n+1−`)π

n+1 , nhân tất

cả trị đặc trưng với ta

2mn m Y k=1 n Y `=1

cos2 kπ

m+ + cos

2 `π

n+

1/2

Bài tập 4.6. Gọi A ma trận n×n, B ma trận m×m, α trị đặc trưng củaA, vàβ trị đặc trưng B Chứng minh α+β trị đặc trưng tổng Kronecker

A⊕B:=Im⊕A+B⊕In

Bội trị đặc trưng (α+β)là gì?

(56)

phương trình sai phân Ma trậnn×n Z =          a c b a c

b a c

b a c

c

b a          có trị đặc trưng a+ 2√bccos kπ

n+1, ≤ k ≤ n Gợi ý: giả sử

v = (v1, , vn) vector đặc trưng với trị đặc trưng λ Từ

phương trìnhZv=λv, ta có phương trình sai phân

vk+1+vk−1 =λvk, với k ∈[n],

trong điều kiện biên v0 = 0, vn+1 = Từ tìm n

nghiệm λ Hỏi: vectors đặc trưng gì? Có phụ thuộc vào

a, b, ckhơng?

5 Chú thích

Nhiều báo sách giải chứng minh khác giả thiết van der Waerden [39,19, 36,37,38] Các chứng minh ngắn gọn dễ hiểu

Có hai chứng minh khác định lý Brègman, dựa chứng minh Schrijver Một chứng minh dùng phương pháp xác suất sách Alon Spencer [2] Chứng minh dùng entropy có điều kiện Radhakrishnan [25] Tuy nhiên chứng minh không tự nhiên chứng minh Schrijver

Quyển Arora Barak [3] tham khảo đại độ phức tạp tính tốn Đọc thêm Mark Jerrum [13] để biết thêm #P

và cách xấp xỉ vấn đề#P-khó

Quyển sách Lovász Plummer [20] tham khảo tốt định thức Pfaff toán Tổ hợp

William McCuaig [21] liệt kê khoảng 40 câu hỏi tương đương với câu hỏi Pólya

Quyển “Chứng minh khẳng định” David M Bressoud [5] sách tuyệt hảo toán tiếng tổ hợp đếm Trong có mơ tả vai trò định thức Pfaff chứng minh giả định gọi giả định ma trận đảo dấu31.

(57)

Kasteleyn chữ K bất đẳng thức FKG có nhiều ứng dụng tốn Tổ hợp mà tìm dịp viếng thăm Như viết, biết cách kiểm tra thời gian đa thức xem đồ thị hai phần có định hướng Pfaff hay khơng Nhưng toán toán mở cho đồ thị tổng quát: vấn đề nằm trongP hay NP-khó

Định thức Pfaff đóng vai trò quan trọng Holographic al-gorithms [34, 6], mơ hình đồ thị xác suất phẳng học máy [10]

Tài liệu tham khảo

[1] ALEXANDROFF, A ăUber die Oberflăachenfunktion eines kon-vexen Kăorpers (Bemerkung zur Arbeit Zur Theorie der gemischten Volumina von konvexen Kăorpern) Rec Math. N.S [Mat Sbornik] 6(48)(1939), 167–174

[2] ALON, N.,ANDSPENCER, J H.The probabilistic method, sec-ond ed Wiley-Interscience Series in Discrete Mathematics and Optimization Wiley-Interscience [John Wiley & Sons], New York, 2000 With an appendix on the life and work of Paul Erdos.˝

[3] ARORA, S., ANDBARAK, B Computational complexity Cam-bridge University Press, CamCam-bridge, 2009 A modern ap-proach

[4] BRÈGMAN, L M Certain properties of nonnegative matrices and their permanents Dokl Akad Nauk SSSR 211 (1973), 27–30

[5] BRESSOUD, D M Proofs and confirmations MAA Spec-trum Mathematical Association of America, Washington, DC; Cambridge University Press, Cambridge, 1999 The story of the alternating sign matrix conjecture

[6] CAI, J., AND LU, P Holographic algorithms: From art to science J Comput Syst Sci 77, (2011), 41–61

[7] EDMONDS, J Paths, trees, and flowers Canad J Math 17

(58)

[8] EGORYCHEV, G P The solution of van der Waerden’s prob-lem for permanents Adv in Math 42, (1981), 299–305 [9] FALIKMAN, D I Proof of the van der Waerden conjecture on

the permanent of a doubly stochastic matrix Mat Zametki 29, (1981), 931–938, 957

[10] GLOBERSON, A., AND JAAKKOL A, T Approximate inference using planar graph decomposition In Advances in Neu-ral Information Processing Systems 19, Proceedings of the Twentieth Annual Conference on Neural Information Process-ing Systems, Vancouver, British Columbia, Canada, Decem-ber 4-7, 2006(2006), B Schăolkopf, J C Platt, and T Hoff-man, Eds., MIT Press, pp 473–480

[11] GURVITS, L Hyperbolic polynomials approach to Van der Waerden/Schrijver-Valiant like conjectures: sharper bounds, simpler proofs and algorithmic applications In

STOC’06: Proceedings of the 38th Annual ACM Symposium on Theory of Computing ACM, New York, 2006, pp 417– 426

[12] HOPCROFT, J E., AND KARP, R M An n5/2 algorithm for maximum matchings in bipartite graphs SIAM J Comput. 2 (1973), 225–231

[13] JERRUM, M Counting, sampling and integrating: algorithms and complexity Lectures in Mathematics ETH Z ăurich Birkhăauser Verlag, Basel, 2003

[14] KASTELEYN, P W The statistics of dimers on a lattice : I The number of dimer arrangements on a quadratic lattice

Physica 27 (Dec 1961), 1209–1225

[15] KASTELEYN, P W Dimer statistics and phase transitions

J Mathematical Phys 4(1963), 287–293

[16] KENYON, C., RANDALL, D., ANDSINCL AIR, A Approximating the number of monomer-dimer coverings of a lattice J. Statist Phys 83, 3-4 (1996), 637–659

(59)

XVII Abdus Salam Int Cent Theoret Phys., Trieste, 2004, pp 267–304 (electronic)

[18] KENYON, R The dimer model In Exact methods in

low-dimensional statistical physics and quantum computing Oxford Univ Press, Oxford, 2010, pp 341–361

[19] KNUTH, D E A permanent inequality Amer Math Monthly

88, 10 (1981), 731–740, 798

[20] LOVÁSZ, L., AND PLUMMER, M D Matching theory North-Holland Publishing Co., Amsterdam, 1986 Annals of Dis-crete Mathematics, 29

[21] MCCUAIG, W Pólya’s permanent problem Electron J Com-bin 11, (2004), Research Paper 79, 83 pp (electronic) [22] MINC, H Upper bounds for permanents of (0, 1)-matrices

Bull Amer Math Soc 69 (1963), 789–791

[23] MINC, H Permanents, vol of Encyclopedia of Mathemat-ics and its Applications Addison-Wesley Publishing Co., Reading, Mass., 1978 With a foreword by Marvin Marcus [24] PÓLYA, G Aufgabe 424 Arch Math Phys 20 (1913), 271 [25] RADHAKRISHNAN, J Entropy and counting Computational

Mathematics, Modelling and Algorithms (2003)

[26] ROBER TSON, N., SEYMOUR, P D., AND THOMAS, R Per-manents, Pfaffian orientations, and even directed circuits

Ann of Math (2) 150, (1999), 929–975

[27] SAMUEL SON, P A Foundations of Economic Analysis, first edition ed Harvard University Press Cambridge, Mass ; London, England, 1947

[28] SCHRIJVER, A A short proof of Minc’s conjecture J Com-binatorial Theory Ser A 25, (1978), 80–83

[29] SEYMOUR, P., ANDTHOMASSEN, C Characterization of even directed graphs J Combin Theory Ser B 42, (1987), 36–45

(60)

[31] SZEGO, G Lăosung zu 424 Arch Math Phys 21 (1913), 291–292

[32] TEMPERLEY, H N V., AND FISHER, M Dimer problem in statistical mechanics-an exact result Philosophical Maga-zine 6 (Aug 1961), 1061–1063

[33] VALIANT, L G The complexity of computing the permanent

Theoret Comput Sci 8, (1979), 189–201

[34] VALIANT, L G Holographic algorithms SIAM J Comput. 37, (2008), 1565–1594

[35] VAN DER WAERDEN, B B Aufgabe 45 Jahresber Dtsch.

Math.-Ver 35(1926), 117

[36] VAN LINT, J H Notes on Egoritsjev’s proof of the van der Waerden conjecture Linear Algebra Appl 39 (1981), 1–8 [37] VANLINT, J H The van der Waerden conjecture: two proofs

in one year Math Intelligencer 4, (1982), 72–77

[38] VAN LINT, J H The van der Waerden conjecture In

Com-binatorics ’81 (Rome, 1981), vol 18 of Ann Discrete Math.

North-Holland, Amsterdam-New York, 1983, pp 575–580 [39] VAN LINT, J H., AND WIL SON, R M A course in

combina-torics, second ed Cambridge University Press, Cambridge, 2001

[40] VAZIRANI, V V., AND YANNAKAKIS, M Pfaffian orientations,

(61)

TỐT HƠN

NGUYỄN TIẾN DŨNG

(Đại học Toulouse, Pháp)

Giới thiệu

Nguyễn Tiến Dũng giáo sư Đại học Toulouse, Pháp Thời học sinh, Nguyễn Tiến Dũng tham gia kỳ thi Olymic Toán học quốc tế dành cho học sinh phổ thông (IMO) năm 1985 đạt huy chương vàng 15 tuổi GS Nguyễn Tiến Dũng có nhiều viết tốn học giảng dạy toán học đăng trang web cá nhân zung.zetamu.net Vừa rồi, thông qua công ty Sputnik, Nguyễn Tiến Dũng phát hành miễn phí điện tử sách “Học toán dạy toán nào?” để chia sẻ quan điểm anh vấn đề thực tế

Được đồng ý GS Nguyễn Tiến Dũng, chúng tơi xin trích đăng mục chương sách Toàn văn sách với nhiều quan điểm thẳng thắn thú vị bạn đọc tải địa sau:

• Trang bán hàng mạng Sputnik Education

• Tạihttp://sputnikedu.com/

(62)

Một ví dụ phép tính tích phân (có chương trình tốn PTTH) Trong thăm dị ý kiến trang facebook Sput-nik Education vào đầu năm 2014, hầu hết người trả lời nói họ chẳng cần dùng đến tích phân Câu hỏi đặt là: dạy tích phân, số phức, v.v chương trình phổ thơng làm gì, chẳng sau dùng đến chúng? Trong thảo luận cải cách giáo dục, có nhiều người nêu ý kiến nên bỏ thứ

Khơng Việt Nam, mà giới có nhiều người, kể trưởng giáo dục, cho chương trình tốn phổ thơng nước họ nặng, thừa Họ muốn cắt giảm bớt chương trình số học tốn phổ thơng đi, chí đến nửa, thay vào mơn học khác, ví dụ mơn chăn ngựa Trong số lý họ đưa ra, chuyện nhiều thứ tốn dạy phổ thơng khơng cần thiết, cịn có thêm lý thời đại máy tính, tính tốn có máy tính cho rồi, cần học tốn nhiều làm

Cả hai lý (đã có máy tính làm tốn thay, chương trình tốn chứa nhiều thứ “vơ dụng”), nhìn có lý, thực khơng hợp lý

Học tốn khơng đơn học phép tính, mà cịn học nhiều kiến thức kỹ quan trọng khác, khả suy luận lơ-gích, chiến lược, phân biệt sai, mơ hình hóa vấn đề, v.v bàn phía Máy tính giúp tính tốn, tra cứu, v.v., khơng thể hiểu thay Chúng ta cần phải hiểu toán, để giao đầu cho máy tính thực hiện, hiểu ý nghĩa kết mà máy tính đưa Và lúc phải ỷ lại vào máy tính người ngày ngu đần đi, trở thành thứ nô lệ

(63)

người học toán đến bậc tiến sĩ chưa hiểu chất khái niệm tích phân

Việc dạy học tốn theo lối “tốn vơ nghĩa” (khơng thấy cơng dụng đâu) có tác hại làm cho nhiều người trở nên chán ghét mơn tốn, cịn người mà “thích nghi” với lối học lại dễ bị tự kỷ hình thức chủ nghĩa

Bởi vậy, cần tăng cường tìm hiểu chất ý nghĩa khái niệm học toán, hình thành chúng ứng dụng chúng, lý chúng tồn Câu hỏi “nó dùng để làm gì” quan trọng câu hỏi “nó định nghĩa nào” Có kiến thức tốn học trở nên có nghĩa hữu dụng

Quay lại ví dụ khái niệm tích phân Einstein có nói: “Chúa khơng quan tâm đến khó khăn tốn học người, Chúa tính tích phân cách thực nghiệm” Trong sống hàng ngày, nhiều “tính tích phân theo cách Chúa”, khơng phải dùng cơng thức tốn học viết cách chi li hình thức, mà quan sát, ước lượng trực giác, v.v Ví dụ như, ước lượng diện tích nhà, thể tích thùng rượu, thời gian để làm việc đó, v.v., “tính tích phân” Tích phân chẳng qua tổng nhiều thành phần lại với nhau, với số thành phần vơ hạn (chia nhỏ thành tổng thành phần “nhỏ li ti”), cơng cụ để tính tốn hay ước lượng độ lớn vạn vật: thể tích, diện tích, độ dài, vận tốc, trọng lượng, thời gian, tiền bạc, tăng trưởng dân số, bệnh dịch, v.v Bản thân ký hiệu phép lấy tích phân chữ S kéo dài ra, mà S có nghĩa summa (tổng)

(64)

cứu, phải học chúng dễ có cảm giác học phải vơ dụng

Khi mà khơng nắm ý nghĩa việc lấy tích phân, việc tính tính phân phân thức máy, nhớ đống cơng thức tính tích phân hồn tồn phí thời gian vơ ích Đấy điều không may mà nhiều người gặp phải: học phép tính tính phân thứ “thánh bảo phải vậy”, giáo điều mà không dùng vào đâu Trong đó, ngành tài giới dùng tích phân “như cơm bữa”

Các mơ hình tài đại dùng tốn đại, khơng tính tích phân theo nghĩa thơng thường nhiều người biết, mà cịn tính tích phân ngẫu nhiên, thứ toán học phát triển từ kỷ 20 Chính mà nhiều người gốc tốn trở thành “át chủ bài” thị trường tài chính, chương trình cao học tài nơi có tốn nặng Có sinh viên Việt Nam sau tốt nghiệp xuất sắc trường kinh tế hay tài chính, học bổng sang Pháp học cao học, bị “gẫy cầu” không theo được, phần khơng thể nhai phép tính tích phân ngẫu nhiên này, không chuẩn bị tốt kiến thức toán

2 Tự học tốt có thầy tốt hơn

Ở Việt Nam có tình trạng học sinh phải học nhiều, từ sáng đến đêm, hết học thức lớp lại học thêm Việc đến lớp nhiều phản tác dụng: có nguy làm cho trẻ trở nên mụ mẫm, thụ động, khơng có thời gian để tự suy nghĩ tiêu hóa kiến thức, khơng có thời gian cho hoạt động khác ngủ, vui chơi, thể thao, âm nhạc, học làm việc nhà, v.v quan trọng cho phát triển Một phần để tránh vấn nạn học thêm mà nhiều người muốn cho vào trường quốc tế hay “tị nạn giáo dục”

(65)

các kiến thức” Xu hướng tương đối Việt Nam xâm nhập vào chương trình cải cách giáo dục nước tiên tiến giới Anh, Pháp từ nửa kỷ nay, dựa chủ thuyết “constructivisme” (“tự xây dựng kiến thức”) Jean Piaget, người “làm mưa làm gió” giáo dục

Theo chủ thuyết “constructivisme” Piaget, học trị “tự xây dựng” kiến thức mình, thầy nói chung khơng giảng kiến thức mà gợi ý cách tìm Chủ thuyết này, với câu nghe bùi tai “lấy học trò làm trung tâm”, khoác áo “khoa học, đổi mới”, nhanh chóng làm “mủi lịng” quan chức giáo dục chuyên gia giáo dục nhiều nước Tuy nhiên, kết đem lại hồn tồn trái ngược với mong đợi: đầu tư cho giáo dục nhiều lên trình độ học sinh giảm

Ví dụ, Pháp, theo báo cáo Viện Hàn lâm Khoa học năm 2004 nhà bác học lớn ký tên (xem đây), 30 năm kể từ thời điểm bắt đầu cải cách giáo dục Pháp theo hướng “constructivisme” vào năm 1970, chương trình mơn tốn bị thụt 1,5 năm, tức tính trung bình học sinh học đến lớp 12 ngày cịn trình độ mơn tốn học sinh học lớp 10 thời năm 1970! Trong phát triển khoa học cơng nghệ ngày địi hỏi nhiều hiểu biết toán

Nhà toán học tiếng Laurent Lafforgue tác giả khác có viết sách bi kịch giáo dục Pháp vào năm 2007: Laurent Lafforgue, Liliane Lurcc¸at et Collectif, La débâcle de l’école: une tragédie incomprise, 09/2007 (Sự “đổ vỡ” trường học: bi kịch không thấu hiểu)

(66)

Khơng mơn tốn, mà mơn học khác Pháp hứng chịu hậu nghiêm chủ thuyết Piaget, người mà thời nhiều nơi tung hô nhà cải cách giáo dục lớn giới Ví dụ, mơn tiếng Pháp, thay dạy chia động từ ngày ưa, với chủ thuyết “constructivisme” người ta bắt học sinh “quan sát thay đổi dạng động từ” Hệ quả: tỷ lệ lớn học sinh Pháp đến vào đại học chia động từ cho Trong môn lịch sử, kiến thức lịch sử trang bị cho học sinh hạn chế, lại địi hỏi học sinh bình luận tài liệu y học sinh nhà sử học Kết “bình luận tư do” thực câu giáo điều viết trước (bởi học sinh có biết đâu bình luận) Mơn lịch sử dạy hời hợt đến mức học sinh lẫn lộn thứ tự thời gian (chronology) kiện, kể học sinh “khá” PTTH khơng biết hồng đế Napoleon Louis XIV sinh trước sinh sau Ở nước khác chịu ảnh hưởng Piaget, tình hình tồi tương tự Một nghiên cứu thống kê Canada (xem đây) cho thấy phương pháp giảng dạy khác phương pháp theo “constructivisme” phương pháp cho kết tồi tệ Ở Thuỵ Sĩ, quê hương Piaget, người ta phải kêu trời rằng, giáo dục phổ thông Thuỵ Sĩ trước năm 1970 coi mẫu mực thế, mà từ bị nhiễm “construc-tivisme” trở nên suy sút nặng

Ở Việt Nam, có số người muốn cải cách giáo dục theo hướng “constructivisme” Piaget, coi “kinh thánh”, tơ điểm thêm cho thành lý thuyết với tên kêu “công nghệ giáo dục”, tương tự mà nước khác người ta trải qua và phải hứng chịu hậu Các bậc phụ huynh người làm ngành giáo dục nên đề phịng chuyện này, khơng phải khốc áo “khoa học, cơng nghệ” khoa học, công nghệ, kể giáo dục

Vì học theo kiểu “tự xây dựng kiến thức” lại chậm nhiều so với có nghe thầy giảng? Thực điều mà từ xưa người ta biết, có điều số học thuyết “lang băm” làm nhiều người quên điều

(67)

Trung Quốc có câu “Nghe thầy tự mị tháng” Theo ước tính, trung bình học có thầy giảng giải nhanh gấp lần tự học Đặc biệt học sinh có học lực trung bình, việc nghe giải thích kiến thức cách rõ ràng cần thiết, có tỷ lệ nhỏ học sinh thơng minh đặc biệt dễ dàng tự tìm qui luật Có người lầm tưởng rằng, kiến thức có sẵn sách hết rồi, cần thầy giảng cho nữa, đọc xong Kiến thức sách khơng có nghĩa nhảy vào đầu cách dễ dàng Nếu khơng có người hướng dẫn, hồn tồn đọc mà khơng hiểu, tưởng hiểu thực chưa hiểu gì, dẫn đến tẩu hoả nhập ma Tệ nữa, đọc phải sách nhảm nhí, sách viết sai, v.v

Quá trình tự tìm kiến thức, tìm chân lý khó khăn lâu dài trình tiếp thu lại từ người nắm Những kiến thức mà học sinh học thứ mà nhà bác học kỷ trước phải đời người để tìm Khơng thể bắt học sinh lại đường khám phá đó, nhiều thời gian Và tất nhiên lãng phí nhiều thời gian tâm trí vào việc “phát minh lại bánh xe”, học sinh không đủ thời gian để tiếp cận kiến thức cần thiết khác

Điều khơng có nghĩa học sinh không nên nghiên cứu sáng tạo Nghiên cứu sáng tạo cần thiết, phải dựa tảng sở có rỗng ruột mà nghiên cứu sáng tạo hay ho Để học nghiên cứu sáng tạo, người thầy tốt cho việc người nghiên cứu sáng tạo: nhà khoa học, nhà sáng chế, nghệ sĩ, v.v (chứ người dạy nghiên cứu mà thân chưa có cơng trình nghiên cứu quan trọng nào) Cũng mà để học văn học tiếng, học sinh nên đọc thơ, mẩu truyện hay nhà văn để nhờ mà thích học đọc, thấy hay đẹp sáng tạo ngôn ngữ từ lớp học khái niệm ngôn ngữ phức tạp ví dụ nhạt nhẽo vơ nghĩa

(68)

bên cạnh” Còn kiểu tự học “ốch hơn”, tự học khơng cần thầy Ở tơi khơng nói đến việc ơn hay làm tập nhà, lúc tự học theo chương trình có thầy hướng dẫn Tơi muốn nói đến việc tự học mà khơng có thầy hướng dẫn, kết sao?

Có vị giáo sư Việt Nam tự hào việc “tự học thành tài”, viết sách dày ngàn trang việc tự học, với ví dụ thân vị Tơi khơng nghi ngờ thơng minh vị giáo sư này, ngồi vị giáo sư cịn nhà quản lý thành cơng Chỉ có điều đáng tiếc rằng, vị hoàn toàn ảo tưởng khoa học: vị tự coi thiên tài khoa học, trăm óc vĩ đại giới, sánh ngang tầm với nhân vật lừng danh tồn cầu, cơng trình vị chẳng giới quan tâm trích dẫn Đấy ví dụ rủi ro việc tự học mà khơng có người hướng dẫn: dễ bị “đâm vào ngõ cụt”, dễ bị hoang tưởng

Bản thân tơi tự học nhiều thứ, với mức độ thành công hay thất bại khác nhau: từ học bơi, học thiên văn, tiếng Tàu, tiếng Anh, v.v chủ yếu tự học, khơng có thầy Khi sang Pháp làm việc, tơi chẳng nói câu tiếng Pháp nào, từ đến chẳng học tiếng Pháp có người dạy, toàn tự học Cả luận án tiến sĩ tơi tự làm, khơng có người hướng dẫn Nói khơng phải để khoe, mà để làm ví dụ cho thấy khả tự học người không nhỏ

Càng nhiều tuổi hay học lên cao cần đến khả tự học, điều kiện để học có thầy có lớp đi, nhu cầu niềm vui học tập tuổi có Nói khơng có nghĩa tự học tốt học có thầy hướng dẫn Tơi đến lúc dạy học cho sinh viên nói sai tiếng Pháp Nếu có điều kiện thời gian tiền bạc để học tử tế tiếng Pháp tốt nhiều không “ngọng”

Vậy, khơng có thầy, ta làm để tự học cho hiệu quả? Sau số biên pháp tốt mà biết:

(69)

Ngày nay, thời đại internet, ngày có nhiều giảng hay đủ mơn, người thầy giỏi nhất, tìm đến mà xem Đừng ngại tiếp cận trao đổi qua thư từ với thầy xa bảo cho Các thầy giỏi thường bận, tỏ thành tâm có lúc dành chút thời gian cho Đối với số mơn học, tìm thầy dạy trực tiếp qua mạng với giá phải Ví dụ, học tiếng Tây Ban Nha thầy (có tiếng mẹ đẻ Tây Ban Nha) trị qua skype với giá 10$/tiếng

Tìm bạn thay thầy.

Tiếng Việt có câu “học thầy không tầy học bạn” Bạn bè, đồng nghiệp tương trợ cho nhiều chuyện học Có bạn giải thích lại dễ hiểu thầy giải thích Có thể kết bạn internet cho việc học Việc ngày trở nên dễ dàng Trong việc học tiếng, kết bạn, nói chuyện với người xứ phương pháp hiệu

Nhúng mơi trường thuận lợi.

Học gì, mà xung quanh có nhiều thứ liên quan đó, vào hơn: sách vở, phim ảnh, đồ thí nghiệm, v.v tốt

Khơng sợ sai.

“Ai khơng làm khơng sai” Cứ làm đi, đừng sợ sai, trừ sai sai gây tai hoạ cho người khác Tất nhiên, cần phải quan sát kiểm tra để biết sai đâu mà cịn sửa

Làm từ từ, vừa làm vừa quan sát cảm nhận suy nghĩ.

(70)(71)

ĐÀM THANH SƠN

(Đại học Chicago, Hoa Kỳ)

Lời giới thiệu

GS Đàm Thanh Sơn GS Vật lý lý thuyết trường Đại học Chicago, Hoa Kỳ Ông đoạt huy chương vàng toán quốc tế với số điểm tuyệt đối 42/42 15 tuổi Đàm Thanh Sơn quan tâm đến việc truyền bá niềm đam mê khoa học cho hệ trẻ Blog ông https://damtson.wordpress.com cập nhật viết hay, toán (vật lý) thú vị dạng thường thức phổ thông hay gợi mở cho nhà khoa học tương lai Được cho phép GS Đàm Thanh Sơn, chọn đăng loạt thư Pyotr Kapitsa khoa học ông chọn dịch giới thiệu blog

Nguồn thư:

Капица П.Л., Письма о науке, 1930-1980 – М.: Моск рабочий, 1989

(72)

Tóm tắt nội dung

Pyotr Kapitsa (trái) Nikolay Semyonov, hai nhà vật lý đạt giải thưởng Nobel vật lý (tranh chân dung Boris Kustodiev, 1921)

(73)

Ông người đưa ý tưởng thành lập tạp chí chuyên khoa học cho hệ trẻ, sau tạp chí Kvant

Rất nhiều thư ông công bố Nhiều thư liên quan đến khoa học, giáo dục đáng đọc cịn giữ tính thời Tơi dịch số thư ông đăng blog

1 Trích thư Kapitsa gửi vợ, A.A.Kapitsa

13 tháng 12 năm 1935, Moskva

Hôm qua anh đánh cờ với Aleksei Nikolaevich Bakh 1 Cụ rất dễ mến, anh không đồng ý với cụ điểm Anh nói với cụ tình trạng khoa học nước ta xấu, cụ bảo: "Đúng thế, làm được, có nhiều thứ quan trọng khoa học " Đây ví dụ điển hình nhà khoa học tự nguyện đẩy xuống hạng ưu tiên thứ nhì, chí thứ ba Anh cho phải coi khoa học việc quan trọng lớn lao, inferiority complex (tiếng Anh nguyên – mặc cảm tự ti, tự cho khơng quan trọng) giết chết khoa học nước ta Các nhà khoa học phải cố gắng chiếm vị trí hàng đầu việc phát triển văn hóa nước nhà khơng lẩm bẩm "ở nước ta có thứ quan trọng hơn" Đánh giá quan trọng nhất, cần phải ý đến khoa học kỹ thuật đến mức cơng việc nhà lãnh đạo Cịn cơng việc nhà khoa học tự tìm chỗ đứng đất nước chế độ mới, khơng đợi người khác cho phải làm Cái thái độ khó hiểu xa lạ anh

Khi anh nói chuyện với nhiều nhà khoa học, anh ngạc nhiên họ tuyên bố "Cậu nhiều cậu làm chả dễ dàng " Và Họ làm bắt đầu nghiệp, hội ban đầu anh họ không giống Họ làm anh có rơi từ trời xuống, anh bỏ công sức, nơron thần kinh đạt Về khía cạnh

1A.N Bakh (1857-1946) nhà hóa sinh lớn Nga, bạn thân của

(74)

người thật hèn hạ, họ cho đời không công bằng, xung quanh có lỗi trừ họ Nhưng đấu tranh làm gì, khơng phải để ta lợi dụng hồn cảnh sẵn có quanh ta cho việc phát triển tài tạo điều kiện làm việc cho mình? Nếu chấp nhận quan điểm Bakh Co khơng xa

2 Bức thư cứu Landau

6 tháng năm 19392 Đồng chí Molotov,

Trong thời gian gần đây, nghiên cứu hêli lỏng gần độ khơng tuyệt đối, tơi tìm loạt tượng làm sáng tỏ lĩnh vực bí ẩn vật lý đại Trong tháng tới định cơng bố phần cơng trình Nhưng để làm việc này, cần nhà lý thuyết giúp đỡ Ở Liên Xơ, người hồn tồn làm chủ lĩnh vực lý thuyết mà cần Landau, hiềm nỗi bị bắt từ năm

Tôi hy vọng thả, tơi phải nói thẳng tin Landau lại tên tội phạm quốc gia Tôi tin điều nhà khoa học trẻ, tài chói lọi Landau, 30 tuổi tiếng châu Âu, lại háo danh, có đầy chiến tích khoa học đến mức khơng thể có sức lực, cảm hứng thời gian cho cơng việc khác Đúng Landau có miệng độc, lạm dụng anh tạo nhiều kẻ thù sẵn sàng gây khó dễ cho Nhưng dù có tính cách xấu mà buộc phải lưu ý đến, chưa thấy làm điều khuất tất

Tất nhiên, nói tất điều đó, tơi can thiệp vào việc khơng phải mình, lĩnh vực thẩm quyền

2Landau bị bắt ngày 28/4/1938 Ngay hơm Kapitsa viết thư

(75)

của NKVD (Bộ Nội vụ) Nhưng dù nghĩ phải nêu lên điểm bất thường sau:

1 Landau ngồi tù năm, mà việc điều tra chưa kết thúc, thời gian điều tra lâu cách khơng bình thường Tơi, với tư cách giám đốc quan anh ta, hồn tồn

khơng biết bị cáo buộc tội

3 Quan trọng năm nay, lý mà khoa học, Xơ-viết lẫn giới, khơng có đầu Landau

4 Landau ốm yếu, bị hại cách khơng cần thiết đáng xấu hổ cho người Xơ-viết

Vì tơi muốn gửi tới đồng chí u cầu sau đây:

1 Có thể đề nghị NKVD đặc biệt ý xúc tiến vụ Landau không;

2 Nếu khơng được, liệu sử dụng đầu Lan-dau cho công việc khoa học lúc bị giam Bu-tyrki [trại giam cách ly để điều tra] khơng Tơi nghe nói kỹ sư đối xử

P Kapitsa

3 Về việc trả lương cho người làm

khoa học

Sau trở thành giám đốc Viện vấn đề Vật lý, Kapitsa viết nhiều thư cho lãnh đạo Xơ-viết than phiền chế tài q cồng kềnh cứng nhắc Nga Trong thư Kapitsa có viết cơng việc kế tốn mà thư ký ông Anh làm chưa đến tiếng ngày Nga phải người làm hết Trong đoạn sau ông đề nghị áp dụng chế trả lương Viện ông giống Anh

(76)

.Nguyên tắc để đánh giá lao động Liên Xô quy định rõ ràng xác Hiến pháp Stalin hưởng theo lao động lực Nguyên tắc áp dụng quán phong trào Stakhanov đưa đến thành chói lọi Nó cho cá nhân khoảng không rộng lớn để phát triển; người công nhân xuất sắc, biết tổ chức lao động vượt mức khốn nhiều lần, trả cơng theo lao động lực người thể cơng việc Cách trả lương theo thang lương nhà nước hành mâu thuẫn với nguyên tắc Các quan hành quan liêu thường hay tuyển người thích làm việc n ổn, khơng thích có sáng kiến, chí trái lại cố gắng làm xác thị từ tốt Với người này, hệ thống trả lương theo thang nhà nước có lẽ hồn tồn bình thường Hệ thống áp dụng ta trước nước Tây Âu dẫn đến tệ quan liêu [Guy de] Maupassant mô tả sống động mà chưa nước tránh

Đáng tiếc chưa tìm cách để áp dụng phương pháp Stakhanov cho nhân viên hành văn phịng, tức vào suất lao động để tăng lương

Áp dụng thang bậc lương nhà nước cứng nhắc viện khoa học hoàn toàn sai Chúng không chấp nhận kiểu làm việc công chức: công việc mở không gian rộng lớn cho việc hoàn thiện lao động, phát triển cá nhân phát huy lực riêng người Hệ thống trả lương cứng nhắc theo thang lương nhà nước hồn tồn khơng phù hợp Nhưng áp dụng hệ thống trả lương kiểu Stakhanov quan nghiên cứu khoa học Trở ngại chỗ cơng việc sản xuất, nên không đưa chuẩn mực cứng để đánh giá cơng việc theo sản phẩm Vì cần tìm phương pháp khác Đó điều

Phương pháp đáng tin cậy để đánh giá công việc viện khoa học dựa vào ý kiến ban giám đốc viện, ý kiến mà theo tơi có thẩm quyền để định mức lương,

3Hiến pháp Liên Xô năm 1936.

(77)

bởi ban giám đốc người giao nhiệm vụ, biết mức độ khó khăn nhiệm vụ, đạo cơng việc đánh giá cơng việc tốt Khơng có lối thoát khác Hệ thống thang lương nhà nước khơng thích hợp, chứng trước mắt tất quan khoa học tìm cách để lách khỏi hệ thống Các mức lương xào xáo, thổi phồng kiểu toán tổ hợp phức tạp để làm rối trốn khỏi mắt tra quan tài Phương pháp chào đón công cụ tự vệ tốt để khỏi phải tuân theo kỷ luật ngân sách nghiêm ngặt, kỷ luật ngược lại nhu cầu khoa học Kiểu tự lừa dối làm vô ghê tởm

4 Về việc đối xử với nhà khoa học

bất đồng quan điểm

Trích thư gửi Yuri V Andropov, chủ tịch Uỷ ban An ninh Quốc gia (KGB)

Ngày 11 tháng 11 năm 1980, Moskva

Yuri Vladimirovich kính mến,

Cũng nhiều nhà khoa học khác, vơ lo lắng tình cảnh số phận hai nhà vật lý lớn nước ta — A D Sakharov Yu F Orlov Tình hình mơ tả đơn giản sau: Sakharov Orlov đem lại cống hiến to lớn hoạt động khoa học, hoạt động họ người bất đồng quan điểm bị coi có hại Hiện họ bị đặt vào hồn cảnh khơng thể có hoạt động Tóm lại, họ khơng thể mang lại lợi ích tác hại Thử hỏi làm có lợi cho đất nước hay khơng? Trong thư, tơi thử phân tích thật khách quan câu hỏi nêu

(78)

người thường biết đến hoạt động khoa học nhà bác học này, họ đưa nhận định ngược lại tình trạng

Trong lịch sử văn hố lồi người, từ thời Socrat đến nay, có khơng trường hợp người ta kịch liệt chống người bất đồng quan điểm Để giải khách quan vấn đề đặt tất nhiên cần cân nhắc bối cảnh xã hội cụ thể đất nước Trong hoàn cảnh xây dựng chế độ xã hội mới, nghĩ đắn vào quan điểm Lênin, quan điểm tồn diện người khơng nhà tư tưởng tiếng, nhà khoa học, mà nhà hoạt động xã hội lớn Cách đối xử Lênin với nhà khoa học trường hợp tương tự nhiều người biết đến Điều thể rõ ràng đầy đủ qua cách Lênin đối xử với I P Pavlov

Sau cách mạng, biết bất đồng quan điểm Pavlov, không nước ta mà nước ngồi Ơng cố tình phơi bày công khai thái độ không tán thành chủ nghĩa xã hội Ơng phê phán, chí chửi lãnh đạo khơng e dè lời phát biểu gay gắt; ông làm dấu thánh qua nhà thờ, đeo huy chương Nga Hoàng trao tặng, huy chương mà trước cách mạng ông không thèm để ý đến, v.v Nhưng Lênin không mảy may để ý đến biểu bất đồng quan điểm Pavlov Đối với Lênin, Pavlov nhà khoa học lớn, Lênin làm tất để đảm bảo điều kiện làm việc tốt cho cơng việc khoa học Pavlov Ví dụ, người biết, thí nghiệm quan trọng phản xạ có điều kiện Pavlov tiến hành chó Vào năm 1920, thực phẩm Petrograd thiếu trầm trọng, theo thị Lênin, thức ăn để ni chó thí nghiệm Pavlov cung cấp bình thường

Tơi cịn biết hàng loạt trường hợp khác mà Lênin quan tâm đặc biệt tới nhà khoa học Điều biết qua thư Lênin gửi K A Timiryazev, A A Bogdanov, Carl Steinmetz v.v

Cần phải đối xử với người bất đồng kiến

(79)

người, mà hoạt động sáng tạo lĩnh vực văn hoá lại đảm bảo tiến nhân loại

Có thể nhận dễ dàng nguồn gốc tất lĩnh vực hoạt động sáng tạo người bất mãn với trạng Ví dụ, nhà khoa học khơng thoả mãn với trình độ nhận thức lĩnh vực khoa học mà quan tâm, tìm phương pháp nghiên cứu Nhà văn không hài lòng với mối quan hệ người với người xã hội tại, cố gắng dùng nghệ thuật để tác động lên cấu trúc xã hội đến hành vi người Người kỹ sư khơng thoả mãn với giải pháp kỹ thuật có tìm dạng kết cấu để giải vấn đề Nhà hoạt động xã hội không lòng với văn luật ước lệ sử dụng để xây dựng nhà nước, tìm hình thức để vận hành xã hội, v.v

Tóm lại, để xuất ước muốn sáng tạo, phải có bất mãn với trạng, nghĩa cần trở thành người bất đồng ý kiến Điều lĩnh vực hoạt động người Tất nhiên, người bất mãn nhiều, để thể cách có hiệu hoạt động sáng tạo cịn cần có tài Cuộc sống cho thấy có tài lớn, cần tôn trọng nâng niu bảo vệ họ Kể có lãnh đạo tốt điều khó thực Khả sáng tạo lớn đòi hỏi tính cách mạnh, điều dẫn đến cách thể bất mãn gay gắt, người tài thường "ngang" Ví dụ, tượng hay thấy nhà văn lớn, họ thích tranh cãi thích phản kháng Trên thực tế, hoạt động sáng tạo thường khơng tiếp đón nhiệt tình lắm, đa số người bảo thủ ưa sống phẳng lặng Kết biện chứng phát triển văn hố lồi người bị kẹt mâu thuẫn bảo thủ bất đồng quan điểm, điều xảy thời đại lĩnh vực hoạt động văn hoá người

Cơng việc sáng tạo lớn thường mang tính chất tư tưởng

(80)

quan điểm gay gắt Pavlov vấn đề xã hội, trân trọng cá nhân hoạt động khoa học Pavlov Điều dẫn đến kết thời Xôviết, Pavlov với tư cách nhà sinh lý học không gián đoạn nghiên cứu xuất sắc phản xạ có điều kiện, nghiên cứu vai trò dẫn dắt khoa học giới Còn vấn đề liên quan đến xã hội tất Pavlov nói bị lãng qn từ lâu

(81)

E

INSTEIN

BAN BIÊN TẬP EPSILON

Lời giới thiệu

Albert Einstein không danh với thuyết tương đối mà ơng cịn để lại cho nhân loại triết lý sống vô thâm thúy Tất người học điều từ triết lý này, cho dù họ học sinh, sinh viên, phụ huynh, thầy cô giáo, nhà khoa học hay doanh nhân Chúng trích đăng số câu nói Einstein bình luận báo Tuổi trẻ Doanh nhân Sài Gòn giới thiệu

(82)

1 Theo đuổi trí tị mị

"Tơi chẳng có tài đặc biệt Tơi ham thích tị mị" Tị mị giúp cung cấp trí tưởng tượng cho Khi đặt câu hỏi thứ khác, tìm thơng tin quan trọng giúp giải vấn đề, mở cánh cửa hình thành mối liên kết Khi đặt câu hỏi thân, đánh thức niềm tin, tiết lộ ham muốn sâu thẳm tạo nên thay đổi tích cực Những câu hỏi chưa có lời giải xuất đầu bạn ?

2 Sự kiên trì vơ giá

"Khơng phải thông minh, dành thời gian lâu hơn để nghiền ngẫm vấn đề."

Nếu bạn có giấc mơ, bạn phải đối mặt với trở ngại; việc kiên trì với nó, Einstein nói, mang ý nghĩa khác biệt thất bại thành công Một số cách để bắt đầu luyện tập kiên trì cam kết giấc mơ bạn, giữ thái độ tích cực, ln tập trung vào bạn muốn ngày trở lại từ nghịch cảnh

3 Tập trung vào tại

"Người đàn ông lái xe an tồn gái xinh đẹp đơn giản người khơng trao cho nụ hôn tập trung xứng đáng"

(83)

4 Trí tưởng tượng quyền năng

"Trí tưởng tượng tất Trí tưởng tượng xem thử của những địa điểm tham quan đến sống Trí tưởng tượng cịn quan trọng kiến thức"

Với ý tưởng, đế chế hình thành Lấy ví dụ, Walt Disney, chuyên gia đích thực việc tưởng tượng Ông lấy cảm hứng cho nhân vật chuột Mickey từ chuột thú cưng già nơng trại ơng Chú chuột trắng đen trở thành huyền thoại hoạt hình Trí tưởng tượng mở cánh cửa đến với vương quốc khả

5 Hãy mắc sai lầm

"Một người chưa mắc sai lầm không làm những điều lạ"

Sai lầm điều chắn xảy đặc biệt bạn theo đuổi điều đáng giá Chúng gây thất vọng khó khăn cho tự tin, thường cần thiết để kiểm tra cam kết thực với mục tiêu cuối Làm có điều tuyệt vời hồn thành mà khơng gặp thất bại lúc đầu theo cách Thất bại thật khơng bắt đầu khơng hồn thành

6 Sống cho khoảnh khắc

"Tôi chẳng nghĩ tương lai, đến sớm thơi"

Như họ nói, khoảnh khắc tất thực nắm giữ, khái niệm khó nắm bắt Eckhart Tolle nói sách ông,Power of Now (Quyền của Hiện tại), thành công người khoảnh khắc thực đo bình an mà anh/ cảm nhận Bằng việc trở nên ý thức thời điểm, tập trung vào vấn đề quan trọng

(84)

hoạch, thứ khơng diễn bạn dự tính Vấn đề quan trọng sống ngày hơm Làm thứ tốt khả khơng phải lo lắng ngày mai

7 Tạo giá trị

"Nỗ lực không để thành cơng, để có giá trị"

Bạn định nghĩa thành công nào? Làm để có sống thành cơng? Những câu hỏi câu hay để bạn tự chất vấn thân Đó nuôi dạy đứa khỏe mạnh hạnh phúc, có mối quan hệ ý nghĩa trọn vẹn, xác hội thoại , viết sách, yêu công việc, cảm thấy khỏe ngày – điều giành cho bạn, nơi để đặt tiếp tục trọng tâm Những tập trung vào phát triển thực

8 Đừng lặp lặp lại

"Điên rồ : làm việc lặp lặp lại nhiều lần mong đợi kết khác nhau"

Nếu bạn không hạnh phúc lĩnh vực sống tài hay mối quan hệ, chọn làm việc khác vào ngày hôm sau ý tưởng nhằm đánh thức thói quen Nếu bạn có cơng việc khơng trọn vẹn làm nản lịng mức độ đó, suy nghĩ bạn làm để thay đổi tình hình Đơi nhìn vấn đề tất cần thiết để nhận xảy Bước nhận bất mãn sau việc hướng tới mục tiêu hài lòng

9 Kiến thức đến từ kinh nghiệm

(85)

Kinh nghiệm thực tế tạo kiến thức tôn trọng đánh giá cao người khác Chúng ta đọc sách, nghe băng, học kinh nghiệm có sống mang lại học tốt cho người khác Câu chuyện sống bạn chứa đựng nhiều kiến thức người sẵn sàng lắng nghe cách hấp dẫn đáng tin cậy để tạo khác biệt với

10 Biết nguyên tắc làm tốt hơn

"Bạn phải biết quy luật trò chơi Và sau bạn phải chơi tốt hơn ai"

Để trở thành chuyên gia lĩnh vực đó, cần tìm hiểu tất vấn đề đó, nghiên cứu thành công người khác đặt mục tiêu làm tốt họ Sự cam kết niềm đam mê với nỗ lực mạnh mẽ tâm đến với thành công lớn

11 Hãy đơn giản thứ

"Nếu bạn khơng thể giải thích cho đứa trẻ tuổi hiểu được, thì chính bạn khơng hiểu cả"

Cố làm cho thứ phức tạp đồng nghĩa với việc bạn không hiểu chất vấn đề Hãy nhớ đến thầy cô giáo dạy bạn Họ giải thích tất trang sách đầy chữ cho bạn lời lẽ giản dị dễ hiểu Bạn nên ghi nhớ điều giao tiếp huấn luyện nhân viên

12 Hãy sáng tạo

"Sáng tạo có tính lây lan, truyền đi!"

(86)

13 Dám mắc sai lầm

"Cách để tránh sai lầm đừng có ý tưởng mới"

Sai lầm thứ xảy ngày với Và thứ thực làm thay đổi giới, quy trình hồn hảo mà ý tưởng Do đó, học cách chấp nhận sai lầm mạo hiểm với sáng tạo điều cần thiết để tạo thay đổi tích cực tổ chức

14 Hãy chăm chỉ

"Bạn không thất bại bạn ngừng nỗ lực" Einstein dành đời để nghiên cứu lý thuyết vật lý nhiều nghiên cứu số khơng đến kết Chúng ta khơng thể chắn kết việc làm, kiên trì chìa khóa Tất làm chăm theo đuổi mục tiêu Thất bại tồn ta dừng lại mà khơng cố gắng bước bước để đến thành công

15 Nghĩ khác

"Tôi chưa khám phá điều cách tư hợp lý" Những thứ tuyệt vời đến đến từ việc nghĩ "ngoài chiếc hộp" làm việc khác thường Mọi người thường khó chịu với kẻ nghĩ khác, lý có người kiệt xuất Khi gặp vấn đề nan giải, cố gắng nghĩ theo hướng khác, bạn có câu trả lời

16 Phát huy trí tưởng tượng

(87)

Sau tưởng tượng điều khác, bạn bắt đầu chia sẻ với người Sau đó, người nhìn thấy giới mà bạn tưởng tượng họ liên kết hỗ trợ bạn Hãy dành thời gian để mơ mộng, tưởng tượng nhớ chia sẻ ý tưởng bạn với người xung quanh

17 Làm điều không thể

"Chỉ nỗ lực đạt điều tưởng chừng không thể"

Nếu bạn sẵn sàng đối mặt với khó khăn bạn tiến gần đến điều "Điều không thể" khái niệm tương đối Bạn ngạc nhiên dám vượt qua điều hợp lý mà người nghĩ

18 Tôn trọng người

"Từ sống thường ngày, biết sống - trước hết là cho người xung quanh, cho người nở nụ cười và khiến ta hạnh phúc"

"Cuộc đời chẳng đáng sống, trừ ta sống người khác"

Hãy quan tâm đến người trước tiên Dành vài phút ngày để tập trung kết nối, trò chuyện với người khác Hãy cho họ biết bạn trân trọng họ Cảm ơn gửi tặng họ lời khen mà họ xứng đáng nhận Điều không giúp tâm trạng họ tốt mà tâm trạng bạn

19 Luôn sẵn sàng học hỏi

(88)

20 Làm điều đúng

"Luôn làm điều Việc làm hài lòng số người và làm người lại ngạc nhiên"

Khi bạn đứng ngã ba đường, chọn việc đắn mà làm Các lựa chọn khác dễ dàng thực mang đến cho bạn nhiều tiền Nhưng bạn nghĩ lựa chọn lại khó khăn cho bạn nhiều hội tương lai, đơn giản điều đắn cần làm bạn chọn

(89)

4

BÀI TOÁN TRONG ĐỀ THI

CHỌN ĐỘI TUYỂN

V

IỆT

N

AM

2015

TRẦN NAM DŨNG

(Đại học KHTN, Tp Hồ Chí Minh)

Tóm tắt nội dung

Trong số trước, bạn Trần Quang Hùng đưa lời giải cho hai tốn hình học đề chọn đội tuyển Việt Nam năm 2015 với phân tích bình luận thú vị liên quan Ở đây, số này, tiếp tục giới thiệu bạn đọc lời giải cho tốn cịn lại đề thi

Bài tốn 1. Gọi α nghiệm dương phương trình x2 +x =

5 Giả sử n số nguyên dương số nguyên không âm

c0, c1, c2, , cn thỏa mãn đẳng thức

c0+c1α+c2α2+· · ·+cnαn = 2015 (∗)

a) Chứng minh rằngc0+c1+c2+· · ·+cn ≡2 (mod 3) b) Tìm giá trị nhỏ tổngc0+c1+c2+· · ·+cn

Lời giải. a) Cách 1. Ta chứng minh quy nạp theo n Với

n = 0, ta có c0 = 2015, mệnh đề Với n = 1, α

là số vô tỷ nên đẳng thức c0 +c1α = 2015 xảy

c1 = 0, c0 = 2015, mệnh đề

Giả sử mệnh đề đến n ≥ Xét số nguyên không

âm c0, c1, c2, , cn, cn+1 thỏa mãn đẳng thức

c0+c1α+c2α2+· · ·+cnαn+cn+1αn+1 = 2015

Sử dụng đẳng thức α2 = 5−α, ta có cn+1αn+1 = cn+1αn−1(5−α)

Suy

(90)

Áp dụng giả thiết quy nạp, ta có

c0+c1+c2+· · ·+ (cn−1+ 5cn+1) + (cn−cn+1)≡2 (mod 3)

Nhưng có nghĩa

c0 +c1+· · ·+cn−1+cn+cn+1 ≡2 (mod 3)

Vậy mệnh đế với n+ Theo nguyên lý quy nạp tốn học, ta có điều phải chứng minh

Cách 2. Xét đa thức P(x) = c0 +c1x+c2x2 +· · ·+cnxn−2015

P(α) = 0.Ta chứng minh rằngP(x)chia hết cho Q(x) = x2+x−5, tức P(x) = Q(x)·S(x) với S(x) đa thức với hệ số nguyên Thật vậy, giả sử P(x) = Q(x)·S(x) +Ax +B, với A, B nguyên Thay x = α vào ta Aα+B = Do α số vô tỷ nên điều xảy A=B = VậyP(x) =Q(x)·S(x).Thay x= vào ta c0+c1+c2+· · ·+cn−2015 =−3·S(1), suy

c0+c1+c2+· · ·+cn≡2 (mod 3)

b) Với số nguyên không âm (c0, c1, c2, , cn) thỏa mãn

(∗),ta gọic0+c1+c2+· · ·+cn giá số Do tính thứ

tự tốt tập số tự nhiên, tồn số (c0, c1, c2, , cn)

thỏa mãn (∗)với giá nhỏ

Ta chứng minh nhận xét quan trọng sau:

Nhận xét. Nếu (c0, c1, c2, , cn) là có giá nhỏ thì ci <

với mọi i= 0, 1, 2, , n

Chứng minh.Ta chứng minh phản chứng Giả sử tồn i

sao cho ci ≥5 Khi dựa vào đẳng thức =α2+α, ta có

ciαi = (ci−5)αi + (α2+α)αi = (ci −5)αi+αi+1+αi+2

Như vậy, số (c0, c1, c2, , ci−5, ci+1+ 1, ci+2+ 1, , cn)cũng

thỏa mãn (∗)và có giá

c0+c1+· · ·+(ci−5)+(ci+1+1)+(ci+2+1)+· · ·+cn=c0+c1+· · ·+cn−3

nhỏ giá bộ(c0, c1, , cn).Điều mâu thuẫn với cách

(91)

Bây giờ, giống phần a) ta đặt:

P(x) = c0+c1x+c2x2+· · ·+cnxn−2015

vàQ(x) =x2+x−5 theo a),P(x) =Q(x)·S(x) Đặt: S(x) = b0+b1x+b2x2+· · ·+bn−2xn−2

So sánh hệ số hai vế, ta

c0−2015 =−5b0,

c1 =b0−5b1,

c2 =b0+b1−5b2, c3 =b1+b2−5b3,

, cn =bn−2

Từ điều kiện ≤ c0 ≤ 4, ta suy c0 = b0 = 403

Tiếp tục sang dòng thứ hai, ta tìm c1 = b1 = 80 Nói

chung dãy (ci, bi) xác định cách theo công

thức ci =bi−2+bi−1mod5 vàbi = bi−2+bi5−1−ci

Sử dụng công thức này, ta tính

I

C 3 1 0 B 403 80 96 35 26 12 0

Từ tìm có giá nhỏ là(0, 3, 3, 1, 1, 1,3,4, 0, 0, 3, 1), giá nhỏ là20.Vậy giá trị nhỏ c0+c1+· · ·+cn

20, đạt (0, 3, 3, 1,1, 1,3,4, 0, 0, 3, 1)

Bình luận. Ý tưởng quy nạp cách giải phần a)

khá tự nhiên Và kiện P(x) chia hết cho Q(x) mà ta dùng cách giải điều đặc biệt Ta có hai tính chất đơn giản quan trọng sau:

Tính chất 1. Nếu P(x) Q(x)là đa thức với hệ số ngun,

ngồi raQ(x)đơn khởi, tức có hệ số cao bằng1thì tồn tại

duy đa thức với hệ số nguyên S(x)R(x)sao cho

(92)

ii) deg R(x)<deg Q(x)

Tính chất 2. Cho P(x), Q(x) là đa thức với hệ số nguyên,

trong đó Q(x)bất khả quy, nhận số thực α làm nghiệm Khi

đó,P(x)chia hết choQ(x)

Trong phần b), ta dùng phương pháp tìm tính chất số tối ưu, sau dùng tính chất để xây dựng số tối ưu Lời giải phần b) liên hệ chặt chẽ đến hai cách giải phần a) Đây toán thú vị liên hệ nhiều vấn đề toán: đa thức, số nguyên, hệ đếm số, chia hết, thuật toán

Một số toán liên quan

1. (Nga 2014) Kho bạc nhà nước nước Cộng hịa tốn học chọn số α > sản xuất đồng xu có mệnh giá

1 rúp αk rúp với mọi k nguyên dương Người ta nhận

thấy mệnh giá (trừ mệnh giá 1) vô tỷ Có thể xảy tình số nguyên dươngn, ta chọn số đồng xu có tổng n mệnh giá chọn không 6lần?

2. (IMO 1976) Tổng số số nguyên dương 1976 Hỏi tích chúng lớn bao nhiêu?

Bài tốn 3. Một số ngun dương k có tính chất T(m)

với số nguyên dươnga, tồn số nguyên dương n cho

1k+ 2k+ 3k+· · ·+nk≡a (mod m)

a) Tìm tất số ngun dương k có tính chất T(20)

b) Tìm số ngun dươngk nhỏ có tính chấtT(2015).

Lời giải. a) Đặt Sk(n) = 1k+ 2k+· · ·+nk theo cơng thức

quen thuộc, ta có S1(n) =

n(n+ 1)

2 , S2(n) =

n(n+ 1)(2n+ 1)

6 , S3(n) =

n2(n+ 1)2

S4(n) =

n(n+ 1)(2n+ 1)(3n2+ 3n−1)

30 =

6n5+ 15n4+ 10n3−n

(93)

Sử dụng tính chất nk+4−nk≡0 (mod 20) với k >1 ta suy

k > thỏa mãn tính chất T(20) k+ thỏa mãn

tính chấtT(20) (∗)

Để có tính chất T(20), trước hết ta phải có tính chất T(5) Lập

bảng mơ-đun 5như sau

N

S1(n) 0

S2(n) 0

S3(n) 0

Ta thấy k = 1, 2, khơng thỏa mãn Vì n5 ≡n (mod 5) nên k =

cũng không thỏa mãn Vậy tất số không bội 4đều không thỏa mãn tính chấtT(20)

Ta chứng minh k = thỏa mãn tính chất T(20), từ đó, sử dụng

(∗) suy bội số thỏa mãn tính chất T(20) Ta thấy bảng đồng dư mô-đun20của n4

N 10 11

n4 16 16 16 16 1

Suy bảng đồng dư mô-đun20 củaS4(n)là:

N 10 11

S4(n) 17 18 14 19 15 16 12 13 13 14

N 12 13 14 15 16 17 18 19 20 21 22

S4(n) 10 11 12 6

N 23 24 25 26 27

S4(n)

Như tất số dư xuất Vậy tất số thỏa mãn điều kiện T(20) bội số

b) Theo kết câu a) k = 1,2, không thỏa mãn điều kiện T(2015). Ta chứng minh rằng k = 4 thỏa mãn điều kiện T(20m)

với m Từ suy k = giá trị nhỏ cần tìm

(94)

Ta chứng minh với a nguyên:

1) Tồn n cho 6n5+ 15n4+ 10n3−n ≡30a (mod 3), giả sử làn1

2) Tồn n cho6n5+ 15n4+ 10n3−n ≡30a (mod 22m+1),giả

sử làn2

3) Tồn n cho 6n5+ 15n4+ 10n3−n ≡30a (mod 5m+1),giả

sử làn3

Khi đó, theo định lý Trung hoa số dư, tồn tạin cho n≡n1 (mod 3), n ≡n2 (mod 22n+1), n ≡n3 (mod 5n+1)

và với n 6n5+ 15n4 + 10n3 −n ≡ 30a (mod 30·20n)

và ta có điều phải chứng minh

Mệnh đề 1) hiển nhiên, ta chọn n=

Để chứng minh 2), ta chứng minh quy nạp theo n với a nguyên với mọim nguyên dương, tồn n cho

6n5 + 15n4+ 10n3−n ≡30a (mod 2m).

Với m = điều đúng, ta chọn n = Giả sử tồn n cho 6n5+ 15n4+ 10n3−n ≡30a (mod 2m).Ta chứng minh tồn

tạin cho 6n5+ 15n4+ 10n3−n≡30a (mod 2m+1). Đặt:

6n5+ 15n4+ 10n3−n= 30a+u·2m

và chọn N =n+v·2m. Khi đó, ta có

6N5+ 15N4+ 10N3−N ≡6n5+ 15n4+ 10n3−n−v·2m = 30a+ (u−v)·2m (mod 2m+1).

Như cần chọnv =u ta cóN cần tìm

Mệnh đề 3) chứng minh cách hoàn toàn tương tự Ta hoàn tất việc chứng minhk= thỏa mãn điều kiệnT(20m)

(95)

Bình luận.

1. Các hệ số đa thức 6n5 + 15n4 + 10n3 −n đóng vai trị quan trọng bước chuyển từ m lên m+ 1, chẳng hạn từ mô-đun 2m lên mô-đun 2m+1, các hệ số 6 và 10

là chẵn nên có đồng dư:

6N5 ≡6n5 (mod 2m+1), 10N3 ≡10n3 (mod 2m+1) Với đồng dư thức 15N4 ≡ 15n4 (mod 2m+1) ta dùng đến nhị

thức Newton Tương tự cho trường hợp từ mô-đun 5m lên

5m+1,các hệ số 15và 10chia hết cho5, còn với hệ thức

6N5 ≡6n5 (mod 5m+1), ta dùng đến khai triển nhị thức Newton

2. Việc sử dụng nâng lũy thừa định lý Trung hoa số dư kỹ thuật số học mơ-đu-la Vì tốn khó bạn chưa quen với kỹ thuật này, ngược lại tập đơn giản Đây điểm yếu toán Trong thực tế chấm thi, số học sinh làm phần b)

Một số toán liên quan

1. (VMO 1997) Chứng minh với số nguyên dươngn, tồn số nguyên dương k cho 19k+ 97chia hết cho 2n.

2. (Saudi Arabia TST 2015) Cho n k số nguyên

dương Chứng minh n nguyên tố với

30 tồn số nguyên a b, số nguyên tố với n, cho a2−b2+k chia hết chon

Bài tốn 4. Có 100 sinh viên tham dự thi vấn đáp

Ban giám khảo gồm 25 thành viên Mỗi sinh viên hỏi thi giám khảo Biết sinh viên thích nhất10giám khảo số thành viên

a) Chứng minh chọn giám khảo mà thí sinh thích 1trong người

(96)

Lời giải. a) Cách 1. Gọi A1 giám khảo nhiều sinh viên

thích a1 số sinh viên thích A1 Khi đó, sinh

viên thích 10giám khảo, ta có

a1 ≥

100·10 25 = 40

Ta chọn giám khảo A1 loại tất sinh viên thích A1,

còn lại 100−a1 sinh viên Ta lại xétA2 giám khảo nhiều sinh viên lại thích gọi a2 số sinh viên

trong số sinh viên cịn lại thíchA2.Ta có

a2 ≥

(100−a1)·10

24

Ta có lúc

a1+a2 ≥a1+

(100−a1)·10

24 =

14a1 + 1000

24 ≥

14·40 + 1000 24 = 65

Ta chọn giám khảo A2 loại tất sinh viên thích A2,

còn lại 100−a1−a2 sinh viên

Tiếp theo, ta xétA3 giám khảo nhiều sinh viên

cịn lại thích gọi a3 số sinh viên số sinh viên

còn lại thích A3.Ta có

a3 ≥

(100−a1−a2)·10

23

Ta có lúc này:

a1+a2+a3 ≥a1+a2+

(100−a1−a2)·10

23 = 13(a1+a2) + 1000

23 ≥

13·65 + 1000

23 = 80,22

Suy raa1 +a2 +a3 ≥81 Hoàn toàn tương tự ta có

a1+a2+a3+a4 ≥a1+a2+a3+

(100−a1−a2−a3)·10

22 = 12(a1+a2+a3) + 1000

23 ≥

12·81 + 1000

23 = 89,63

Suy raa1+a2+a3+a4 ≥90.Tiếp tục vậy, ta cóa1+· · ·+a5 ≥95,

(97)

Cách 2. Cũng cách 1, ta gọi A1 giám khảo nhiều

sinh viên thích a1 số sinh viên thích A1 Khi đó,

mỗi sinh viên thích nhất10giám khảo, ta có

a1 ≥

100·10 25 = 40

Ta chọn giám khảo A1 loại tất sinh viên thích A1,

cịn lại tối đa 60 sinh viên Nếu số sinh viên là60,ta bổ sung thêm số sinh viên “ảo” cho đủ 60 Các sinh viên ảo thích tất giám khảo Rõ ràng ta chọn

giám khảo cho sinh viên kể sinh viên ảo

7 giám khảo phù hợp cho 100 sinh viên ban đầu Bổ sung thế, ta có60sinh viên, sinh viên thích nhất10giám khảo Ta gọi A2 giám khảo nhiều sinh viên số 60 sinh

viên (cả thật lẫn ảo, có) thích vàa2 số sinh viên thích giám khảo A2, ta có

a2 ≥

60·10 24 = 25

Ta lại chọn giám khảoA2 loại tất sinh viên thíchA2

Số sinh viên lại tối đa 60−25 = 35 Ta lại bổ sung số sinh viên ảo số sinh viên nhỏ 35, tiếp tục gọi A3, a3 có định nghĩa tương tự Ta có

a3 ≥

35·10

23 = 15,21

Suy raa3 ≥16.Ta lại chọnA3 loại tất sinh viên thích

A3 Số sinh viên cịn lại khơng q 35−16 = 19.Cứ tiếp tục

vậy ta

• a4 ≥9,số sinh viên lại sau chọnA4 loại tất

các sinh viên thíchA4 khơng q 10

• a5 ≥5,số sinh viên cịn lại sau chọnA5 loại tất

các sinh viên thíchA5 khơng q

• a6 ≥2,số sinh viên cịn lại sau chọnA6 loại tất

các sinh viên thíchA6 khơng q

• a7 ≥2,số sinh viên lại sau chọnA7 loại tất

(98)

Vậy sau chọn7giám khảo A1, A2, , A7 khơng cịn sinh

viên nào, tức sinh viên thích1 7giám khảo chọn (đpcm)

b)Ta xếp lịch thi cho thí sinh cho giám khảo hỏi thi khơng q10thí sinh, đến khơng thể thực dừng lại Giả sử cịn thí sinh A chưa thi Vì A thích 10 giám khảo nên ta xét 10 số giám khảo mà A thích Theo giả sử ta khơng thể cho A thi giám khảo đủ “quota”, tức hỏi đủ 10 thí sinh Vì thí sinh hỏi giám khảo nên số thí sinh hỏi thi 10×10 = 100, mâu thuẫn Vậy khơng cịn thí sinh khơng hỏi thi ta có điều phải chứng minh

Bình luận.

1. Câu a) tốn mơ hình hóa dạng tốn tập hợp: Có 100 tập A1, A2, , A100 tập hợp

X = {1, 2, , 25}, |Ai| ≥ 10 Khi tồn Y ⊂ X, |Y| =

sao cho Ai ∩Y 6= ∅ với i = 1, 2, , 100 Với mơ hình

này, ta chuyển sang mơ hình ma trận thuộc Chẳng hạn, ta theo hướng chứng minh phản chứng đếm số số ma trận thuộc: Giả sử ngược lại, không tồn tạiY thỏa mãn yêu cầu đề Khi với

cột ta ln tìm dịng mà giao dịng với cột số Suy số số dịng C257 Mặt khác, dịng có nhiều nhất15 số nên số bộ7số 0trên dịng nhiều là100C157 Nếu ta có C257 >100C157 suy mâu thuẫn Đáng tiếc (!) ta lại có C257 <100C157 nên cách không khả thi Thú vị cần giảm số giám khảo từ25xuống24thì thật C247 > 100C147 cách giải trót lọt Ngược lại, thay

25bằng 26 quy trình giải cho số giám khảo cần thiết là8 Như số25 chọn có chủ ý

2. Với ý trên, có hai câu hỏi đặt là:

1) Nếu cố định thông số 100 (số sinh viên), 25 (số giám khảo), 10 (số giám khảo tối thiểu mà sinh viên thích) thay 7bằng 6được khơng? 2) Nếu cố định thông số100(số sinh viên), 10(số giám

(99)

khảo cần chọn) thay25bằng26được khơng? Và thay số lớn bao nhiêu?

3. Cách giải câu b) trình bày cho thấy kết câu b) tầm thường thực dễ kỳ thi Xét tổng thể bối cảnh ngày thi thứ hai điều hợp lý Tuy nhiên, kết làm thí sinh lần cho thấy tổ hợp yếu điểm then chốt học sinh Việt Nam

Một số toán liên quan

1. (IMC 2002) 200 sinh viên tham dự thi toán Họ phải giải 6bài toán Biết tốn giải nhất120sinh viên Chứng minh tồn hai sinh viên mà hợp lại giải 6bài toán

2. (Putnam) Cho tập hữu hạnXcó|X|>10 A1, A2, , A1066

các tập củaXsao cho|Ai|> |X2| với mọii= 1,2, , 1066

Chứng minh tồn tập conA gồm10phần tử X cho |A∩Ai| ≥1 với i= 1, 2, , 1066

3. (Nga 1993) Mỗi cư dân thành phố N quen với

30% cư dân thành phố Một người dân bầu cử người quen người tranh cử Chứng minh tổ chức bầu cử thị trưởng thành phố gồm hai ứng cử viên, cho có nửa cư dân bầu

Bài toán 6. Tìm số nguyên dương n nhỏ cho tồn n

số thựca1, a2, , an thỏa mãn điều kiện:

a1+a2+· · ·+an >0, a31+a32+· · ·+a3n<0, a51+a52+· · ·+a5n>0

Lời giải. Ta chứng minh n = giá trị n nhỏ thỏa mãn

điều kiện đề Ta nhận xét tồn n số thực thỏa mãn điều kiện đề với m > n tồn m số thực thỏa mãn điều kiện đề (chỉ cần bổ sung thêm m−n số 0) Vì vậy, để chứng minh n = giá trị nhỏ thỏa mãn điều kiện đề bài, ta cần chứng minh:

1) Không tồn 4số thực a1, a2, a3, a4 cho

(100)

2) Tồn số thựca1, a2, a3, a4, a5 cho

a1+a2+· · ·+a5 >0, a31+a32+· · ·+a35 <0, a51+a52+· · ·+a55 >0

Ta chứng minh 1) 2) Để chứng minh 1), ta giả sử ngược lại tồn tại4 số thựca1, a2, a3, a4 thỏa mãn

a1+a2+a3+a4 >0, a31+a32+a33+a34 <0, a51+a25+a53+a54 >0

Khi số a1, a2, a3, a4 có số dương

nhất số âm

Trường hợp 1: Có số dương, số không dương. Không

tính tổng quát, giả sử a1 > ≥ a2, a3, a4 Đặt bi = −ai ta có

a1 > b2+b3+b4 suy

a31 >(b2+b3+b4)3 ≥b32+b33+b34 =−(a23+a33+a34)

Mâu thuẫn

Trường hợp 2: Có3 số khơng âm, 1số âm. a1, a2, a3 >0> a4 Đặt

b4 =−a4 Ta có

a1+a2+a3 > b4, a31+a32+a33 < b34, a51+a52+a53 > b54

Từ bất đẳng thức thứ hai suy raa1, a2, a3 < b4

a51+a52+a53 < a31b24+a32b24+a33b24 = (a31+a23+a33)b23 < b53 Mâu thuẫn

Trường hợp 3: Có2 số dương, số âm. Gọi hai số dương a, b,

hai số âm −c, −d ta cóa, b, c, d >0và

a+b > c+d, 5+b5 > c5+d5, a3+b3 < c3+d3

Ta chứng minh điều dẫn đến mâu thuẫn Khơng tính tổng qt, giả sử a≥b c≥d Thật vậy,

a3+b3 = (a+b)(a2−ab+b2)<(c+d)(c2−cd+d2)

vàa+b > c+d nên c2−cd+d2 > a2−ab+b2,suy

(101)

suy rac−d > a−b.Nếuc≤a từ suy rab−d > a−c≥0, suy b > d Từ a3+b3 > c3+d3, mâu thuẫn Vậy c > a Và ta cóc > a ≥b > d

Đến ta có số hướng giải khác sau:

Cách 1. ĐặtC = c

d, A= a d, B =

b

d C > A, B Xét hàm số

f(x) =Cx+ 1−Ax−Bx

thì theo điều kiện ta có f(1) < 0, f(3) >0, f(5) <0 Ngoài ra,

C > A, B nên f(+∞) = +∞ Theo định lý tính chất hàm liên

tục, phương trìnhf(x) = 0có nghiệm Áp dụng định lý Rolle, suy phương trình f0(x) = có hai nghiệm (∗)

Nhưng ta lại có

f0(x) =CxlnC−AxlnA−BxlnA

=Cx

1− A C x

lnA−

B C x lnB

Hàm ngoặc hàm liên tục đồng biến R+ đồng

biến nên có nhiều nhất1nghiệm, suy f0(x)có nhiều

1nghiệm, mâu thuẫn với (∗)

Cách 2. Ta có b≤c3+2d3

1

vàa <(c3+d3−b3)13.Do đó:

c3+d3−b3

5

3 +b5 > c5+d5. (∗∗)

Xét hàm số f(x) = (c3+d3−x3)53 +x5 trên

d,

c3+d3

2

1

Ta có

f0(x) = c

3+d3

−x3

2 3(

−3x2) + 5x4 = 5x2hx2− c3+d3−x3

2 3i

≤0

do x≤c3+d3

2

1

.Vì f hàm nghịch biến ta có f(x)< f(d) = c3 +d3

Do (∗∗)khơng thể xảy Mâu thuẫn

Cuối cùng, để hoàn tất phép chứng minh, ta chứng minh tồn số thực a1, a2, , a5 cho a1 + a2 + · · ·+ a5 > 0,

(102)

Ta số dương a, b, c, d, e cho a +b + c > d+ e,

a3 +b3 + c3 < d3 +e3, a5 +b5 + c5 > d5 +e5 Đầu tiên ta chọn

a= 2x, b =c= 1, d=e=x+ a+b+c=d+e

Ta chọnxsao cho(2x)3+2<2(x+1)3 và(2x)5+2>2(x+1)5.Giải bất phương trình này, ta chọn x = 1,5 Sau điều chỉnh d chút, ta chọn a = 3, b = c = 1, d = e = 2,45 Bài tốn giải hồn tồn

Bình luận.

1. Bài toán thú vị cách đặt vấn đề, mẻ cho dù bất đẳng thức nghiên cứu cách rầm rộ có nhiều cơng cụ mạnh trang bị cho học sinh Để giải này, học sinh cần áp dụng chút tư tổ hợp (trong việc phân trường hợp), phép chứng minh phản chứng biến đổi đại số (hoặc giải tích) tinh tế Việc xây dựng ví dụ địi hỏi khả dự đoán khoanh vùng

2. Tuy nhiên toán nặng kỹ thuật, khơng thật phù hợp cho tốn olympic, thí sinh phải làm điều kiện thời gian hạn chế không sử dụng công cụ trợ giúp

3. Trong trình tìm ví dụ cho trường hợpn= 5,một cách tự nhiên ta phát toán sau:Nếua, b, c, d, elà các số thực dương choa+b+c=d+e, a5 +b5+c5 =d5+e5

thìa3+b3 +c3 < d3+e3

4. Một cách tự nhiên, ta đặt câu hỏi tổng qt: Tìm số

nnhỏ cho tồn tạinsố thựca1, a2, , an có tổng các luỹ thừa bậc 4k + dương (k = 0, 1, , m) và tổng lũy thừa bậc4k−1âm (k = 1,2, , m)

Lời giải cho tốn tổng qt có lẽ khó Ngay trường hợp m= khó

5. Một câu hỏi khác xuất phát từ lời giải toán trên:

Cho a1, a2, , am;b1, b2, , bn m+n số thực dương Hỏi

phương trình Pmi=1ax i =

Pn

(103)

Một số tốn liên quan

1. (Ucraina 2014, lớp 9) Tìm tất số nguyên dươngnsao cho tồn n số thực thỏa mãn điều kiện a1 +· · ·+an >

vàa31+· · ·+a3n<0

(Bài đơn giản: Với n = 1, rõ ràng không thỏa mãn, cịn vớin = chọna1 =a2 = 23, a3 =−1.)

2. (IMC 2006) Cho a, b, c, d, e số thực dương cho a2+b2+c2 =d2+e2, a4+b4+c4 =d4+e4 Hãy so sánh số

a3+b3+c3 d3+e3

3. (Tournament of the Towns, 2006) Các số thực dươngx1, ,

xk thỏa mãn điều kiện

x21+· · ·+xk2 < x1+· · ·+xk

2 , x1+· · ·+xk<

x31+· · ·+x3k

2

a) Chứng minh rằngk >50

(104)(105)

N

GƯỜI L ÀM VƯỜN CỦA TOÁN HỌC

ĐẶNG NGUYỄN ĐỨCTIẾN

(Đại học Trento, Italy)

Roses are red,

Violets are blue, Sugar is sweet,

And so are you.

Có đồng dao thế, có vĩ nhân nguệch ngoạc viết thêm vào báo sau:

Roses are red

Violets are blue, One point 414

Is the square root of two.

Giới thiệu

Nền toán học kỷ hai mươi sản sinh nhiều đại thụ John Nash,Andrew Wiles, Grothendieck,Paul Erdăos, Alan Turing H u l nhng cỏi tờn vĩ đại mà tầm ảnh hưởng vượt xa khỏi thời đại Nhưng viết lại muốn giới thiệu đến vĩ nhân khác: Martin Gardner, nhân vật trọng yếu lịch sử tốn học thân lại khơng phải nhà tốn học, chí chưa kinh qua đào tạo toán

(106)

1 Giải toán theo cách Martin Gardner

Trước tiên, khởi động với ba toán theo phong cách Martin Gardner sau:

Bài toán 1. Góc hai đường thẳng màu đỏ vẽ từ hai

mặt khối hộp hình bao nhiêu?

Bài tốn 2. Xét vịng kim loại hình bên

đốt nóng bị nở Điều xảy với lỗ bên nó? Nó co lại, nở hay giữ ngun kích thước?

Bài tốn 3. Hai bù-loong giống hệt đặt lại đối

(107)

Đây số toán phổ biến theo phong cách Martin Gard-ner Nếu bạn đọc đọc qua sách Martin, hẳn nắm rõ câu trả lời Tuy nhiên, điều thú vị ba tốn chúng khơng phải dành riêng cho người nghiên cứu tốn Chúng giải có kiên nhẫn, đam mê tất nhiên với chút kiến thức phổ thông vật lý Chúng không hứa hẹn trao cho bạn khám phá vượt bậc nào, mà đơn giản, chúng mang lại cảm giác thoả mãn ngào cho tìm lời đáp Đúng nhưRobert P Crease viết: “Tìm kiếm câu trả lời Google là phong cách Gardner Cách Gardner tự đốt cháy niềm đam mê trải nhiệm niềm vui vào lúc tự tìm câu trả lời.”

Và cách đó, suốt 25 năm với chun mục “Các trị chơi toán học” tờ báo khoa học thường thức danh tiếng Sci-entific American, Gardner gieo mầm đam mê toán học cho hàng triệu người lứa tuổi, ông đem toán học đến với nhiều người lịch sử toán học làm

“Martin biến hàng ngàn đứa trẻ thành nhà toán học và hàng ngàn nhà toán học thành đứa trẻ”

(108)

2 Cuộc đời nghiệp

Martin Gardner sinh vào ngày 21 tháng 10 năm 1914 Tulsa, Oklahoma, ngày 22 tháng năm 2010, không xa nơi ông sinh ra, thành phố Norman, Oklahoma

Từ trái sang phải: Em trai, cha, Martin Gardner

Cha ơng có tiến sĩ địa chất mẹ ông dạy tiểu học Lexiton, sau bà nghỉ việc để chăm sóc cho ba anh em ơng Thuở nhỏ, Gardner thích chơi bài, trò ảo thuật đọc tác phẩm phiêu lưu “Phù thủy xứ Oz” nhà văn Braum, hay “Alice lạc vào xứ thần tiên” Lewis Caroll Chính thú đam mê trị chơi trí tuệ câu chuyện phiêu lưu thuở ấu thơ ảnh hướng lớn đến định gắn bó với Tốn học giải trí đời Gardner Ơng kể rằng:

“Mẹ tơi đọc “Phù thủy xứ Oz” cho nghe bé, đã nhìn qua vai mẹ lần bà đọc Tôi học chữ đấy.”

(109)

thông Đại học Chicago, năm 1937 ông trở lại Oklahoma trở thành phóng viên cho tờ báo Tulsa Tribune Sau ơng chuyển đến làm việc cho tạp chí Humpty Dumpty, tạp chí dành cho thiếu nhi Ông viết đặn tháng câu truyện ngắn thơ đưa lời khuyên đạo đức cho thiếu niên

Đến năm 1956, ông mời viết cho chuyên mục trò chơi toán học (Mathematical Games) với Scientific American Cũng từ nghiệp huyền thoại Gardner bắt đầu Cùng với niềm say mê ảo thuật đầu óc hiếu kỳ, Gardner trụ vững tạp chí khoa học có uy tín đặn đưa câu đố làm bối rối độc giả Lạ lùng, mẻ, sinh động dễ hiểu yếu tố khiến chuyên mục xuất đặn từ 1956 đến 1980 Gardner nhiều người đón đọc Chuyên mục ông mở cánh cổng dẫn đến giới toán học đầy rẫy điều thú vị Chúng không mang lại niềm vui cho người yêu toán hay nhà toán học mà cịn góp phần ni dưỡng tình u tốn cho hệ trẻ tuổi, truyền cho họ cảm hứng niềm say mê giải vấn đề

(110)

Martin nói với tơi ơng dành 25 ngày tháng cho chuyên mục tờ Scientific American.

Persi W Diaconis

Một bị khơng có kỹ giải vấn đề con tinh tinh, chúng biết cách dùng gậy để chọc vào tổ để đuổi đám mối khỏi mặt đất Sự tiến hóa phát triển khả năng của não để giải vấn đề, thời điểm niềm vui việc giải vấn đề tạo thành trong não chúng ta.

Martin Gardner Science Good, Bad and Bogus (1981), 123

Ông tin giây phút người tận hưởng niềm vui tìm đáp án vấn đề điều quan trọng văn hố nhân loại Ơng tin điều làm nên khác biệt xã hội công nghiệp đại với thời kỳ Hy Lạp cổ khả giải câu Một hình tivi thời đại, nỗ lực cá nhân hai mà thành hàng trăm người góp trí lực vào giải câu đố nhỏ Đối với ông niềm vui đến từ câu đố nhỏ nhặt Một thứ niềm vui khiết diệu kỳ không nhà khoa học phát kiến điều vĩ đại Đó thứ niềm vui mà Gardner truyền vào chuyên mục suốt phần tư kỷ

Ơng đơn giản hố phát kiến tốn học lớn lao Ơng nhìn vấn đề vĩ đại với mắt nhà khám phá viết lại chúng theo cách nhà văn kể chuyện xứ Oz Ông đặt lửa dẫn đường cho hệ trẻ tự mày mị say đắm giới thần tiên toán học

(111)

Trong lễ hội Toán học (1975) ông viết: “Tôi cách tốt nhất để đánh thức sinh viên đưa cho họ trị chơi tốn học hấp dẫn, câu đố, trị ảo thuật, câu nói đùa, nghịch lý, mơ hình, hay thơ câu (limerick) Không phải tơi đề nghị giáo viên khơng nên làm khác ngoại trừ giới thiệu trị chơi giải trí cho sinh viên Rõ ràng có một hốn đổi nghiêm túc tính giải trí Giải trí giữ cho người đọc tỉnh táo Sự nghiêm túc làm cho trò chơi đáng giá.”

Martin Gardner vợ

Bên cạnh Scientific American, Martin Gardner tác giả 70 đầu sách, trò chơi, ảo thuật, triết học, thiên văn, tôn giáo Ơng có bạn bè đồng nghiệp ông khắp lĩnh vực sáng tạo, từ nhà logic học, toán học nhưRaymond Smullyan, Roger Penrose, Piet Heinđến nhà văn Isaac Asimov Vladimir Nabokov, nghệ sĩ M C Escher Salvador Dalí Vì khơng có ngạc nhiên tác phẩm ơng bao hàm nhiều lĩnh vực khác Ngồi ông chuyên gia hàng đầu Mỹ Lewis Carroll – tác giả “Alice lạc vào xứ thần tiên”

(112)

3 Những hạt mầm khu vườn của

Martin Gardner

Trong phần tư kỷ (1956-1980), đề mục chung Các trị chơi tốn học, Martin Gardner làm say mê khơng biết bạn đọc trị chơi trí tuệ vừa dễ hiểu vừa sâu sắc gợi mở nhiều khí cạnh mẻ Tất chúng khơng trị chơi ơng sáng tạo mà phần lớn ơng tìm thấy sách Sau thơng qua trao đổi với nhà khoa học để nắm bắt ý tưởng, ơng trình bày lại chúng dạng trị chơi đơn giản

Trong phần viết, muốn giới thiệu đến độc giả vài toán quan trọng tiếng Gardner giới thiệu qua chuyên mục “Các trò chơi toán học" tờ Scientific American qua số sách ông

3.1 Flexagon

Flexagon viết tính từ Gardner thức tham gia Scientific American (ơng có trước vào năm 1952 “Các máy logic” làm việc Humpty Dumpty) Bài viết hoàn thành vào tháng 12 năm 1956, nhờ vậy, vào tháng Giêng 1957, chun mục “Các trị chơi tốn học” thức đời

Flexagon mẫu giấy/vải phẳng, gấp nếp hay uốn cong để lật hai mặt trước sau cách sáng tạo, độc đáo Ví dụ sau cho thấy cách tạo Flexagon với mảnh hình tam giác gấp “uốn cong” lại thành lục giác Sau đó, vài động tác, mặt thay đổi hốn đổi vị trí cho để tạo thành hình dạng khác Đơi người ta viết lên mặt Flexagon số hay lời tiên đoán vận may dùng trị chơi

Độc giả liên tưởng Flexagon với trị chơi “đơng tây nam bắc” quen thuộc Việt Nam

(113)

một nghiên cứu sinh đại học Princeton Chuyện đường từ Anh sang Mỹ, Stone tình cờ cắt mẫu báo lớn gấp nó, Flexagon đời Stone sau với bạn Princeton làJohn Tukey (nổi tiếng với phép biến đổi Fourier nhanh (FFT)), Bryant Tuckerman (nhà tô-pô học, người phát số nguyên tố Mersenne thứ 24), Richard Feynman

(114)

3.2 Bài toán thư ký

Năm 1960, tờ Scientific American, lần Mar-tin Gardner giới thiệu toán sau:

“Yêu cầu người lấy số mảnh giấy tùy ý viết vào mảnh số nguyên dương khác Các số nhỏ hay lớn đến kích thước googol (bắt đầu theo sau trăm số 0) chí lớn Các mảnh đặt úp mặt xuống xếp ngẫu nhiên mặt bàn Tại thời điểm bạn phép chọn mảnh lật lên Mục tiêu tốn tìm thời điểm kết thúc việc lật số để chọn mảnh giấy có số lớn Bạn khơng thể quay lại chọn cách mảnh giấy lật trước Nếu bạn lật tất mảnh giấy mảnh cuối mảnh chọn."

Một toán tương tự đề xuất bới Arthur Cayley vào năm 1875 tốn đưa từ trước lâu

Johannes Kepler Về sau, toán sau thường biết đến với tên "bài toán thư ký", chi tiết độc giả xemtại Bài tốn có lời giải đẹp Đầu tiên chọn lật hếtn/e mảnh (trong đóelà số lơgarit tự nhiên) Sau chọn mảnh số mảnh cịn lại có giá trị lớn tất mảnh lật Nếu áp dụng thuật tốn xác suất chọn mảnh có số lớn khoảng

1/evà xác suất tối ưu

Bài toán thư ký toán tiếng lý thuyết dừng tối ưu Bài toán nghiên cứu xác suất ứng dụng, thống kê, lý thuyết định

3.3 Các toán nghịch lý

Các toán chọn Gardner bao gồm nhiều lĩnh vực, nghịch lý vấn đề u thích ơng Một tốn kinh điển thường nhắc đến giáo trình xác suất nghịch lý ngày sinh:

(115)

Trái với dự đốn cảm tính thông thường (theo nguyên lý Dirichlet chẳng hạn), xác suất cao buổi tiệc có 70 người xác suất có người có ngày sinh lên đến 99.9% Chi tiết hơn, độc giả xem

Một nghịch lý tiếng khác Gardner giới thiệu vào tháng năm 1974 mà ngày gọi "Nghịch lý New-comb", bắt nguồn từ nhà toán học William Newcomb Bài toán sau:

Giả sử bạn tham gia trị chơi trước mặt bạn có hai chiếc hộp đậy kín Hộp chắn có chứa 1.000$, hộp thứ có thể trống, chứa 1.000.000$ Bạn phép chọn cả hai hộp chọn hộp thứ Luật chơi cho biết thêm là người dẫn trị dự đốn trước xem bạn chọn phương án nào: họ đoán bạn chọn hộp để trống hộp thứ 2, và họ đoán bạn chọn phương án chọn hộp thứ 2, họ sẽ đặt vào 1.000.000$ Nếu họ đoán bạn chọn ngẫu nhiên giữa hai phương án, họ để hộp trống Và quan trọng hơn, người dẫn trị ln đốn ý định bạn!

Các trường hợp có cho nghịch lý Newcomb

Dự đoán Lựa chọn Số tiền nhận

TH1 Cả hộp Cả hộp 1.000

TH2 Cả hộp Chỉ mở hộp

TH3 Chỉ mở hộp Cả hộp 1.001.000

TH4 Chỉ mở hộp Chỉ mở hộp 1.000.000

(116)

3.4 Bài tốn tơ màu

Trên Scientific American số tháng năm 1975, Gardner đăng đồ giấy đặt yêu cầu dùng màu khác để tô cho vùng cho hai vùng kề phải tô hai màu khác Trong trị chơi ơng cố tình chơi trị

"Cá tháng tư"và nói đồ với 110 vùng bên phản ví dụ cho định lý màu

Nguồn:Wolfram

Bài tốn tơ màu nêu khoảng năm 1840 biMăobius, cha ca di bng Măobius ni ting Gi thuyết định lý 4-màu phát biểu thức lần bởiGuthrie vào năm 1852, gọi định lý Guthrie, phát biểu với đồ phẳng tô màu cho vùng có đường biên không tô giống màu Hai số người tiên phong chứng minh định lý Alfred Kempe, người nghiên cứu luật, vào năm 1879, Peter Tait, nhà vật lý, vào năm 1880 Nhưng 10 năm sau, Percy Heawood sai lầm cách chứng minh Kempe đồng thời đưa định lý 5-màu Đến năm 1891,

(117)

Haken trường Đại học Illinois với trợ giúp máy tính Mặc dù nhiều nhà tốn học khơng chấp nhận kết nghi ngờ tính đắn xử lý máy tính vấn đề liên quan Năm 1996, chứng minh độc lập, ngắn gọn đưa Robertson cộng Chi tiết cho tốn tiếng này, độc giả xemtại Dưới cách tô màu đồ đặt câu hỏi Gardner, tìm thấy Stan Wagon, nhà tốn học quen thuộc với Epsilon hai số Mặc dù đồ phẳng chứng minh tô màu, thực tế việc tìm cách tơ với màu cho đồ cụ thể thường thách thức

Lời giải cho tốn tơ màu đồ Martin Gardner với màu Nguồn:Wolfram

3.5 Phân dạng

Phân dạng, hay quen thuộc Fractal hay hình (học) Frac-tal, hình có cấu trúc tự đồng dạng, giới thiệu Gardner vào tháng 12 năm 1976, với tên gọi ban đầu "đường cong quái vật" (Monster curve)

(118)

định nghĩa đường thằng đường phân dạng chuẩn: "các đường thẳng đường cong, phần tương tự với toàn bộ".Benoˆıt Mandelbrotlà người đưa khái niệm fractal, bắt nguồn từ tiếng La-tinh fractus nghĩa "đứt gãy"

Mandelbrot sống không xa Gardner viết, thời gian này, nhà riêng mình, Gardner giới thiệu Mandelbrot vớiConway, nhà toán học người Anh chuyên nghiên cứu lý thuyết số, mã hóa, tổ hợp đặc biệt người phát minh "trò chơi đời" Như Gardner thuật lại hồi ký mình, "Conway có phát cho tốn ghép hình Penrose (Penrose tiling), Mandelbrot quan tâm đến phát minh mẫu ghép Penrose có dạng Fractals Bạn mở rộng thu hẹp chúng, chúng ln ln có hình dạng tương tự."

3.6 Mã hóa RSA

(119)

Ý tưởng RSA độc giả tìm hiểuở Để chứng tỏ khả RSA, tác giả gửi cho Gardner thông điệp E mã hóa gồm 128 ký tự, với khóa cơng khai s = 9007 trao giải thưởng 100 $ cho giải mã thông điệp gốc E

Thơng điệp mã hóa thách thức Gardner gửi đến độc giả.

Mật mã RSA trở thành tiêu chuẩn công nghiệp biến thể sử dụng ngày hôm Trong thời gian gần câu hỏi độ an tồn xem xét lại Và viết Gardner mang tính chất đột phá, tiêu đề khơng hồn tồn xác Thơng điệp thách thức giải mã thành công vào khoảng tháng năm 1994, 17 năm sau toán đăng thay hàng triệu năm

Ngồi vấn đề nêu trên, nhiều toán tiếng quan trọng khác Gardner phổ biến đến đông đảo độc định lý lớn Fermat, trò chơi đời, giả thuyết 3n + hẳn độc giả cịn nhớ, "bài tốn đội nón" mà giới thiệu hai số Epsilon ươm mầm từ người bạn tốt mà tốn học có

(120)

tình yêu Gardner tác phẩm nhà văn tiếng người Anh, Lewis Carroll (cũng nhà toán học Charles Dodgson) Những giải Gardner giải đáp ẩn ý lơ-gic tốn học “Alice lạc vào xứ thần tiên” "Through the Looking-Glass"

Gardner Alice Central Park

2 Bên cạnh niềm say mê toán học, Gardner bút nhiệt tình việc lên tiếng chống lại tượng mạo danh khoa học Trong số 100 sách, “Những trò lố bịch nhân danh khoa học” nói nhiều lỗi khoa học trò thủ đoạn Năm 1976, Gardner tham gia vào nhóm Carl Sagan, Isaac Asimov nhiều người khác thành lập Ủy ban điều tra tượng huyền bí, siêu nhiên

3 Ơng xuất báo khoa học (có bình duyệt) tuổi 74! Bài báo ông viết chung với cặp vợ chồngFan Chung

(121)

5 Chú thích

Vì viết trích dẫn đề cập đến nhiều nhà khoa học, nhà văn, nghệ sĩ khác nên liên kết trực tiếp tiểu sử người thông qua Wikipedia mà không sử dụng thích

Bài viết tổng hợp biên dịch từ nguồn sau: BBC -Martin Gardner, puzzle master extraordinaire Tham

khảo chủ yếu cho phần

2 Huffingtonpost -Martin Gardner – The Best Friend Mathe-matics Ever Had- Colm Mulcahy Tham khảo chủ yếu cho phần

3 AMS -Magical Mathematics - A Tribute to Martin Gardner

- Joseph Malkevitch Tham khảo chủ yếu cho phần Tia sáng - Martin Gardner Người gợi cảm hứng cho niềm

say mê toán học- Ngọc Tú Tham khảo chủ yếu cho phần

2

5 New York Times - Martin Gardner, Puzzler and Polymath, Dies at 95- Douglas Martin Tham khảo chủ yếu cho phần

2

6 Scientific American (blog) - The Top 10 Martin Gardner Scientific American Articles - Colm Mulcahy Tham khảo chủ yếu cho phần3

7 Trò chơi đời - Hà Dương Tường Tham khảo chủ yếu cho phần2

8 Recreational Mathematics Magazine No 2014 - Martin Gardner’s Mathemagical Life- Tereza Bártlová Tham khảo cho toàn

(122)(123)

TRẦN MINH HIỀN

(Trường THPT Chuyên Quang Trung, Bình Phước)

1 Một số định lý quan trọng

Định nghĩa 1.1. Cho F họ tập hợp tập n phần tửX Khi F gọi họ giao với A, B ∈ F ta cóATB 6=∅

Định lý 1.2. Cho F họ giao tập n phần tử X Khi |F| ≤2n−1.

Chứng minh. Lấy tập A ⊆ X Khi với cặp tập (A, X\A)của X, nhiều hai tậpA X\A thuộc vào F (vì hai tập A, X\A đều thuộc vào F thì do

A∩(X\A) = ∅

mâu thuẫn vớiF là họ giao tập củaX) VìX có2ntập

con, cặp tập (A, X\A) có nhiều tập thuộc

F Do

|F| ≤ 12 ·2n = 2n−1.

Định lý 1.3 (Định lý Erdos-Ko-Rado). Một họF cáck_tập (một tập hợp có k phần tử gọi k_tập) tập n phần tử X (k ≤ n

2) Khi

|F| ≤

n−1

k−1

Chứng minh. Với n, k số nguyên dương với n ≥ 2k Mộtk_cung tập{i, i+ 1, , i+k}, với số nguyên lấy theo modulo n Một cách hình dung cho k_cung k đoạn cung tròn liên tiếp, nối hai điểm i i+

k(modn)trên đường trịn Ta nói hai cungAvàA0 giao nhau

(124)

2 Một họ {A1, A2, , At} k_cung giao đơi

[n] t ≤ k Thật vậy, điểm i (điểm gắn cho cung tròn) điểm kết thúc hai cung: cung nhận i điểm cung nhận ilà điểm cuối

Do hai cung không giao nhau, dẫn đến nhiều nhất

một hai cung thuộc vào họ F Xét cung

A1 cho trước Vì cung cịn lại phải giao với A1

(125)

trong điểm cungA1, điểm i1, i2, , ik−1,

là điểm kết thúc Vì điểm kết thúc có khơng q k_cung thuộc F, nên có k −1 điểm kết thúc có khơng q k_cung thuộc F, cộng với cung A1 có khơng q

k_cung thuộc tập F

3 Bây xét một hốn vị [n] có dạng (a1, a2, , an) Ta

đánh dấu đoạn cung tròn số hình vẽ

ban đầu Mỗi tập F xem k_cung của

hoán vị này Theo kết trên, với hốn vị ta

có ≤k k_cung

• Lấy tổng hết tất hốn vị [n] đường trịn, có

(n−1)! hốn vị, ta thấy có nhiều k(n −1)!

các tập

• Tuy nhiên, xếp đường trịn cách đếm trên, tập A1 đếm làm nhiều lần (vì xếp

trên đường trịn, tập A1 hay tập lấy để

làm thứ tự giống nhau) Vì nhiều lần hoán vị của(n−1)!, tạo hoán vị khác nhau, phần tử A1 Do có k! cách xếp

các phần tử A1 (n−k)! cách xếp phần

tử bù tập A1 Do

(126)

⇒ |F| ≤ k(n−1)!

k!(n−k)! =

n−1

k−1

Định lý 1.4 (Bollobas). Cho A1, A2, , Am B1, B2, , Bm

các tập S = [n] Đặt = |Ai| bi = |Bi| với i =

1,2, , m Giả sửAi∩Bj =∅ khii=j Khi

m

X

i=1

ai+bi

ai

−1

≤1

Chứng minh. Với hoán vị sốn!hốn vị phần tử S, ta nói hoán vị π chứa B sau A tất phần tử A đứng trước phần tử B π

2 Với hốn vị π có tính chất chứaBi sau Ai, mà có

thêm tính chất chứa Bj sau Aj, Ai∩Bj = ∅ (nếu Ai

đứng trướcBj, minh họa hình dưới)

hoặc Aj ∩Bi = ∅ (khi Ai kết thúc sau Bj) Nhưng điều

mâu thuẫn với giả thiết Ai∩Bj =∅khi i =j Do

đó, với hốn vị π, tồn tạinhiều một chỉ sối để π chứa Bi sau Ai

3 Ngược lại, với i∈ {1,2, , m}, ta đếm số hoán vị màAi

đứng trướcBi

• Chọn ai+bi ví trí để xếp cho Ai Bi, ta có ai+nbi

cách

• Đặtai phần tử củaAi vàoai vị trí ai+bi

ví trí vừa chọn có ai!cách, sau xếp bi phần

tử Bi vào vị trí cịn lại cóbi! cách Vậy cóai!.bi!

(127)

• Sắp xếp n−ai−bi phần tử lại củaS vàon−ai−bi

vị trí cịn lại có (n−ai −bi)!cách

Vậy ta có

n ai+bi

·ai!·bi!·(n−ai−bi)! =

n!

ai+bi

Lấy tổng số tất các số i= 1,2, , mta có

m

X

i=1

n!

ai+bi

≤n!⇒

m

X

i=1

ai+bi

ai

−1

≤1 Định lý chứng minh

Định nghĩa 1.5. ChoS tập hợp hữu hạn Một họ tập

con A1, A2, , An củaS gọi xíchnếu

Ai ⊂Ai+1 |Ai+1|=|Ai|+ ∀i= 1,2, , n

Do xích poset tập hợp {x1, x2, , xk}

thỏa mãn

x1 ≤x2 ≤ .≤xk

Định nghĩa 1.6. Cho P tập hợp hữu hạn Một họ tập

con A1, A2, , An củaS gọi phản xích

Ai 6⊂Aj,∀i6=j

Khi phản xích poset tập phần

tử y1, y2, , yk mà yi, yj so sánh theo quan hệ

poset với i6=j

Định lý 1.7 (Bất đẳng thức LYM, Lubell, Yamamoto, Me-shalkin). Cho F một phản xích [n] (ta dùng ký hiệu

[n] thay cho {1,2, , n}) Khi ta có bất đẳng thức X

a∈F

n

|A|

−1

≤1

Chứng minh. Gọi C tập hợp tất xích C, xích C gồm n tập hợp, tập hợp tập [n] xích chứa nhiều phần tử nhất, C1 ⊂ C2 ⊂ ⊂ Cn,

|Ci| = i, ∀i = 1,2, , n Hỏi C có phần tử? (mỗi

(128)

• Có n cách chọn cho tập C1 Thật vậy, |C1| = nên

C1 ∈ {{1},{2}, ,{n}}

• Với cách chọn tập C1, có n−1 cách chọn tập C2

Thật vậy, minh họa cho C1 ={3},

C2 ∈ {{3,1},{3,2},{3,4}, ,{3, n}}

• Cứ tiếp tục vậy, ta cón−2cách chọn tậpC3,n−3

cách chọn tập C4, cuối cách chọn tập

Cn Cn = [n]

VậyC có n! phần tử

2 Một tập A ⊂ [n] A vừa thuộc vào xích C, vừa thuộc vàođối xích F Do ta đếm số cặpN gồm

(A, C) với C ∈ C A tập hợp vừa thuộc xích C vừa thuộc đối xíchF

• Với C ∈ C, có nhiều tập A mà A ∈ C A∈F Thật vậy, giả sử có hai tập A1, A2 vừa thuộc vào

C, vừa thuộc vào F Vì A1, A2 thuộc xích C nên A1 ⊂ A2 A2 ⊂ A1, A1, A2 khơng

thể thuộc vào F được, tập F khơng có tập tập tập khác Vậy

|N| ≤n!

• Với tập A ∈ F mà |A| = k Khi có k! cách chọn tập A1, A2, , Ak−1 (cách đếm giống hệ ý 1) mà

A1 ⊂A2 ⊂ .⊂Ak−1 ⊂A, |Ai|=i,∀i= 1,2, , k−1

và có (n−k)!cách chọn tập Ak+1, An mà

A⊂Ak+1 ⊂ .⊂An, |Aj|=j,∀j =k+ 1, k+ 2, , n

Suy A∈F ta có

k!(n−k)! =|A|!.(n− |A|)!

cách chọn xích C chứa A Do

|N|=X

A∈F

(129)

Từ hai cách đếm trên, ta có X

A∈F

|A|!(n− |A|)!≤n!⇒ X

A∈F

|A|!(n− |A|)!

n! ≤1

đây điều phải chứng minh

Nhận xét 1. Bất đẳng thức LYM dễ dàng suy từ định lý Bollobas, định lý 1.4 cách, đặt F = {A1, A2, , Am}

đặt Bi = [n]\Ai Khi điều kiện Ai ∩Bi = ∅ thỏa mãn điều

kiệnAi∩Bj 6=∅trở thànhAi 6⊆Aj, tức điều kiện phản

xích Chú ý bi =n−ai Đến m X i=1 n −1 = n X i=1

ai+bi

ai

−1

≤1

Định lý 1.8. Một họ F tập tập n phần tử X gọi không so sánh được A, B hai phần tử

F A6⊆B

Định lý 1.9(Định lý Sperner). ChoF họ không so sánh tập tậpn phần tử X Khi

|F| ≤C[

n

2]

n

Chứng minh. Mặt khác, theo tính chất nhị thức Newton hệ số khai triển (1 +x)n thỏa tính chất

n < n < n

< <

n

n−1

2 < n n

Áp dụng vào ta có

n kj < n n Thay vào (*) ta có đánh giá

m 1n

[n 2] ≤ m X j=1 n kj

≤1

Từ ta có điều phải chứng minh Dấu xảy khiB chứa tất tập gồm có hn

2

i

(130)

Hệ 1.10 (Bất đẳng thức Lubell). Cho F họ không so sánh tập tập n phần tử X Gọiak số

k_tập thuộcF Khi

n

X

k=1

ak

Ck

n

≤1

2 Một số phương pháp giải toán cực trị

tập hợp

2.1 Khoảng cách Hamming - chặn Plotkin

Cho n số nguyên dương, ký hiệu

[0,1]n =

{x1x2 xn|xi ∈ {0,1}, i= 1,2, , n}

là tập tất xâu nhị phân độ dài n

Định nghĩa 2.1. Cho hai xâu nhị phân x = x1 xn y =

y1 yn Khi khoảng cách hai xâux y

d(x, y) =số vị trí i mà xi 6=yi

Khoảng cách gọi khoảng cách Hamming thỏa tất điều kiện

• d(x, x) = 0,∀x∈[0,1]n;

• d(x, y)>0,∀x6=y ∈[0,1]n;

• d(x, y) =d(y, x),∀x, y ∈[0,1]n;

• d(x, z)≤d(x, y) +d(y, z),∀x, y, z ∈[0,1]n.

Định nghĩa 2.2. Cho d số nguyên dương Gọi C tập hợp tất các xâu nhị phân trong[0,1]n sao cho

d(x, y)≥d,∀x6=y∈C

(131)

1 Nếu d chẵn,2d > n

M ≤2

d

2d−n

2 Nếu d lẻ, 2d+ 1> n

M ≤2

d+

2d+ 1−n

Để chứng minh định lý, ta sử dụng số nhận xét sau:

Bổ đề 2.4. Giả sử N, M số nguyên dương ≤ N ≤ M

N(M−N)≤

(

M2

4 M chẵn

M2−1

4 M lẻ

Bổ đề 2.5. Nếux∈R [2x]≤2[x] +

Bổ đề 2.6. Cho v xâu nhị phân∈[0,1]n Khi đó d(v+w)bằng

với số số xuất trongv+w

Chứng minh. Bằng việc kiểm tra tất khả vi

wi, ta thấy

(v+w)i =

(

0 vi =wi

1 vi 6=wi

Do

d(v, w) =|{i|vi 6=wi}|

=|{i|(v+w)i = 1}|

Chứng minh định lý vớidchẵn. Ta viết ma trận A = M2×n, với dòng phần tửu+v, vớiu, v ∈C Tađếm số lần xuất hiện số ma trận trên.

(132)

cặpu, w chov+wcó số vị trí j làN(M−N) Khi số số xuất cột thứj

N(M−N)≤

(M2

4 M chẵn

M2−1

4 M lẻ

Điều cho j = 1,2, , n Do số số xuất ma trậnA

(

nM42 M chẵn

nM24−1 M lẻ

2 Ta đếm số lần xuất số dòng Với dòng chứau+w Ta biết số số dịng d(v, w) Mà theo giả thiết d(v, w) ≥ d Do có d số dịng Do số số A

M

2

·d Từ ta thấy

1 Nếu M chẵn Khi kết hợp hai kết

M

2

≤n·M

2 ⇒ dM 2 − dM

2 ≤n·

M2

4 ⇒(2d−n)M2 ≤2dM

Theo giả thiết thì2d−n M số nguyên dương, nên

M ≤ 2d

2d−n Lại vìM số nguyên dương nên

M ≤

2d

2d−n

≤2

d

2d−n

+ 1;

mà M chẵn, d

2d−n

+ lẻ nên

M ≤2

d

2d−n

(133)

2 Nếu M lẻ, d· M

≤nM

2−1

4 ⇒d·

M

2 ≤n

M +

4

Do (2d−n)M ≤n nên

M ≤ n

2d−n =

2d

2d−n −1 Do M số nguyên, ta

M ≤

2d

2d−n −1

=

2d

2d−n

−1

≤2

d

2d−n

+ 1−1 =

d

2d−n

,

tức có chặn Plotkin cho dchẵn

Ví dụ 2.7 (Vĩnh Phúc 2012). Có em học sinh tham gia vào m nhóm hoạt động ngoại khóa, học sinh tham gia nhiều nhóm hoạt động Biết với hai nhóm tùy ý có học sinh tham gia vào hai nhóm đó Tìm giá trị lớn có m

Chứng minh. Gọi bảy học sinh a1, a2, , a7 đặt X tập

hợp học sinh Tamỗi nhóm xâu nhị phân

x1x2 x7,

trong xi = nhóm chứa xi = nhóm

khơng chứa Xét hai nhóm A, B tùy ý số m nhóm,

đó có học sinh, giả sử a1, a2, a3, a4 Gọi hai xâu biểu diễn cho A, B làv, w

• Nếu{a1, a2, a3, a4} ∈Athìai 6∈B,∀i= 1,2,3,4 Khi hai xâu

(134)

Khi đód(v, w)≥4

• Nếua1 ∈A,{a2, a3, a4} ∈B Khi hai xâu có dạng

Khi đód(u, v)≥4

Xét tất trường hợp tương tự, ta ln cód(v, w)≥4với v, w Vậy d= 4, n = Khi theo định lý 3.3

m ≤2

4 2.4−7

=

Khi m ≤ Với m = ta có cách xây dựng nhóm là,

với a, b, c, d, e, f, h học sinh

A1: a b c

A2: a d e

A3: a f g

A4: b d f

A5: b e g

A6: c d f

A7: c e f

A8: a b c d e f g

Ví dụ 2.8 (China TST 1994). Cho A1, A2, , Ak tập

củaX ={1,2, ,10} cho

(

|Ai|= 5,∀i= 1,2, , k

|Ai∩Aj| ≤2,∀1≤i < j ≤k

(135)

Chứng minh. Ta coi tập At xâu nhị phân dạng

x1 x10, đóxi = i∈At vàxi = i6∈At Do

|Ai∩Aj| ≤2,∀1≤i < j ≤k

nên suy rad(x, y)≥6

(Giả sử xâu v biểu diễn choAi, xâu wbiểu diễn cho Aj Nếu|Ai ∩

Aj|= 2thì có hai xâuv, wchỉ cóđúng hai số trùng vị trí. Như hình bên minh họa cho hai số bên trái Đối với xâuv cịn số 1, tương ứng vị trí xâu w phải số và ngược lại xâuwcịn số 1, tương ứng ví trí xâuv

phải số Trong trường hợp làd(v, w) = 6 Nếu|Ai∩Aj|= hayAi∩Aj =∅thì rõ ràng d(x, y)>6.) Theo định lý 3.3

k≤2

6 2.6−10

=

Vậy k lớn 6, cách xây dựng tập

A1:

A2:

A3: 10

A4:

A5: 10

A6: 10

(136)

A1:

A2:

A3: 10

A4: 10

A5: 10

A6:

Ví dụ 2.9 (PTNK 2012). Cho số nguyên dương n tập hợp

X = {1,2,3, ,4n} Hai tập A, B X gọi không

giống nhau

|A∆B| ≥2n+ 1,

vớiA∆B = (A\B)∪(B\A) Xétm tập hợpA1, A2, , Am tập

con đôi không giống củaX Chứng minh

m≤ 4(n+ 1)

3

Chứng minh. Ta đồng tập At xâu nhị phân độ

dài4n

x1x2 x4n

với xi = i ∈At xi = i 6∈At Xét hai tập Ai, Aj tùy ý

trong m tập, gọi v, w hai xâu nhị phân biểu diễn chúng Do

|A∆B| ≥2n+

nên tương tự ví dụ 3.1

d(v, w)≥2n+

Từ suy d = 2n+ Do 2.d = 4n+ > 4n, nên theo định lý 3.3

m≤2

2n+ + 4n+ + 1−4n

=

2n+

≤ 4(n3+ 1)

(137)

giám khảo tùy ý, số thí sinh mà họ cho kết giống nhau

nhiều là k Chứng minh

k

n ≥

p−2

2(p−1)

Chứng minh. Giả sử n thí sinh S1, S2, , Sn Mỗi giám khảo

cho tương ứng với xâu nhịn phân độ dài n: x1x2 xn

với xi = nếu thí sinh Si đỗ và xi = nếu thí sinh Si rớt

Theo điều kiện tốn d=n−k Xét hai trường hợp Nếu 2(n−k)≤n

k

n ≥

1

k >

p−2

2(p−1)

2 Nếu 2(n−k)> n, xét hai khả xảy

• Nếu n−k chẵn, theo định lý 3.3

p≤2

n−k

2(n−k)−n

≤2 n−k

2(n−k)−n =

2(n−k)

n−2k

Do

p(n−2k)≤2n−2k ⇒ k

n ≥

p−2

2(p−1) • Nếu n−k lẻ, theo định lý 3.3

p≤2

n−k+

2(n−k) + 1−n

≤2 n−k+

2(n−k) + 1−n =

2(n−k+ 1)

n−2k+

Do

p(n−2k+1) ≤2n−2k+2⇒ k

n ≥

p−2

2(p−1)

n+

n >

p−2

2(p−1)

Bài tốn chứng minh hồn tồn

2.2 Sử dụng nguyên lý tổ hợp

(138)

Giải. Ta có

A={1,3,7,9,11,13,17,19,21,23,27,29},|A|= 12 Xét tập

B ={9,11,13,17,19,21,23,29}

Khi hai phần tử thuộcB khơng chia hết cho Từ ta suy k ≥ Ta chứng minh k = thỏa đề Xét S tập A |S| = Xét ba cặp

{21,7},{27,9},{1,11}ta thấy cặp bội Nếu cặp có cặp thuộc S tốn giải Giả sử ba cặp khơng có cặp thuộc S, |S| = nên S phải chứa số cặp chứa số cịn lại Từ suy trongS phải có cặp{3,9} hoặc{3,27}và cặp bội Hay nói cách khác S tồn hai số chia hết cho Vậy mink =

Nhận xét 2. Mấu chốt toán phát tập A0 để từ ta khẳng định k ≥ dự đốn

mink = Để tìm tập A0, ta liệt kê hết số A mà khơng

có hai số bội Với tốn này, việc tìm tập A0 đơn giản

Ví dụ 2.12 (KMO 1990). Cho n tập hợp A1, A2, , An thỏa mãn

    

|Ai|= 30,∀i= 1,2, , n

|Ai∩Aj|= 1,∀i6=j

A1∩A2∩ · · · ∩An=∅

Chứng minh n <872

Chứng minh. Giả sửn ≥872 Xét tập hợp A1,|A1|= 30 Do

|Ai∩A1|= 1,∀i= 2,3, , n

nên theo nguyên lý Dirichlet, tồn phần tửa ∈A1 thuộc

vào

hn

30

i

+ 1≥

872 30

+ = 30

(139)

2 Vì A1∩A2∩ .∩An =∅, nên tồn tập B số

tậpA32, , An không chứa phần tử a

3 Xét 31 tập hợpA2, A3, , A31 vàB với a∈Aj,∀j = 2,3, ,30

vàa6∈B

• Vì |Aj∩B| = 1,∀j = 2,3, ,31, tập Aj chứa a,

cịn B khơng chứa a, nên {xj}=Aj ∩B xj ∈B\{a}

• Có 29 phần tử x2, , x29 tập B\{a}, phần tử chúng thuộc vào tập hợp 30 tập Aj, j ∈ {2, ,31}, nên tồn phần tử xt,

với t ∈ {2,3, ,29} thuộc vào hai tập hợp Ar, As (r, s ∈

{2,3, ,31})

• Khi {a, xt} ⊂ Ar∩As, mâu thuẫn với giả thiết |Ar ∩

As|=

Vậy điều giả sử sai Bài tốn chứng minh

Ví dụ 2.13 (China TST1994, Ví dụ 3.1.2). ChoA1, A2, , Ak

các tập X={1,2, ,10} cho (

|Ai| ≥5,∀i= 1,2, , k

|Ai∩Aj| ≤2,∀1≤i < j ≤k

Tìm giá trị lớn có k

Chứng minh. Trong ví dụ 3.1.2 ta giải toán chặn Plotkin, xây dựng với k = Do k ≥ Nếu k ≥ 7, ta trình bày lời giải kết hợp đếm số tập hợp chứa phần tử nguyên lý bao hàm loại trừ (inclusion - exclusion principle) Với mỗii∈X, đặt

ni =|{j ∈ {1,2, , k|i∈Aj}}|

tức đếm số tập hợp trongA1, A2, , Ak chứa phần tử i Khi

10

X

i=1

ni =

k

X

j=1

|Ak≥5k = 35

Do phải tồn số i0 cho ni0 ≥ 4, tất ni ≤

thìP10i=1ni ≤3×10 = 30, mâu thuẫn với đánh giá Tức tồn

(140)

tậpA1, A2, , Ak Khơng tính tổng quát, giả sửi0 thuộc vào

4 tập hợp A1, A2, A3, A4 Khi với 1≤i < j < t≤4

|Ai∩Aj ∩At| ≥ |A1∩A2∩A3∩A4| ≥1

Theo nguyên lý IE

10 = |X| ≥ |A1∪A2∪A3∪A4|

=

4

X

i=1

|Ai| −

X

1≤i<j≤4

|Ai∩Aj|+

X

1≤i<j<t≤4

|Ai∩Aj∩At|−

|A1∩A2∩A3∩A4|

≥4×5−

4

×2 +

−1

×1 = 11,

mâu thuẫn Do điều giả sử sai Suy rak≤6 Vậyk=

Ví dụ 2.14 (India Postal Coaching 2014 Set Problem 4, Ví dụ 3.3.3). Tập M viết dạng M = A1 ∪A2 ∪ ∪An

và Ai ∩Aj = ∅ với ≤ i < j ≤ n, tập A1, A2, , An

được gọi n_phân hoạch M Giả sử A1, A2, , An

B1, B2, , Bn hai n_phân hoạch củaM thỏa mãn

|Ai∪Bj| ≥n,∀1≤i, j ≤n

Chứng minh |M| ≥ n

2

2

Dưới ta trình bày lời giải khác cách cơng thức IE.

Chứng minh. Đặt k = min{|Ai|i = 1,2, , n} Khơng tính

tổng quát, giả sử|A1|=k Khi

|M|=|A1 ∪A2∪ .∪An|=|A1|+· · ·+|An| ≥n|A1| ⇒ |A1| ≤

M

n

Tương tự đặt j = min{|Bi|i = 1,2, , n} Khơng tính tổng

quát, giả sử|B1|=j tương tự đánh giá

|B1| ≤

M

n

Theo giả thiết tốn |A1 ∪B1| ≥ n nên theo cơng thức IE

thì

n≤ |A1∪B1|=|A1|+|B1| − |A1∩B1| ≤ |A1|+|B1| ≤

2|M|

n ⇒M ≥

n2

(141)

Ví dụ 2.15. Cho 2005 tập hợp, tập hợp có có 44 phần tử Biết hai tập hợp có phần tử chung Chứng minh tồn phần tử thuộc tất 2005 tập hợp cho

Ta cần khẳng định tồn phần tử thuộc tất 2005 tập hợp đã cho Do ta làm việc với phần tử x thuộc nhiều tập hợp (vì có phần tử có cơi hội nhiều thỏa đề bài) Để làm điều này, ta phải biết được x thuộc bao nhiêu tập hợp Nếuxthuộc 2005 tập hợp, lập luận dẫn đến vô lý.

Giải. Ta có 2005 tập hợp, tập có 44 phần tử nên có tối đa 2005×44 phần tử Với x phần tử chúng, đặt

Ax ={số tập hợp 2005 tập hợp mà chứa phần tử x} ∈N

Vì {Ax}x ⊂ N hữu hạn nên tồn phần tử lớn trong

chúng, mà ta giả sử x Tức x phần tử thuộc nhiều tập hợp nhất, gọi tập hợp A1, A2, , Ak Nếuk = 2005

thì toán chứng minh Giả sử k <2005, tức tồn tập hợpB,trong số 2005 tập hợp, không chứa x Theo giả thiết toán

x1 =B∩A1 6=x,

x2 =B∩A2 6=x,

dễ thấy x1 6=x2 khơng x1 = x2 ∈A1 ∩A2,

theo điều kiện tốn thìA1∩A2 =x, Tương tự ta có B chứa phần tửx1, x2, , xk Tuy nhiênB chứa

44 phần tử nên k ≤44 Vì phần tử B khơng thuộc q 44 tập hợp (vì phần tử x thuộc nhiều tập hợp là k tập hợp k ≤44) nên B có giao khác rỗng với nhiều

442 = 1936<2004

tập hợp, mâu thuẫn với giả thiết B có giao khác rỗng với 2004 tập hợp cịn lại

(142)

Ví dụ 2.16. Cho 2014 thùng trái cây, giỏ trái có ba loại trái Táo, Xồi, Cam Chứng minh chọn 1008 thùng trái cây, cho tổng số Xoài, tổng số Táo, tổng số Cam 1008 thùng trái lớn nửa tổng số Xoài, tổng số Táo, tổng số Cam 2014 thùng trái ban đầu

Chứng minh. Đánh sốA1, A2,· · · , A2014 2014 thùng trái

Gọi (ai, bi, ci)lần lượt số táo, xoài, cam thùng Ai Chọn

ra thùng Ai, Aj có số táo số xồi lớn nhất Nếu i = j

ta chọn thùng Ai thùng Gọi A B số táo số xoài lớn 2012 thùng lại Ta chứng minh 2012 thùng cịn lại, chia vào nhóm I, II (mỗi nhóm có 1006 thùng) cho

      

Pi∈Iai −

P

i∈II

ai ≤A

Pi∈Ibi−

P

i∈II

bi

≤B

Giả sử a1 ≥ a2 ≥ ≥ a2012, ta xét cặp Xi = (a2i−1, a2i),(i =

1,2, ,1006) Với cặp

(

a2i−1 ∈Xi

a2i ∈Xi

ta cho cặp vào nhóm Khi với cách chia X

i∈I

ai−

X

i∈II

ai ≤a1+a3+· · ·+a2011−a2−a4− · · · −a2012

=a1+ (a3−a2) +· · ·+ (a2011−a2010)−a2012 ≤a1;

X

i∈I

ai−

X

i∈II

ai ≥a2+a4+· · ·+a2012−a1−a3− · · · −a2011

=−a1+ (a2−a3) +· · ·+ (a2010−a2011) +a2012 ≥ −a1

Nên với cách chia X

i∈I

ai−

X

i∈II

ai ≤A GọiT cách chia Nếu

X

i∈I

bi−

X

i∈II

(143)

thì chọn cách Nếu X

i∈I

bi−

X

i∈II

bi > B

khơng tính tổng qt, giả sử X

i∈I

bi >

X

i∈II

bi ⇒

X

i∈I

bi−

X

i∈II

bi > B

Khi tồn tạij ∈I, k ∈II cho bj > bk Khi ta việc đổi

chỗ (aj, bj) từ nhóm I sang nhóm II (ak, bk) từ nhóm II sang

nhóm I (cách đổi chỗ khơng ảnh hưởng đến điều kiện (1))

Khi P

i∈I

bi giảm 1, cịn P i∈I

bi tăng lên

Do X

i∈I

bi−

X

i∈II

bi

sẽ giảm Nếu sau đổi chỗ mà thỏa điều kiện (2) dừng, chưa thỏa tiếp tục làm đến lúc thỏa hiệu giảm ngặt tập số nguyên dương Vậy ta ln chia thành nhóm cho

X

i∈I

ai−

X

i∈II

ai ≤A X

i∈I

bi−

X

i∈II

bi ≤B

Trong hai nhóm I II, tổng số cam nhóm xác định, nên ta chọn nhóm số số cam nhiều Giả sử nhóm I Khi thêm thùng lấy cho vào nhóm I, số cam lấy lớn nửa

−A≤X

i∈I

ai−

X

i∈II

ai ≤A⇒

X

i∈II

ai ≤

X

i∈I

ai+A,

tức số Táo thỏa mãn điều kiện Tương tự kiểm tra cho số Xồi

Ví dụ 2.17.ChoXlà tập hợp hữu hạn,|X|=nvàA1, A2, , Am

là tập củaX thỏa mãn

(144)

Chứng minh tơ phần tử X hai màu, phần tử tô màu, cho tậpAi chứa phần

tử tô hai màu

Chứng minh. Giả sử X = {x1, x2, , xn} Giả sử kết luận

bài toán sai Trong tất cách tô màu phần tử tập X, chọn cáchtô tập cực đại X, giả sửY ={x1, x2, , xk}(k < n)thỏa mãn toán theo

nghĩa: thêm phần tửxk+1 vào Y khơng thể tơ màu

choxk+1 để thỏa mãn tốn

2 Điều dẫn đến có hai tập, giả sử làAr, As chứa xk+1

Ar, As⊆ {x1, x2, , xk, xk+1} cho

• Nếu xk+1 tơ màu xanh Ar ∩Y gồm toàn

phần tử màu xanh;

• Cịn xk+1 tơ màu đỏ As∩Y gồm toàn

phần tử màu đỏ

Nhưng thìAr∩As ={xk+1}, mâu thuẫn với giải thiết

bài toán Vậy điều giả sử sai Bài toán chứng minh

Ví dụ 2.18 (2000 Hungarian-Israeli). Đặt S = {1,2, ,2000} NếuAvàB hai tập S, ta ký hiệu |A|và|B|là số phần tử tập A tập B tương ứng Giả sử

|A|.|B| ≥3999

Chứng minh hai tập hợp A−A B −B chứa phần tử chung Ký hiệu

X−X ={s−t, s ∈X t∈X, s6=t}

Giải. ĐặtT ={(a, b)|a∈A, b∈B}

|T|=|A|.|B| ≥3999

TậpW ={a+b|a ∈A, b∈B}là tập tập{2,3, ,4000}

Nếu W = {2,3, ,4000} hai số 4000 nằm

(145)

Nếu W 6= {2,3, ,4000} W có 3999 phần tử Theo ngun lý Dirichlet, phải tồn hai cặp(a, b)6= (a0, b0)trongT để

a+b =a0+b0 =⇒ a−a0 =b−b0

Khi hai tập hợpA−A vàB−B chứa chung phần tử làa−a0 =b−b0 Ta có điều phải chứng minh

2.3 Thiết lập ánh xạ tập hợp

Ví dụ 2.19 (Italy 2000). Cho X tập hợp hữu hạn với

|X|=n, đặt A1, A2, , Am tập chứa phần tử

X thỏa mãn |Ai

T

Aj| ≤1 với i6=j Chứng tỏ tồn tập

A củaX chứa [√2n] phần tử mà không nhận tập

Ai(i= 1,2, , m)nào tập

Giải. Để tập A không chứa tập Ai nào làm tập con, thì lẽ tự nhiên tậpAchứa phần tử tốt.

2 Tuy nhiên đề lại yêu cầu |A| ≥ √2n, tức chặn dưới cho |A| Nghĩa đề yêu cầu tồn tập A có số lượng phần tử "tương đối" nhiều Để làm việc với tập tương đối nhiều phần tử, thông thường ta làm tập có nhiều phần tử nhất. Giả sử A tập X mà không chứa tập Ai nào, với phần tử lớn Đặt

k=|A|

Ý nghĩa sau: tập của X có số phần

tử > k, chứa tập conAi nào đó.

Do ta khảo cứu tập vi phạm sinh từ A, những tập tập A thêm phần tử ngoài A, tức thêm vào Amột phần tử tập X\A.

3 Cách Vì |A| chứa k phần tử nên X\A có n−k phần tử Cách Xétxlà phần tử củaX khơng thuộcA,

x∈ X\A Theo tính tối đại tập A, AS{x} khơng thỏa mãn điều kiện toán Nghĩa tồn số

i(x)∈ {1,2, , m} cho

Ai(x)⊆A

[

(146)

VìAi(x)6⊂A Ai(x) ⊆AS{x}nên x∈Ai(x) Từ suy

Ai(x)\{x} ⊂A

Vì tập Ai có ba phần tử nên Ai(x)\{x} có hai phần

tử, mà lại tập củaA, dẫn đến tập

Lx =A

\

Ai(x)

có hai phần tử Lại theo giả thiết toán|Ai

T

Aj| ≤1

với mọii 6=j nên tập Lx tập phân biệt (theo nghĩa, hai tập hợp Lx Ly , đây x 6= y thuộc X\A thì

Lx 6= Ly Vì nếu Lx = Ly, giả sử Lx = Ly = {a, b} Khi đó

{a, b} ⊂Ax,{a, b} ⊂Ay, dẫn đếnAx

T

Ay ={a, b}, mâu thuẫn)

5 Từ xác lập đơn ánh từ tập X\A đến tập hợp chứa tập hai phần tử A Do theo tính chất đơn ánh

n−k ≤

k

2

= k

2 −k

2

=⇒ k2+k ≥2n =⇒ k≥ −1 +

√ + 8n

2 ≥

√ 8n

2 = √

2n

Ví dụ 2.20 (AMM, E3459). Cho X tập hợp, |X|=n ≥12

vàF ={A1, A2, , Am} họ 4_tập X cho

|Ai∩Aj| ≥2,∀i6=j ∈ {1,2, , m}

Chứng minh tồn tập conS củaS,|S| ≥ √3

6n−6sao choAi 6⊂S,∀i∈ {1,2, , m}

Chứng minh. Ta gọi tập T X tập độc lập nều

Ai 6⊂T,∀i∈ {1,2, , m}

2 Xét S tập độc lập có kích thước lớn nhất X, đặt

|S| = k Khi k ≥ Vì S lớn nhất, nên với phần tử x∈X\S, tồn tại 3_tậpf(x)⊆S cho

f(x)∪ {x} ∈F

(147)

3 Lấy x1 6= x2 (x1, x2 thuộc X\S) Theo kết trên, tồn

hai tập chứa phần tửf(x1), f(x2)trong S cho

f(x1)∪ {x1} ∈F, f(x2)∪ {x2} ∈F

Rõ ràng f(x1) 6= f(x2), f(x1) ≡ f(x2) Khi hai tập

A1 =f(x1)∪ {x1}, A2 =f(x2)∪ {x2} có

|A1∩A2|=|f(x1)|=

mâu thuẫn giả thiết

4 Từ ta thấy f đơn ánh từ X\S vào tập chứa phần tử củaS Do

|X\S| ≤ |S| ⇔n−k ≤

k

3

Từ suy

6n ≤6

k

3

+ 6k =k3−3k2+ 8k ≤k3 −3⇒k ≥√3

6n+ Bài toán chứng minh hồn tồn

Ví dụ 2.21 (India Postal Coaching 2014 Set Problem 4).

Tập M viết dạng M =A1∪A2∪ .∪An Ai∩Aj = ∅

với ≤ i < j ≤ n, tập A1, A2, , An gọi

n_phân hoạch M Giả sử A1, A2, , An B1, B2, , Bn

hain_phân hoạch củaM thỏa mãn

|Ai∪Bj| ≥n,∀1≤i, j ≤n

Chứng minh |M| ≥ n

2

(148)

Chứng minh. Đặt k = min{|Ai|,|Bj|,1≤i, j ≤n} Không tính

tổng quát, giả sử|A1|=k

1 Do B1, B2, , Bn tập phân biệt Và |A1| =k, nên

có nhiều nhất k tập Bj {B1, B2, , Bn} màA1∩Bj 6=

∅ Khi tồn tạiít nhất n−k tập Bj {B1, B2, , Bn}

mà A1∩Bj =∅

2 Giả sử tập B1, B2, , Bn có chính xác m tập Bj

màA1∩Bj 6=∅, đóm ≤k Khơng tính tổng qt, gọi

mtập làB1, B2, , Bm Khi đón−mtậpBm+1, Bm+2, , Bn

cịn lại có A1∩Bj =∅,∀j =m+ 1, m+ 2, , n

3 Vì Bj ∩ A1 = ∅,∀j = m + 1, m + 2, , n, lại theo giả thiết

|Bj ∪A1| ≥n,∀j =m+ 1, m+ 2, , n Chứng tỏ

|Bj| ≥n− |A1|=n−k,∀j =m+ 1, , n

4 Theo định nghĩa k

|Bj| ≥ |A1|=k,∀j = 1,2, , m

Từ ta có

|M|=|B1∪B2∪ .∪Bn|

=|B1|+· · ·+|Bm|

| {z }

m tập

+|Bm+1|+· · ·+|Bn|

| {z }

n−m mìk+ (nm)ì(nk) =n(nk)m(n2k)

ã Nun2k, thỡn2k0 Do mk nên

|M| ≥n(n−k)−k(n−2k) = n

2

2 +

n

2 −k

2

≥ n

2

2

• Nếun <2k,

|M|=|B1∪B2∪ .∪Bn|=|B1|+· · ·+|Bn| ≥n×k >

n2

2

(149)

Ví dụ 2.22. Cho Ai tập hợp hữu hạn, với i = 1,2, , n

Chứng minh X

1≤i<j≤n

|Ai∩Aj|

|Ai|.|Aj|

<1,

thì tồn ∈Ai(i= 1,2, , n) để 6=aj,∀1≤i < j ≤n

Chứng minh. Đặt S = {1,2, , n} T = A1 ∪A2 ∪ .∪An Xét

ánh xạ f :SoT˜ cho: với i∈S f(i)∈ Ai (i= 1,2, , n)

GọiM tập hợp tất ánh xạ Khi

|M|=|A1|.|A2| .|An|

Ta chứng minh có đơn ánh trongM Nếu f ∈M mà không đơn ánh, tồn tạii, j ∈S, i6=j chof(i) =f(j) Với ánh xạf vậy, thìf(i) =f(j)nhận nhiều là|Ai∩Aj|

giá trị khác nhau, vàf(k)(k 6=i, j) nhận nhiều là|Ak|giá trị

khác Do với i, j ∈ S(i 6= j), số ánh xạ f mà f(i) = f(j)

nhiều

|Ai∩Aj| · n

Y

k=1,k6=i,j

|Ak|= |

Ai∩Aj|

|Ai|.|Aj|

.|M|

Từ suy số ánh xạ M mà đơn ánh

nhiều

X

1≤i<j≤n

|Ai∩Aj|

|Ai|.|Aj|

.M <|M|

Từ suy tồn đơn ánhf0 ∈M Đặt

f0(i) = ai(i= 1,2, , n)

Khi ∈ Ai(i = 1,2, , n) f0 đơn ánh nên i, j ∈S(i 6=j)

thì

ai =f0(i)=6 f0(j) = aj

Ví dụ 2.23 (Iran 1999). Cho n số nguyên dương tập hợp

X ={1,2, , n} Các tập A1, A2, , Ak tập X

cho với 1≤i1, i2, i3, i4 ≤n ta có

|Ai1 ∪Ai2 ∪Ai3 ∪Ai4| ≤n−2

(150)

Chứng minh. Một tập T ⊂ X gọi 2−phủ T ⊆ Ai∪Aj với i, j thuộc {1,2, , k}(i, j không thiết phân biệt)

Trong số tất tập X khơng bị 2−phủ, ta chọn tập có số lượng phần tử nhỏ nhất Gọi tập A

1 Xét họ tập hợpS1 ={A∩A1, A∩A2, , A∩Ak} (ở A∩Ai

có thể trùng A∩Aj, xảy điều ta bỏ khỏi tập S1

những tập trùng nhau) Vì Akhơng phải 2−phủ nên X ∈S1 A\X 6∈S1 (thật vậy, giả sử cảX vàA\X ∈S1,

đó

X =A∩Ar, A\X =A∩As

với r, sthuộc{1,2, , k} Nhưng

A=X∪(A\X) = (A∩Ar)∪(A∩As) = A∩(Ar∪As)⇒A⊆Ar∪As,

mâu thuẫn với A không 2−phủ.) Như có khơng q nửa số tập củaA nằm S1 Do

|S1| ≤2|A|−1

2 Bây lấy B =X\A (dĩ nhiên B 2−phủ, vì|B|>|A|), lại xét tiếp tập hợp

S2 ={B∩A1, B∩A2, , B∩Ak}

Khi nếuX ∈S2 B\X 6∈S2 Thật vậy, giả sử hai tập

X B\X nằm S2 Khi

X =B∩Ap B\X =B ∩Aq

thì

B ⊆Ap∪Aq

Theo tính nhỏ của|A|, suy tập có <|A| phần tử là2−phủ Do với m∈A

A\{m}=Ac∪Ad

với c, d∈ {1,2, , k} Khi

|Ac∪Ad∪Ap∪Aq| ≥ |B∪(A\{m})|=|X\{m}|=n−1,

mâu thuẫn với giả thiết Từ đây, tương tự lập luận trên, suy

(151)

3 Mặt khác, tậpAi xác định bởi(B∩Ai)∪(A∩Ai)

Do

k ≤ |S1|.|S2| ≤2|A|−1.2n−|A|−1 = 2n−2

2.4 ĐẾM SỐ TẬP HỢP CHỨA PHẦN TỬ - ĐẾM HAI

CÁCH

Ví dụ 2.24(PUTNAM 1980). ChoA1, A2, , A1066 tập

của tập hữu hạn X cho |Ai| >

1

2|X|, với ≤ i ≤ 1066

Chứng minh tồn 10 phần tử x1, x2, , x10 X

cho tập Aj(j ∈ 1,1066) chứa phần tử

mười phần tử

Chứng minh. Bài toán xét khi|X| ≥10 ĐặtX ={x1, x2, , xm},

với m=|X|

1 Với t (t∈ {1,2, , m}), đặt

nt=|{j ∈ {1,2, ,1066}|xt∈Aj}|

tức nt đếm số tập hợp Aj chứa phần tử xt Khi

• n1 đếm số tập hợp Aj chứa phần tử x1;

• n2 đếm số tập hợp Aj chứa phần tử x2;

• ;

• nm đếm số tập hợp Aj chứa phần tử xm

Do

n1+n2+· · ·+nm =|A1|+|A2|+· · ·+|A1066|>1066·

m

2 = 533m

Từ suy có số ni, khơng tính tổng qt giả

sử n1, mà n1 > 533 Tức ta có hơn nửa tập hợp

trong số tậpA1, A2, , A1066 chứa phần tửx1.

2 Gọi B1, B2, , Bs tập hợp số tập Ai mà

khơng chứa phần tử x1 Ta có

(152)

ĐặtY =X\{x1}={x2, , xm} Ngoài

|Bi|=|Ai|>

1 2m >

1

2(m−1) = 2|Y|

Với mỗit (t∈ {2, ,1066}), đặt

kt=|{j ∈ {1,2, , s}|xt∈Bj}|

tức kt đếm số tập hợp Bj chứa phần tử xt Khi

k2+k3+· · ·+km =|B1|+|B2|+· · ·+|Bs|> s·

m−1

2 =

s

2.(m−1)

Vế trái tổng củam−1số, phải tồn mộtki, khơng

mất tính tổng quát giả sử k2 cho k2 >

s

2 Nghĩa có hơn nửa tập hợp sốB1, B2, , Bschứa phần tửx2.

3 Đến lại đặt C1, C2, , Cr tập hợp số

tậpBj mà không chứa phần tử x2 Ta có

r =s−k2 < s− s

2 =

s

2 ≤ 532

2 = 265

Tiếp tục trình trên, ta lại được> r

2 (hơn nửa

trong số tập C1, , Cr) chứa phần tử x3 Ta lại

được dãy tập hợpD1, D2, , Du với u <

r

2 = 132 không

chứa phần tử x4

4 Cứ tiếp tục trình lần thứ đến lần thứ 10, dãy không nhiều 65, 32, 15, 7, 3, Do ta nhận phần tửx1, x2, , x10 thỏa mãn u cầu

tốn

Ví dụ 2.25 (China 1996, Romania 1994). Cho 11 tập hợp

Mi, i= 1,2, ,11thỏa mãn

(

|Mi|= 5,∀i= 1,2, ,11

Mi∩Mj 6=∅,∀1≤i < j ≤11

(153)

Gọi m số nguyên dương lớn cho tồn tập

Mi1, , Mim số 11 tập để

m

\

k=1

Mik 6=∅

Tìm giá trị nhỏ m tất cách chọn 11 tập hợpMi

thỏa mãn (I)

Chứng minh. Đặt X =S11i=1Mi Với x∈X, đặt

nx =|{j ∈ {1,2, ,11}|x∈Mj}|

vàm = max{nx|x∈X} Khi

X

x∈X

nx =|M1|+|M2|+· · ·+|M11|= 55

Theo giả thiết Mi∩Mj 6=∅ với 1≤i < j≤11

• Có 112 cách chọn giao khác rỗng, cách chọn giao khác rỗng cách chọn cặp tập hợp Mi, Mj

số 11 tậpM1, M2, , M11;

• Mặt khác, phần tử x xuất nx2 giao khác rỗng tập Mi, Mj

Do

X

x∈X

nx ≥ 11

= 55 Do

X

x∈X

nx(nx−1)

2 ≥55

Vìnx≤m,∀x∈X,

55≤ X

x∈X

nx(nx−1)

2 ≤

m−1

2

X

x∈X

nx =

m−1

2 55⇒

m−1

2 ≥1⇒m≥3

Nếu m = 3, dấu phải xảy tất bất đẳng thức trên, nên nx = m = 3,∀x ∈ X Nhưng Px∈Xnx = 55

(154)

củax∈X Từ suy m ≥4

Tiếp theo ta chứng minh m= thỏa mãn Xét

a b b d

e f g h

1

Khi 11 tập Mi lấy sau:

• tập dịng

M1 ={a, b, c, d, D}, M2 ={e, f, g, D},

M3 ={1,2,3,4, D}, M4 ={5,6,7,8, D}

• tập cột

M5 ={a, e,1,5, C}, M6 ={b, f,2,6, C},

M7 ={c, g,3,7, C}, M8 ={d, h,4,8, C}

• tập đường chéo

M9 ={a, f,3,8, D}, M10={b, g,4,5, D}, M11 ={c, h,1,6, D}

(Mỗi tập lấy dòng, cột phần tử)

Ví dụ 2.26 (USAMO 2011). Cho A1, A2, , A11 hợp

cho (

|Ai|= 45,∀1≤i≤11

|Ai∩Aj|= 9,∀1≤i < j ≤11

Chứng minh |A1∪A2∪ .∪A11| ≥165 cho ví dụ với

trường hợp dấu xảy

Chứng minh. Đặt X =A1∪A2∪ .∪A11, với mỗix∈X, đặt

nx =|{j ∈ {1,2, ,11}|x∈Aj}|

Khi

X

x∈X

(155)

• Có 112 cách chọn hai tập giao khác rỗngAi, Aj số 11

tậpA1, A2, , A11

• Mặt khác, phần tử x xuất nx2 giao khác rỗng tập Ai, Aj

Theo giả thiết |Ai∩Aj|= với 1≤i < j ≤11nên tập

giao hai tập Ai, Aj phần tử đếm lặp lần Do

đó

X

x∈X

nx

2

= 9×

11

= 495 Từ suy

X

x∈X

nx =

X

x∈X

nx

2

⇒nx = 3,∀x∈X

Mặt khác X

x∈X

nx =

X

x∈X

nx

2

= 495⇒X

x∈X

n2x = 3×495

Đặtn =|X|, theo bất đẳng thức Cauchy - Schwarz X

x∈X

1.nx

!2

≤ X

x∈X

1

! X

x∈X

n2x

!

⇒4952 ≤n×3×495⇒n≥165

Để kết thúc toán, ta thuật xây dựng Coi 11 tập hợp 11 mặt phẳng trong R3, khơng có hai mặt phẳng nào song song với Khi ba mặt phẳng tùy ý cắt tại một điểm Tổng số giao điểm đạt là

11

= 165 là điểm tập X.

Ví dụ 2.27(USA TST 2005). Chon số nguyên dương lớn Với số nguyên dương m, đặt Xm = {1,2, , mn} Giả sử

tồn họ F ={A1, A2, , A2n} gồm 2n tập Xm

cho

(156)

2 |Ai∩Aj| ≤1,∀1≤i < j ≤2n;

3 Mỗi phần tử Xm nằm hai tập hợp

thuộcF

Tìm giá trị lớn m theo n

Chứng minh. Với i∈Xmn, đặt

ni =|{j ∈ {1,2, ,2n}|i∈Aj}|

Khi đó, theo giả thiết thứ thìni = 2,∀i= 1,2, , mn Mặt khác,

sử dụng giả thiết thứ

mn X i=1 ni = X

1≤i<j≤mn

|Ai∩Aj| ≤

X

1≤i<j≤mn

1 = 2n n

Từ suy

mn≤

2n

2

⇒m≤2n−1

Mặt khác, ta lấy 2n đường thẳng R2, khơng có

hai đường thẳng song song Tổng số giao điểm cặp đường thẳng

2n

2

=n(2n−1) =n.m

là phần tử củaXmn Khi tậpAi chứa 2n−1giao điểm

trên đường thẳng i Nhận thấy tập Ai thỏa mãn điều kiện

bài toán Vậy giá trị lớn củan 2m−1

Ví dụ 2.28. Tìm số ngun dương n lớn cho tồn n tập hợp A1, A2, , An thỏa mãn

    

|Ai|= 4,∀i= 1,2, , n

|Ai∩Aj|= 1,∀1≤i < j≤n

|A1∪A2∪ .∪An|=n

Chứng minh. Đặt A1 ∪A2 ∪ .∪An = {1,2, , n} Với

i∈ {1,2, , n}, đặt

(157)

Khi ta có

n

X

i=1

ni =|A1|+· · ·+|An|= 4n (∗)

Dẫn đến với mỗii, trung bình xuất tập hợpAj

2 Giả sử tồn phần tử, khơng tính tổng qt, giả sử nằm trongnhiều tập hợp Khơng tính tổng qt, giả sử A1, A2, , A5

• Vì |Ai ∩Aj| = 1,∀1 ≤ i < j ≤ ta ln có Ai ∩Aj =

{1},∀1 ≤ i < j ≤ 5, nên 3×5 = 15 phần tử cịn lại

các tập A1, A2, A3, A4, A5 phải khác

• Ngồi khơng thể nằm tất tập hợp

A1, , An, khơng áp dụng lập luận

3×n = 3n

phần tử lại tậpA1, , Anphải phân biệt

Nhưng

|A1∪A2∪ .∪An|= 3n+ > n,

mâu thuẫn

• Do tồn tập hợp A6, , Ankhông chứa

phần tử Giả sửA6∩Ai (i= 1,2, ,5) tập phân

biệt Nếu ngược lại, giả sử

A6∩A1 =A6∩A2 ={b}

thì b 6= 6∈ A6 Nhưng b ∈ A1, b ∈ A2, mâu thuẫn với ý Từ dẫn đến tập A6 phải

có phần tử, mâu thuẫn với giả thiết

Vậy phần tử nằm khơng q tập hợp Ngồi có phần tử nằm phần tử, theo (*) phải có phần tử nằm ≤ tập hợp, dẫn

đến mâu thuẫn Vậy phần tử xuất đúng

(158)

3 Giả sử nằm trongA1, A2, A3, A4 Khi đó, giả sử

A1 ={1,2,3,4}, A2 ={1,5,6,7},

A3 ={1,8,9,10}, A4 ={1,11,12,13}

Khi đón ≥13

4 Nếu n ≥ 14 phần tử 14 nằm tập hợp, giả sử A5 Khi phần tử cịn lại A5 nằm tập

A1, A2, A3 Dẫn đến A5 vàA4 khơng có phần tử chung, mâu

thuẫn

5 Vậy n lớn 13, 13 tập hợp thỏa mãn A1 ={1,2,3,4}, A2 ={1,5,6,7}, A3 ={1,8,9,10},

A4 ={1,11,12,13}, A5 ={2,5,8,11}, A6 ={2,6,9,12},

A7 ={2,7,10,13}, A8 ={3,5,10,12}, A9 ={3,6,8,13},

A10={3,7,9,11}, A11 ={4,5,9,13}, A12={4,6,10,11},

A13={4,7,8,12}

Ví dụ 2.29(China 1999). Cho nnguyên dương X tập hợp với |X|=n Gọi A1, A2, , An tập củaX cho

|Ai| ≥2,∀i= 1,2, , n

Giả sử với tập A0 ⊂X,|A0|= tồn số i cho

A0 ⊆Ai

Chứng minh

Ai∩Aj 6=∅,∀1≤i < j ≤n

Chứng minh. Vì tập con, giả sử{a, b} củaX tồn số i cho {a, b} ⊆ Ai tập

chứa hai phần tử Ai khơng thể tập

tậpAj khác Do n

X

i=1

|Ai|

2

=

n

2

(159)

2 Đặt X ={x1, x2, , xn} Với i∈ {1,2, , n}X, ký hiệu

ni =|{j ∈ {1,2, , n}|xi ∈Aj}|

Khi

n

X

i=1

ni =

n

X

i=1

|Ai| (2)

Mặt khác

• Mỗi phần tử xi ∈ X nằm ni tập hợp Do

mỗi phần tử xi xuất ni2

tập giao

• Xét tập giao, giả sử {xi, xt}=Ar∩As Khi cảxi

và xt tính tập giao Do n X i=1 ni = X

1≤i<j≤n

|Ai∩Aj| (3)

3 Theo giả thiết tốn, |Ai ∩Aj| ≤ Và kết luận bài

toán cần chứng minh |Ai∩Aj|= Khi (3) trở thành

n X i=1 ni = n

Từ theo (1), (2) đẳng thức ta

n

X

i=1

|Ai|

2 = n X i=1 ni , n X i=1

ni =

n

X

i=1

|Ai|

dẫn đến kết luận toán đưa chứng minh

n

X

i=1

n2i =

n

X

i=1

|Ai|2 (∗)

4 Với phần tử xi ∈ X, xét tập Aj mà xi 6∈ Aj Giả sử

Aj ={y1, y2, , ys} Khi tập hợp chứa phần tử sau

đây

{xi, y1},{xi, y2}, ,{xi, ys}

là tập tập, chẳng hạn A1, A2, , As (lưu ý

(160)

chẳng hạn A1 Vì {y1, y2} tập A1,

mâu thuẫn với giả thiết Điều cho tất tập lại) Từ suy xi thuộc vào s = |Aj| tập hợp

A1, A2, , As (dĩ nhiên x thuộc vào số tập khác

nữa) Điều chứng tỏ nếuxi 6∈Aj

ni ≥ |Aj| ⇒

ni

n−ni ≥

|Aj|

n− |Aj|

Số tập hợpAj khơng chứaxi n−ni Do

X

j|xi6∈Aj

ni

n−ni

= (n−ni)·

ni

n−ni

=ni

Do

n

X

i=1

ni =

n

X

i=1

X

j|xi6∈Aj

ni

n−ni

n

X

i=1

X

j|xi6∈Aj

|Aj|

n− |Aj|

=

n

X

j=1

X

i|xi6∈Aj

|Aj|

n− |Aj|

=

n

X

j=1

|Aj|

Nhưng theo (2) đẳng thức phải xảy ước lượng Do ni =|Aj| Từ suy

n

X

i=1

(n−ni)ni = n

X

i=1

X

j|xi6∈Aj

di = n X i=1 X

j|xi6∈Aj

|Aj|

=

n

X

j=1

X

i|xi6∈Aj

|Aj|

=

n

X

j=1

(n− |Aj)|Aj|

(161)

2.5 Xây dựng - Quy nạp - Truy hồi

Ví dụ 2.30. Cho n nguyên dương tập hợp M = {1,2, ,20}

GọiA1, A2, , An tập phân biệt khác rỗng M

cho

|Ai∩Aj| ≤2,∀1≤i < j ≤n

Tìm giá trị lớn n

Chứng minh. Giả sử A1, A2, , An tập M

thỏa mãn điều kiện toán Nếu tậpA1, A2, , An

có phần tử, khơng tính tổng qt, giả sử

|A1| ≥4 Gọi a phần tử trongA1

• Khi A1\{a} tập có ba phần tử Do |A1 ∩

Ai| ≤ 2,∀i = 2,3, , n nên tập A1\{a} phải phân biệt

với tất tập A2, , An (vì nếuA1\{a} ≡Ai nào đó, thì

A1\{a} ∩Ai =A1\{a}

có ba phần tử Khi đó A1∩Ai cũng có ba phần tử, vơ lý.)

• Ngồi

|(A1\{a})∩Aj| ≤ |A1∩Aj| ≤2,∀j = 2,3, , n

2 Từ kết trên, thay A1 bởiA1\{a}thì hệ tập hợp

A1\{a}, A2, A3, , An

vẫn thỏa mãn toán Hệ có số tập hợp với hệ ban đầu, số phần tử tập hợp ≤ so với số phần tử tập hợp hệ ban đầu Cứ tiếp tục trình này, ta thu dãy tập hợp A∗1, , A∗n thỏa mãn toán và

|A∗i| ≤3,∀i= 1,2, , n

Tức tập hợp này, tập hợp có tối đa ba phần tử Do

n≤

20

+

20

+

20

(162)

3 Tiếp theo, tổng số tập chứa phần tử, chứa phần tử, chứa phần tử củaM

20

+

20

+

20

= 1350

và tất tập thỏa mãn điều kiện toán (rõ ràng hai tập phân biệt tập có giao khơng vượt q phần tử) Do n= 1350 đạt

Từ suy giá trị lớn n = 1350

Ví dụ 2.31(Indian 2014). Cho số tự nhiênnvàX ={1,2, , n} Với hai tập A vàB X, ký hiệu

A∆B ={i∈X|i∈(A\B)∪(B\A)}

GọiF họ tập X cho với A, B ∈ F

|A∆B| ≥2

Chứng minh rằng|F| ≤2n−1 và tìm tất họF có 2n−1 phần

tử

Chứng minh. Với tập A⊂ {1,2, , n−1}, cặp tập hợp(A, A∪ {n}),tối đa tập hợp thuộc vàoF Thật

A\(A∪ {n}) = ∅, (A∪ {n})\A={n} ⇒A∆(A∪ {n}) ={n}

Mặt khác, ta có tối đa 2n−1 cặp tập hợp (A, A∪ {n}) Do đó

tậpF có tối đa 2n−1 phần tử.

2 Tiếp theo ta chứng minh quy nạp theon nếu|F|=

2n−1 thìF hoặc chứa tất tập chứa số lẻ phần

tử hoặcF chứa tất tập hợp chứa số lẻ phần tử.

• Kết hiển nhiên với n=

• Giả sử kết đếnn =m−1, vớimnguyên dương lớn Xét trường hợp với n=m Đặt

F1 ={A∈ F|m∈A}, F2 ={A ∈ F|m 6∈A}

Theo giả thiết quy nạp, tập F∈ có nhiều 2m−2

(163)

• Với tập hợp A∈ F1, ta xét tập hợp A\{m} Khi

đó

F3 ={A\{m}|A∈ F1}

Khi tập F3 thỏa mãn yêu cầu toán theo

giả thiết quy nạp lại có |F3| ≤2m−2 Nhưng

|F2|=|F3| ≤2m−2 ⇒ |F|=|F1|+|F2| ≤2m−2+2m−2 = 2m−1

Do |F|= 2m−1 thì |F

1|=|F2| = 2m−2 Nhưng

đó lại theo giả thiết quy nạp, họF∈ chứa tất tập của\1,2, , m−1với, giả sử, số chẵn phần tử Khi F∞ chứa tất tập {1,2, , m}, chứa m, dạngA∪ {m} Tuy nhiênAkhông thể thuộc vào F2,

vì khơng A, A∪ {m} thuộc vào F vô lý Khi đóA phải có số lẻ phần tử, A⊂ {1,2, , m−1},

|F1|= 2m−2 nênA phải chạy tất tập có số lẻ

phần tử Do F1 chứa tập có số chẵn phần

tử Vậy F chứa tất tập hợp có số chẵn phần tử Tương tự F2 chứa tất tập có số lẻ phần tử

Bài tốn chứng minh

Ví dụ 2.32 (Bankal 2012). Cho n số nguyên dương Đặt tập

hợp Pn = {2n,2n−1.3,2n−2.32, ,3n} Với tập X Pn,

đặt SX tổng tất phần tử X, S∅ = Cho y

là số thực thỏa mãn điều kiện ≤ y ≤ 3n+1 −2n+1 Chứng

minh tồn tập Y Pn thỏa mãn điều kiện

0≤y−SY <2n Giải. Ta có

SPn = 3n+ 3n−1.2 +· · ·+ 32.2n−2 + 3.2n−1+ 2n

= (3−2)(3n+ 3n−1.2 +· · ·+ 32.2n−2 + 3.2n−1+ 2n) = 3n+1−2n+1

Bằng cách chia cách phần tử Pn cho 2n ta đưa toán

dạng tương đương sau:

Chonlà số nguyên dương,a=

2, vàQn ={1, a, a

2, , an} Chứng

tỏ với giá trị củaxthỏa mãn 0≤x≤1 +a+a2+· · ·+an, luôn tồn tập conX của Qn sao cho 0≤x−SX <1

(164)

1, S{a} =

3

2, S{1,a} =

2 Từ kiểm chứng thấy x

một số thực thỏa ≤ x ≤

2 ln có tập X Q1

thỏa yêu cầu

Giả sử kết toán đến số nguyên dương n Xét x số thực với 0≤x≤1 +a+a2+· · ·+an+an+1.

+ Nếu ≤x ≤1 +a+a2+· · ·+an thì theo giả thiết quy nạp tồn

tại tập X ⊂Qn ⊂Qm+1 thỏa 0≤x−SX <1

+ Xét vớix >1 +a+a2+· · ·+an= a

n+1−1

a−1 ,

an+1−1a−1 = a

n+1−1 −1

= an+1−1

=an+1+ (an+1−2)> an+1+a2−2 = an+1+

4 > a

n+1

nên

0<(x−an+1)≤1 +a+a2+· · ·+an

Theo giả thiết quy nạp, tồn tập X ⊂Qn thỏa mãn

0≤(x−an+1)−SX <1 =⇒ 0≤x−SX0 <1

với

X0 =X[{an+1} ⊂Qn+1

Bài toán chứng minh

Ví dụ 2.33 (Romania, grade 9, final round). Cho p, q hai số nguyên dương, p ≥ 2, q ≥ Một tập hữu hạn X gọi có tính chất(S)nếu: với cách chọnptập conBi ⊂X, i= 1,2, , p,

không thiết phân biệt, tập có q phần tử, ln tồn tập Y ⊂X có p phần tử, cho

|Y ∩Bi| ≤2,∀i= 1,2, , p

Chứng minh

(165)

Chứng minh. Nếu X có pq−q= (p−1)q phần tử, ta chọnp−1tập, tập cóq phần tử (là phân hoạch củaX) sau

B1 ={1,2, , q}, B2 ={q+ 1, q+ 2, ,2q}, ,

Bp−1 ={(p−2)q+ 1, ,(p−1)q}

và tậpBp tập tùy ý cóqphần tử Khi đó, theo nguyên

lý Dirichlet, với tậpY có p phần tử X, tồn tậpBi (i ∈ {1,2, , p−1}) cho giao

nó vớiY có≥2 phần tử Vậy tậpX khơng có tính chất (S) Bây với i cho trước, nhận xét tập

[

j6=i

Bi

có ≤ (p−1)q phần tử Trong tập X có pq−q+ = (p−1)q+ 1phần tử Do ta tìm ít phần

tử tập X mà khơng nằm tập Bj, với

mọi j 6=i

• Với i = 1, sử dụng lập luận trên, ta tìm phần tử

x1 mà x1 6∈ Bj,∀j = 2, , p Nếu x1 ∈ B1, ta tiếp tục

qua bước 2, nếux1 6∈B1, ta xây dựng tậpB1 mới bằng

cách bỏ khỏi B1 một phần tử y1, thêm vào

tập B1 phần tử x1. Dĩ nhiên phần tử y1 phần

tử X

• Tiếp tục vớii= 2, lưu ý tậpB1 sử dụng tậpB1

"mới" Lại sử dụng lập luận trên, ta tìm phần tử x2 màx2 không nằm tập B1, B3, B4, , Bn

Khi x2 6= x1 (vì x1 ∈ B1, x2 6∈ B1) Nếu x2 ∈ B2 ta tiếp

tục quy trình, x2 6∈ B2, ta xây dựng tập B2 mới

bằng cách bỏ khỏi B2 một phần tử y2, thêm

vào tập B2 phần tử x2.

3 Cứ tiếp tục quy trình đến bước p, ta tìm tập

Y = {x1, x2, , xp} Tập Y giao với tập Bi "mới"

xác phần tử, xi Bây thay ngược trở lạixi

yi ta quay trở lại tập Bi "cũ" Nếu yi 6∈Y Y ∩Bi =∅, cịn

(166)

Ví dụ 2.34 (China TST 2015). Cho X tập khác rỗng

A1, A2, , An tập X cho

1 |Ai| ≤3,∀i= 1,2, , n;

2 Bất kỳ phần tử X nằm tập sốA1, A2, , An

Chứng minh chọn

3n

7

tập hợp số tập

A1, A2, , An mà hợp chúng bằngX

Chứng minh. Kết luận toán yêu cầu chọn số tập hợp, hợp lại X, ta

1 Chọn tập có phần tử, giả sử A1,|A1| = 3 Sau

đó, ta chọn tiếp tập A2 |A2| = A2 ∩ A1 = ∅ Sau

khi chọn tập A2, ta chọn tiếp tập A3 |A3| =

A3∩A1 = ∅, A3∩A2 = ∅, tiếp tục đến không

thể chọn thêm tập vào hệ. Trong tất cách cách lựa chọn tập hợp A1, A2, , An, ta

chọn hệ tập hợp S3 cực đại, giả sử S3 = {A1, A2, , Ai}

(i ≤ n) (tức họ S3 chứa nhiều tập hợp có) mà

|At|= 3,∀t = 1,2, , i vàAr∩As=∅,∀1≤r < s ≤i(điều

có nghĩa, lần bổ sung tập hợp vàoS3 tậpX3 có

số lượng phần tử tăng lên 3)

ĐặtX3 =SAr∈S3Ar Do tính tối đại tập S3, nên với bất

kỳ tập hợpAj (j > i)

(167)

vì khơng ta tiếp tục bổ sung Aj vào tập S3, mâu

thuẫn với tính tối đại S3 Và

|X3|= 3i

2 Bây ta tiếp tục chọn họ S2 cực đại chứa tập Aj

còn lại số Ai+1, , An, cho mỗi lần thêm một

tập hợp vào họ S2, số lượng phần tử hợp của

chúng tăng lên 2 Khơng tính tổng quát, giả sử

S2 ={Ai+1, Ai+2, , Aj}

Đặt

X2 =

[

Ar∈S2

Ar∩(X\X3)

thì theo cách xác định S2 ta có |X2| = 2j theo tính

tối đại tậpS2

|At∩(X\(X2∪X3))| ≤1,∀t=j+ 1, , n

3 Bây ta tiếp tục chọn họS1 chứa tậpAs lại

sốAj+1, , An, chomỗi lần thêm tập hợp vào họ

S1, số lượng phần tử hợp chúng tăng lên

1 dĩ nhiên tập hợp họ S1 chứa hết tất cả

các phần tử của X\(X3 ∪X2) = X1 Khơng tính tổng

qt, giả sử

(168)

4 Khi đó|X|=|X1|+|X2|+|X3|= 3i+ 2j+k,X =X1∪X2∪X3

|S3|+|S2|+|S1|=i+j +k =m

Ta cần chứng minh m≤ 37n

• Vì phần tử X1 nằm tập hợp,

nhưng |Ar∩X1| ≤1,∀r =j + 1, , n nên

n≥i+j+ 4k (1)

Mỗi phần tử X1 ∪X2 xuất

tập hợp, |Ar| ∩(X1∪X2)≤2,∀r=i+ 1, , n nên

n ≥i+4(2j+k)

2 =i+ 4j+ 2k (2)

Mỗi phần tử trongX xuất tập hợp,

và |Ar∩X| ≤3,∀r = 1,2, , n,

n 4(3i+ 2j+k)

3 (3) ã Ly 20ì(1) + 12×(2) + 27×(3) ta có

59n≥140(i+j+k) = 140m ⇒m≤ 59n

140 < 3n

7

Bài tốn chứng minh hồn tồn

Ví dụ 2.35 (Romania TST 2006). Cho n số nguyên dương Một tập S ⊂ {0,1,2, ,4n−1} gọi tập rời rạc với số k bất kỳ, hai điều kiện sau thỏa mãn

1 Tập ST{4k−2,4k−1,4k,4k+ 1,4k+ 2} có tối đa phần tử Tập ST{4k+ 1,4k+ 2,4k+ 3} có tối đa phần tử

Hỏi tập {0,1, ,4n−1} có tập rời rạc?

(169)

• GọiSn tập hợp chứa tập rời rạc tập

{0,1,2, ,4n−1}

Rõ ràng các số có dạng4k+ 1,4k+ bị ràng buộc hai điều kiện Đây sở cho việc xây dựng công thức truy hồi

• GọiXn tập hợp chứa tập củaSn mà tập

của chứa phần tử 4n−1 (tức tập của Sn các tập chứa phần tử chia dư 3)

• GọiYn tập hợp chứa tập củaSn mà tập

của không chứa hai phần tử 4n−1 4n−2 (tức là chứa tập củaSn mà tập khơng chứa hai phần tử chia dư chia dư 3, nghĩa chứa các phần tử chia hết cho chia dư 1)

• GọiZn tập hợp chứa tập Sn mà tập

của chứa phần tử 4n−2(tức chứa tập củaSn mà tập chứa phần tử chia dư 2)

Khi

|Sn|=|Xn|+|Yn|+|Zn|

1 Xét tập hợpXn+1 xây dựng từ tập hợp lại Xét

tập hợp {1,2, ,4n−2,4n−1,4n,4n+ 1,4n+ 2,4n+ 3}

• Từ tập Xn, ta thêm vào tập

Xn phần tử 4n không thêm vào, thân tập

Xn+1 chứa phần tử 4n+ Vậy |Xn+1|= 2|Xn|

• Từ tập Yn, ta thêm vào tập củaYn

phần tử4n khơng thêm vào Vì vậy|Xn+1|= 2|Yn|

• Từ tập Zn, ta thêm vào tập Yn

phần tử 4n khơng thêm vào Vậy |Xn+1|= 2|Zn|

Từ ta có quan hệ truy hồi

|Xn+1|= 2|Xn|+ 2|Yn|+ 2|Zn|= 2|Sn| (1)

2 Xét tập hợp Yn+1 xây dựng từ tập hợp cịn lại

• Từ tập Yn, ta thêm vào tập

Yn phần tử 4n 4n + 1, thêm hai không

(170)

• Từ tập Xn, ta thêm vào tập

của Xn phần tử 4n hoặc4n+ 1nhưng thêm

đồng thời hai phần tử này, khơng thêm vào Khi |Yn+1|= 3|Xn|

• Từ tập Zn, ta thêm vào tập

Zn phần tử4n hoặc4n+ 1nhưng thêm đồng

thời hai phần tử này, khơng thêm vào Khi

|Yn+1|= 3|Zn|

Từ ta có quan hệ hồi quy

|Yn+1|= 3|Xn|+ 4|Yn|+ 3|Zn| (2)

3 Tương tự ta có

|Zn+1|= 2|Xn|+ 2|Yn|+ 2|Zn|= 2|Sn| (3)

Từ ba quan hệ (1), (2), (3) ta có|Sn+1|= 7.|Sn| Từ ta tính

|Sn+1|= 7n.|S1|= 8.7n

3 Bài tập vận dụng

Bài tập 3.1 (Olympic KHTN lần 1). Xét tập M ={1,2,3, ,10}

và A1, A2, , An dãy tập khác rỗng, phân biệt M

sao cho

|Ai∩Aj| ≤3,∀1≤i < j ≤n

Tìm giá trị lớn n

Đáp số:n = 385

Bài tập 3.2 (Taiwan 1998). Cho n ≥ k ≥ X = {1,2, , n} GọiFk họ k_tập X cho với A, B ∈Fk

|A∩B| ≤k−2

Chứng minh tồn tập conMk củaX,|Mk| ≥[log2n]+1

sao cho Mk không nhận phần tử Fk tập

(171)

Bài tập 3.3 (THTT 444 - Trần Nam Dũng). Cho 100 điểm

A1, A2, , A100 nằm hình vng ABCD có cạnh

Chứng minh tồn tập conXcủaE ={1,2, ,100}

gồm 50 phần tử cho

X

i∈X

−−→

AAi−

X

i∈E\X

−−→

AAi

Bài tập 3.4(China 2006). Cho trước số nguyên dươngn ≥2và tậpX hữu hạn Gọi B1, B2, , Bn n tập tùy ý X,

tập chứa hai phần tử Tìm giá trị nhỏ |X| cho tồn tập Y củaX thỏa mãn hai điều kiện

1 |Y|=n;

2 |Y ∩Bi| ≤1,∀i= 1,2, , n Đáp số:|X|= 2n−1

Bài tập 3.5(Tuyển tập Olympic 30-4 năm 2007). ChoA1, A2, , A10

là tập hợp thỏa mãn điều kiện |Ai|= 8,∀i= 1,2, ,10;

2 |Ai∩Aj|= 1,∀1≤i < j ≤10

Chứng minh |A1∪A2∪ .∪A10| ≥39

Bài tập 3.6 (Dan Schwarz). Cho X tập hợp Gọi n m≥1 số nguyên không âm cho

|X| ≥m(n−1) +

Giả sử B1, B2, , Bn n tập X cho |Bi| ≤ m,∀i ∈

{1,2, , n} Chứng minh tồn tập Y X

cho|Y|=n |Y ∩Bi| ≤1,∀i= 1,2, , n

Bài tập 3.7 (China 2010). Cho số nguyên dương n ≥

A1, A2, , A2n tập phân biệt tập {1,2, , n} Xác

định giá trị lớn

2n

X

i=1

|Ai∩Aj|

|Ai|.|Aj|

(172)

Đáp số:

Bài tập 3.8 (Chọn đội tuyển KHTN 2010). Cho số nguyên dương n >10 Tìm số nguyên dương m lớn thỏa mãn điều kiện: Tồn tạimtập conAj tập {1,2, ,2n}, tập gồm

n phần tử cho

|Ai∩Aj∩Ak| ≤1,∀1≤i < j < k ≤m

Đáp số:m =

Bài tập 3.9 (China 2014). Với tập hợp khác rỗng S, T, ta định nghĩa hai tập

S+T ={s+t|s ∈S, t ∈T}, 2S ={2s|s∈S}

Cho n số nguyên dương A, B tập khác rỗng

{1,2, , n} Chứng minh tồn tập D A+B

cho

1 D+D⊆2(A+B); |D| ≥ |A|.|B|

2n

Bài tập 3.10. Cho n k số nguyên dương cho n > k2 −k + Cho n tập hợp, tập hợp có k phần tử cho hai tập hợp tùy ý n tập hợp có phần tử chung Chứng minhn tập hợp có phần tử chung

Bài tập 3.11. Cho ke ≥ số tự nhiên Tìm số tự nhiên nhỏ n cho với tập gồm n số ngun ln có số mà tổng hiệu chúng chia hết cho 2k+

Bài tập 3.12. Xác định số n lớn cho tồn cho tồn tập phân biệt S1, S2, , Sn thỏa mãn

1 |Si

S

Sj| ≤2006 với 1≤i, j ≤n

2 SiSSjSSk={1,2, ,2010}với 1≤i < j < k≤n

(173)

Bài tập 3.14(Stars of Mathematics 2014). Cho số nguyên dương M, m, n thỏa mãn ≤ m ≤ n,1 ≤ M ≤ m(m+ 1)

2 A

một tập {1,2, , n} cho |A| =m Chứng minh tồn tập conB ⊆A cho

0≤X

b∈B

b−M ≤n−m

Định lý 3.1 (Erdos). Cho F họ tập tập n

phần tửX thỏa mãn

1 |F| ≥2 với mọiA∈ F |A| ≥2

2 Với hai phần tử x, y ∈ X tồn A ∈ F để

{x, y} ∈A Khi

|F| ≥n

Tài liệu tham khảo

1 Một số định lý lý thuyết tập hợp cực trị, Vũ Thế Khôi, Bài giảng Viện Toán Học dành cho trường hè 2012 Combinatorial Mathematics, Stefan H M van Zwam,

Prince-ton University 2013

3 Khoảng cách Hamming, giảng mạng Combinatorics of sets, Po-Shen Loh, June 2013

5 Combinatorial problems in Mathematical Competitions, Yao Zhang, World Scientific 2011

(174)(175)

VỚI NHIỀU CÁCH GIẢI

NGUYỄN DUY LIÊN

(THPT Chuyên Vĩnh Phúc)

Lời giới thiệu

Giải toán Số học hay khó, ta cảm thấy thích thú Nhưng tốn Số học hay khó mà giải nhiều cách mà từ ta giải được, hay tạo số toán lớp tốn niềm vui cịn nhân lên nhiều lần Bài viết này, xin giới thiệu với bạn cách giải cho toán số Số học hay khó kỳ thi Olympic Toán học Quốc tế (IMO) lần thứ 42 Hoa Kỳ

Chúng ta bắt đầu với tốn

Bài tốn.

Cho số nguyên dương a, b, c, d với a > b > c > d > Giả sử

ac+bd = (b+d+a−c) (b+d−a+c) Chứng minh ab+cd

không phải số nguyên tố

Lời giải 1. Giả sử ab+cd số nguyên tố Ta có

ab+cd= (a+d)c+ (b−c)a=m·gcd (a+d, b+c) (∗)

( với m số nguyên dương gcd (a+d, b−c) ước số chung lớn củaa+d vàb−c)

Từ(∗)suy m = hoặcgcd (a+d, b−c) =

Trường hợp :m =

gcd (a+d, b−c) =ab+cd > ab+cd−(a−b+c+d) = (a+d) (c−1) + (b−c) (a+ 1)≥gcd (a+d, b−c)

(176)

Trường hợp :gcd (a+d, b−c) = Ta có

ac+bd= (a+c)b−(b−c)a

kết hợp với đề

ac+bd= (b+d+a−c) (b+d−a+c)

ta :

(a+c)b−(b−c)a= (b+d+a−c) (b+d−a+c)

Suy

(a+d).(a−c−d) = (b−c) (b+c+d) (∗∗) Từ đẳng thức(∗∗)tồn số nguyên dương k cho:

a−c−d=k(b−c) vàb+c+d=k(a+d)

từ suy ra:

a+b=k(a+b−c+d)⇔k(c−d) = (k−1) (a+b)

kết hợp với a > b > c > d >0ta có:

• Nếuk = 1⇒c=d vơ lý

• Nếuk ≥2thì 2≥ k

k−1 =

a+b

c−d >2 vô lý

Từ vô lý trường hợp 2, nênab+cdkhông phải số nguyên tố

Lời giải 2. Theo đề

ac+bd= (b+d+a−c) (b+d−a+c)

biến đổi ta

a2−ac+c2 =b2+bd+d2 (1)

Xét tứ giácABCD với

AB=a, BC =d, CD =b, DA=c ;\BAD= 600;BCD\ = 1200

(177)

Áp dung định lý hàm số côsin hai tam giácBAD vàBCD , ta có

BD2 =a2+c2−2ac cosBAD\=b2+d2−2bd cosBCD\

Suy đẳng thức (1)

Áp dung định lý hàm số côsin hai tam giácABC ACD, ta có

AC2 =a2+d2−2ad cosα =b2+c2+ 2bc cosα

Suy

2 cosα = a

2+d2−b2−c2

ad+bc

AC2 =a2+d2−ada

2+d2−b2 −c2

ad+bc =

(ab+cd) (ac+bd)

ad+bc

Tứ giác ABCD nội tiếp đường tròn, theo định lý Ptơlêmê ta có

(AC·BD)2 = (ab+cd)2

suy

(ac+bd) a2−ac+c2= (ab+cd) (ad+bc) (2)

Từa > b > c > d >0, ta suy

ab+cd > ac+bd > ad+bc (3)

Giả sử rằngab+cdlà số nguyên tố

Từ(3)ta thấy hai số ab+cdvàac+bdnguyên tố Cho nên từ đẳng thức(2) ta có ac+bd chia hết ad+bc theo (3) vơ lý Nên ab+cdkhông phải số nguyên tố

Lời giải 3. Từ a > b > c > d >0, ta suy

ab+cd > ac+bd > ad+bc (3)

Theo đầu

ac+bd= (b+d+a−c) (b+d−a+c)

nên ta có

(178)

Do :

(ab+cd) (ad+bc) =ac b2+bd+d2+bd a2−ac+c2 (5) Từ(4) (5), suy

(ab+cd) (ad+bc) = (ac+bd) a2−ac+c2 (6) Giả sử rằngab+cdlà số nguyên tố

Từ(3)ta thấy hai số ab+cdvàac+bdnguyên tố Cho nên từ đẳng thức(6) ta có ac+bd chia hết ad+bc theo (3) vô lý Nên ab+cdkhông phải số nguyên tố

Lời giải 4. Theo đề ac+bd= (b+d+a−c) (b+d−a+c)biến đổi ta

a2−ac+c2 =b2+bd+d2 (7)

Giả sử rằngab+cdlà số nguyên tố, đặt

ab+cd=p⇒ab≡ −cd(m

¯odp) kết hợp với (7) ta có

0 = b2 a2−ac+c2+b2 b2+bd+d2

≡c2d2+bc2d+b2c2+b4 +b3d+b2d2

≡ b2+c2 b2+bd+d2 (modp) (8)

Từ(8) suy b2+c2 ≡0 (modp)hoặc b2+bd+d2 ≡0 (modp)

Trường hợp 1.b2+c2 ≡0 (modp)

0< b2+c2 <2 (ab+cd) = 2p⇒b2+c2 =p

nên ta suy

b2+c2 =ab+cd⇔b(a−b) = c(c−d)

dẫn tới

⇒c(c−d)≡0 (modp) (9) Theo giả thiếtab+cdlà số nguyên tố hai số(b, c) = 1, từ(9) suy c−d≡0 (modp) vơ lý

Trường hợp b2+bd+d2 ≡ (modp) Điều tương đương với

(179)

0< a2−ac+c2 <2 (ab+cd) = 2p

nên ta suy

a2−ac+c2 =p=ab+cd

do đó, ta có (

a2−ac+c2 =ab+cd

b2+bd+d2 =ab+cd

(

c(c−d) = ab+ac−a2

d(c−d) =b2+bd−ab

(

a| c(c−d)

b| d(c−d) (10)

Mà ab+cd số nguyên tố (a , c) = (b , d) = nên

từ (10)suy (

a| c−d

b| c−d

điều vô lý

Từ vô lý trường hợp 2, nênab+cdkhông phải số nguyên tố

Lời giải 5. Theo đề ac+bd= (b+d+a−c) (b+d−a+c)suy

a+b−c+d| ac+bd⇒ a+b−c+d| ac+bd +a(a+b−c+d)

hay

a+b−c+d| a2+bd+ab+ad= (a+b) (a+d)

Giả sử (a+b−c+d, a+d) = 1⇒ a+b−c+d| a+b

Đặta+b=k(a+b−c+d) (11) với k số nguyên dương Nếuk = 1, từ (11) ⇒a+b =a+b−c+d⇒c=d vô lý Nếuk ≥2, từ (11) ta suy

a+b=k(a+b−c+d)≥2 (a+b−c+d)> a+b

điều vô lý a > b > c > d >0 Vậy (a+b−c+d, a+d)6= Giả sử có số nguyên tốp cho p | (a+b−c+d , a+d) Ta có

p| a+d

(180)

a≡ −d(modp)

b ≡c(modp)

dẫn tới

ab≡ −cd(modp)⇔ab+cd≡o(modp)

Màab+cd > p cho nênab+cdkhông phải số nguyên tố,

hợp số

Từ cách giải bạn vận dụng vào giải toán tương tự sau

Bài 1. Chứng minh : a2+ac−c2 =b2+bd−d2 với số nguyên dương a > b > c > d > ab+cd khơng phải số ngun tố

Bài 2. Cho số nguyên dương a, b, c, d với a > b > c > d >

0 thoả mãn điều kiện a+b−c+d| ac+bd Chứng minh rằng: anbm+cmdn không phải số nguyên tố ( đóm,n là những

số nguyên dương n số lẻ )

Bài 3. Cho số nguyên dương a, b, c, dvà số nguyên tố pthoả mãn hệ thức a

p+bp

cp+dp =

1

p−1 Chứng minh rằng:a+b+c+d chia

hết cho p

(181)

CỦA CÁC ĐƯỜNG VNG GĨC

VỚI CÁC CẠNH CỦA TAM GIÁC

VÀ ỨNG DỤNG

VŨ THANH TÙNG, NGUYỄN CHƯƠNG CHÍ

1 Lời giới thiệu

Vừa qua Forum "Bài toán hay–Lời giải đẹp–Đam mê toán học" diễn thảo luận sôi thành viên đề tài mà đề cập báo Đầu tiên toán hay giả thiết đưa Đào Thanh Oai Khi đó, giả thiết chưa có lời giải

Bài toán 1 (Đề Đào Thanh Oai [1]) Cho tam giác

ABC có Ma, Mb, Mc trung điểm cạnh BC, CA, AB

Ha, Hb, Hc chân đường cao tương ứng với đỉnhA, B, C

Gọi A1, B1, C1 tâm ba đường tròn (AMbHc),(BMcHa)

(CMaHb) Chứng minh ba đường thẳng qua A1, B1, C1

vng góc với ba cạnh BC, CA, AB đồng quy

Bài toán thử thách khơng nhỏ có nhiều điểm, nhiều đường Trong đó, phương pháp chứng minh đồng quy lại đa dạng sử dụng tứ giác nội tiếp, Ceva, Desargues, v.v Tuy vậy, tốn khơng q phức tạp Sử dụng định lý Carnot, chứng minh toán cách gọn gàng

Ngay sau toán chứng minh, có nhiều nghiên cứu sâu hướng mở rộng khác

Bài toán 2 (Nguyễn Ngọc Giang [1]) Cho tam giác ABC có

Ma, Mb, Mc ba trung điểm cạnh BC, CA, AB Gọi P

một điểm mặt phẳng chứa tam giác ABC có hình chiếu vng góc xuốngBC, CA, AB Pa, Pb, Pc Gọi

A1, B1, C1 tâm ba đường trịn (AMbPc),(BMcPa),(CMaPb)

(182)

Hình 11.1: Giả thiết Đào Thanh Oai

Bài toán 3(Nguyễn Văn Lợi [1]) Cho tam giác ABC hai điểm

bất kỳ P, N mặt phẳng Gọi hình chiếu vng góc P xuống BC, CA, AB Pa, Pb Pc N Na, Nb Nc Gọi

A1, B1, C1 tâm ba đường tròn (ANbPc),(BNcPa)và(CNaPb)

Chứng minh ba đường thẳng qua A1, B1, C1

vuông góc với cạnhBC, CA, AB đồng quy

Như ban đầu Đào Thanh Oai dùng trực tâm tâm ngoại tiếp, Nguyễn Ngọc Giang dùng điểm P tâm ngoại tiếp, Nguyễn Văn Lợi dùng hai điểm P N làm liệu cho giả thuyết Điều đáng lưu ý ba giả thuyết chứng minh cách gọn gàng dùng định lý Carnot cơng cụ

Đến câu hỏi đặt ra: Nếu bỏ qua điều kiện sáu điểmNa, Nb, Nc, Pa, Pb, Pc hình chiếu vng góc hai

(183)

Hình 11.2: Hai tam giác trực giao

tốn đưa tìm điều kiện cần đủ cho sáu điểm Na, Nb, Nc, Pa, Pb, Pc cho kết luận toán

đúng

Bài toán 4 (Bài toán tổng quát [1]) Cho tam giác ABC sáu

điểm Na, Pa ∈ BC, Nb, Pb ∈ CA, Nc, Pc ∈ CA Gọi A1, B1, C1 lần

lượt tâm đường tròn ngoại tiếp tam giác ANbPc, BNcPa

và CNaPb Chứng minh đường thẳng qua A1, B1, C1

lần lượt vng góc với BC, CA, AB đồng quy đường trung trực củaNaPa, NbPb, NcPc đồng quy

(184)

Hình 11.3: Bài tốn tổng quát

2 Định lý Carnot

Xét hai tam giác ABC A0B0C0 mặt phẳng Các đường thẳngd0A,d0B,dC0 quaA0, B0, C0và vng góc vớiBC, CA, AB Ta định nghĩa đại lượng Carnot tam giác A0B0C0 tam giácABC sau:

cABC(A0B0C0) = (A0B2−A0C2) + (B0C2−B0A2) + (C0A2−C0B2)

Đại lượng đưa tiện lợi cho việc phát biểu định lý Carnot theo cách Chú ý đại lượng Carnot phụ thuộc vào thứ tự đỉnh ta xét hai tam giác Nói cách kháccABC(A0B0C0)là khác với cABC(B0A0C0)hay cABC(A0C0B0)

Định lý 1(Định lý Carnot - 1803). : [2] Ba đường thẳngd0A, d0B, d0C qua A0, B0, C0 vng góc với BC, CA, AB đồng quy vả

(185)

hay

cABC(A0B0C0) =

Từ định nghĩa đại lượng Carnot ta thấy: cABC(A0B0C0) +cA0B0C0(ABC) =

Nhờ định lý Carnot, ta thấy nếud0A, d0B, d0C đồng quy điểm X0 ba đường thẳng dA, dB, dC qua A, B, C vng

góc vớiB0C0, C0A0, A0B0 đồng quy điểm X Trong hình học, hai tam giácABC vàA0B0C0 thỏa mãn điều kiện định lý Carnot gọi trực giao với (orthologic), hai điểm đồng quy gọi hai tâm trực giao (orthologic centers) X gọi tâm trực giao tam giácABC với tam giác A0B0C0 vàX0 gọi tâm trực giao tam giác A0B0C0 với tam giácABC [3]

Sau số tính chất đại lượng Carnot

Bổ đề 1. Gọi Ta, Tb, Tc trung điểm BC, CA, AB

số thựck ∈R Ta có:

1 cABC(A0B0C0) = −2.(−−→BC.−−→A0Ta+−→CA.−−→B0Tb +−→AB.−−→C0Tc)

2 Nếu A01A02⊥BC, B10B20⊥CA, C10C20⊥AB thì: cABC(A01B10C10) =cABC(A02B20C20)

3 Những mệnh đề sau tương đương: (a) d0A, d0B, d0C đồng quy

(b) (A0B2−A0C2) + (B0C2−B0A2) + (C0A2−C0B2) = (c) cABC(A0B0C0) =

(d) cA0B0C0(ABC) =

(e) dA, dB, dC đồng quy

4 Nếu −−−→A01A00 =k−−−→A10A02,−−−→B01B00 =k−−−→B10B20,−−−→C10C00 =k−−−→C10C20 thì:

(a) cABC(A00B00C00) = k.cABC(A01B10C10) + (1−k).cABC(A02B20C20)

(186)

Hình 11.4: Bổ đề 1.4

2.0.0.1 Chứng minh.

1 Ta có:

A0B2−A0C2 = −−→A0B.(−−→A0C−−−→BC)−−−→A0C.(−−→A0B+−−→BC) = −−−→BC.(−−→A0B +−−→A0C)

= −2.−−→BC.−−→A0Ta

Tương tự vậy, ta có:

B0C2−B0A2 =−2.−→CA.−−→B0Tb; C0A2 −C0B2 =−2.−→AB

−−→

C0Tc

Cộng ba đẳng thức trên, ta được: cABC(A0B0C0) =−2.(−−→BC

−−→

A0Ta+−→CA

−−→

B0Tb+−→AB

−−→

C0Tc)

2 Từ phần ta có:

cABC(A01B10C10)−cABC(A02B20C20) = −2.(−−→BC

−−−→

(187)

=

3 Trước hết ta thấy rằng: b⇔c⇔d Ta chứng minha ⇔c GọiH giao điểm d0B vàd0C Khi đó, từ phần ta có:

cABC(A0B0C0) = cABC(A0HH) =cABC(HHH)−2.−−→BC

−−→

A0H =−2.−−→BC.−−→A0H Suy cABC(A0B0C0) = ⇔ −−→BC

−−→

A0H = ⇔ BC⊥A0H Như a⇔c Tương tự ta có: e⇔d

4.a Từ giả thiết ta có:

−−−→

TaA00 = k

−−−→

TaA01+ (1−k)

−−−→

TaA02;

−−→

TbB00 = k

−−→

TbB10 + (1−k)

−−→

TbB20;

−−→

TcC00 = k

−−→

TcC10 + (1−k)

−−→

TcC20

Từ ba đẳng thức từ phần ta suy hệ thức cần chứng minh

b GọiX00 điểm cho:−−−→X10X00 =k.−−−→X10X20.Từ định lý Thales ta có: A01X10kA00X00kA02X20 Mặt khác A01X10⊥BC nên A00X00⊥BC Tương tự ta có: B00X00⊥CA, C00X00⊥AB Do X00 tâm trực giao tam giácA00B00C00 tam giác ABC

3 Lời giải tốn tổng qt bình luận

3.1 Lời giải toán tổng quát

Gọi Xa, Xb, Xc trung điểm NaPa, NbPb, NcPc Ta

chứng minh rằng:cABC(XaXbXc) = cABC(A1B1C1)

Thật vậy, gọi Ta, Tb, Tc điểm choATa, BTb, CTc đường

kính đường trịn (A1),(B1) (C1) Khi A1, B1, C1

trung điểm ATa, BTb vàCTc Nhờ bổ đề ta có:

cABC(TaTbTc) +cABC(ABC) = 2.cABC(A1B1C1)

Tuy nhiên ta thấy cABC(ABC) = 0, đó:

cABC(TaTbTc) = 2.cABC(A1B1C1)

Mặt khác theo bổ đề ta lại có:

(188)

Hình 11.5: Lời giải tốn tổng qt

= (TaB2−TaA2)−(TaC2−TaA2) + (TbC2−TbB2)

−(TbA2−TbB2) + (TcA2−TcC2)−(TcB2−TcC2)

= (BPc2−APc2)−(CNb2−APc2) + (CPa2−BPa2) −(ANc2−BNc2) + (APb2−CPb2)−(CNa2−BNa2) = cABC(NaNbNc) +cABC(PaPbPc)

= 2.cABC(XaXbXc)

Từ đó, ta suy cABC(XaXbXc) = cABC(A1B1C1) Áp đụng định lý

Carnot ta có: đường thẳng qua A1, B1, C1 vng góc

vớiBC, CAvàABđồng quy⇔cABC(A1B1C1) = 0⇔cABC(XaXbXc) =

0 ⇔ đường thẳng qua Xa, Xb, Xc vuông góc với

BC, CAvàABđồng quy⇔các đường trung trực củaNaPa, NbPb, NcPc

(189)

3.2 Một số trường hợp đặc biệt

Trong toán trên, tập sáu điểm Na, Pa, Nb, Pb, Nc, Pc thỏa mãn

điều kiện tốn tổng qt có số trường hợp đặc biệt sau

• Na, Pa, Nb, Pb, Nc, Pc giao điểm đường tròn(O)với

các cạnh tam giác ABC Khi đó, đường trung trực củaNaPa, NbPb, NcPc đồng quy O

• (Giả thuyết Nguyễn Văn Lợi nêu [1]) Gọi

Na, Nb, Nclà hình chiếu điểmN xuốngBC, CA, AB;Pa, Pb, Pc

là hình chiếu điểmP xuốngBC, CA, AB Khi ta dễ dàng nhận thấy đường trung trực củaNaPa, NbPb, NcPc

đồng quy trung điểm củaN P

Một trường hợp đặc biệt (Na, Pa, Nb, Pb, Nc, Pc) xây

dựng tập

Ngoài thấy vai trò hai điểm Na Pa

BC ngang nên hốn đổi hai điểm Tương tự, hoán đổi hai điểm Nb vàPb, hai

điểm Nc Pc Như vậy, có tám ba đường thẳng

đồng quy Các bạn thử vẽ hình tìm tính chất tám điểm đồng quy nhé!

4 Một số toán luyện tập

Tiếp theo số toán minh họa cho định lý Carnot lời giải chúng

Bài tốn 5 (ví dụ sáu điểm thỏa mãn điều kiện toán

tổng quát) Cho tam giác ABC, đường tròn qua B, C cắt

CA, AB Ab, Ac; đường tròn qua C, A cắt BC, BA Bc, Ba;

đường tròn qua A, B cắt CA, CB Ca, Cb Chứng minh

các đường trung trực đoạn CaBa, AbCb, BcAc đồng quy

Lời giải 6. GọiAa, Bb, Cclà trung điểm củaCaBa, AbCb, BcAc Theo

định lý Carnot, cần phải chứng minh rằng:

(190)

Hình 11.6: điểm thỏa mãn điều kiện toán tổng quát Ta xét phần biểu thức trên:

ACc2−BCc2 = (ACc+BCc).(ACc−BCc) =AB.(ACc−BCc)

ACc = 1/2.(AAc+ABc) = 1/2.(AAc+AB−BBc)

BCc = 1/2(BBc+AB−AAc)

Do

ACc2−BCc2 =AB(AAc−BBc)

Tương tự trên:

BA2a−CA2a=BC(BBa−CCa); CBb2−ABb2 =CA(CCb−AAb)

Do ta cần phải chứng minh:

(191)

Vì bốn điểmB, C, Ac, Ab đồng viên nênAB.AAc=AC.AAb Tương

tự, BC.BBa =BA.BBc CA.CCb =CB.CCa Như biểu thức

trên Từ định lý Carnot, ta suy điều phải chứng minh

Bài toán 6 (Định nghĩa orthopole (Soons-1886) [4]) Cho tam

giác ABC đường thẳng d mặt phẳng Gọi A0, B0, C0 hình chiếu A, B, C d Chứng minh đường thẳng quaA0, B0, C0và vng góc vớiBC, CA, AB đồng quy P (điểm đồng quy P gọi orthopole đường thẳngd tam giác ABC)

Hình 11.7: Orthopole

Lời giải 7. Ta có:

cABC(A0B0C) = (A0B2−A0C2) + (B0C2−B0A2) + (C0A2−C0B2)

= (B0B2+A0B02−C0C2−A0C02) + (C0C2+B0C02−A0A2−A0B02) +(A0A2+A0C02−B0B2−B0C02)

(192)

Từ định lý Carnot, ta có điều phải chứng minh

Bài toán 7 (Hai tam giác trực giao hoàn toàn (định lý Pantazi)

[3]) Giả sử tam giácABC trực giao với tam giácA0B0C0 vàB0C0A0 Chứng minh tam giác ABC trực giao với tam giác C0A0B0

Lời giải 8. Từ định nghĩa của đại lượng Carnot ta dễ dàng chứng minh được:

cABC(A0B0C0) +cABC(B0C0A0) +cABC(C0A0B0) =

Từ theo định lý Carnot:cABC(A0B0C0) = 0;cABC(B0C0A0) = 0.Do

đó ta suy cABC(C0A0B0) = Theo định lý Carnot, ta có điều

phải chứng minh

5 Kết luận

Chúng giới thiệu định lý Carnot, công cụ hiệu cần chứng minh đường thẳng vuông góc với ba cạnh tam giác đồng quy Đại lượng Carnot khái niệm đưa vào tiện lợi việc trình bày lời giải tốn cách có hệ thống Sử dụng định lý Carnot toán tổng quát gần xuất giải gọn gàng

Tài liệu tham khảo

[1] Nhóm "Bài tốn hay - Lời giải đẹp - Đam mê toán học":

www.facebook.com/groups/Loicenter/

[2] Định lý Carnot www.cut-the-knot.org/pythagoras/Carnot shtml

[3] Hai tam giác trực giao www.mathworld.wolfram.com/

OrthologicTriangles.html

(193)

TỪ DIỄN ĐÀN

A

O

PS

TRẦN QUANG HÙNG, NGUYỄN BẢO NGỌC

(Trường THPT Chuyên KHTN, ĐHKHTN, ĐHQG Hà Nội)

Tóm tắt

Bài viết toán diễn đàn AoPS trình bày ứng dụng với cơng cụ hình học túy

1 Mở đầu

Trên diễn đàn AoPS có tốn thú vị sau (ký hiệu toán sửa lại để phù hợp với viết)

Bài toán 1. Cho tam giác nhọn ABC nội tiếp đường tròn (O)

Gọi H giao điểm hai đường cao BE, CF; AH cắt (O)

D khácA;DE cắt (O)tại G khác D Chứng minh BG chia đôiEF

Lời giải sau ý tưởng Jeck Lim, nick name oneplusone diễn đàn AoPS

A

B C

O

H

K F

E

D

G

(194)

Lời giải. Gọi K giao điểm AH BC Ta dễ dàng chứng minh K trung điểm HD hai tam giác EF B EHK đồng dạng GọiM trung điểm củaEF, ta có

EF

HE =

BF

HK hay

EF

2HE = BF

2HK hay

F M

HE =

BF

HD

Kết hợp với điều kiện ∠BF M = ∠DHE, ta suy hai tam giác

BF M DHE đồng dạng Do

∠F M B =∠HDE =∠F BG

VậyB, M, G thẳng hàng

Ta có điều phải chứng minh

Nhận xét. Việc dùng kỹ thuật đồng dạng "gấp đôi chia đơi cạnh" lời giải tốn hay, giúp có nhiều cách để phát triển mở rộng toán

2 Các mở rộng

Trong tốn trênE, F giao điểm đường trịn đường kính BC với cạnh CA, AB Vậy thử thay đường tròn đường kính BC thành đường trịn bất kỳ, ta có tốn sau:

Bài tốn 2. Cho tam giác ABC nội tiếp đường tròn (O) Một

đường tròn (K) qua B, C cắt AC, AB E, F; BE giao với CF tạiH; AH cắt(O)tại P khácA; P E cắt(O)tại R khácP Chứng minh BRchia đôi EF

Lời giải. Gọi D hình chiếu K lên AH Lấy điểm N đối xứng vớiF quaDK Suy N thuộc (K) Vì

∠BCP =∠BAP =∠BF N =∠BCN

nên C, P, N thẳng hàng Gọi AH cắt BC L vàQ đối xứng với

P qua D Ta có

∠F QA=∠QF N =∠F N C =∠AP C =∠ABC

(195)

A

B C

O

K E

F

H

P D

Q

L

N

R

I

Tương tự tứ giácCEQLnội tiếp nên tứ giác AEQF nội tiếp Mặt khác ta dễ dàng chứng minh hai tam giác EF B EQD đồng dạng Gọi I trung điểm EF Ta có hai tam giácIF B EQP đồng dạng Suy

∠F BI =∠EP Q=∠ABR

VậyBR quaI chia đôi EF

Nhận xét. Lời giải sử dụng kỹ thuật đồng dạng "gấp đôi chia đôi cạnh" Tuy nhiên rõ ràng tốn mở rộng dùng khéo léo để vận dụng hết kiện mở rộng toán

Trong tốn gốc, ta coi H nằm đường tròn đối xứng với đường tròn ngoại tiếp tam giác ABC qua BC Vậy ta thử thay điểmH thành điểm đường trịn này, ta có kết thú vị sau:

Bài toán 3. Cho tam giác nhọn ABC nội tiếp đường tròn (O)

P thuộc cung nhỏBC cho Q đối xứng P qua BC Q

nằm tam giác ABC QB, QC cắt CA, AB E, F;

(196)

A

B C

O

P Q

E F

D G

R

I

Lời giải. GọiDlà giao điểm đường tròn ngoại tiếp tam giác ABE vàBC Ta dễ có tứ giác AEQF nội tiếp nên

∠EQC =∠BAC =∠EDC

suy tứ giácEQDC nội tiếp Từ

∠BAD =∠BED =∠QCD =∠DCP =∠BAP

vậy ta thu đượcA, D, P thẳng hàng GọiGđối xứngP quaD, từ GQkBC suy

∠DGQ=∠GDB =∠AEB

vì tứ giác AEQG nội tiếp Từ dễ có

∠EGD =∠EF B

∠F BE =∠GDE

Vậy hai tam giác EF B EDG đồng dạng Gọi I trung điểm EF suy tam giác IF B vàEGP đồng dạng Vậy ta thu

∠F BI =∠GP E =∠ABR

nên BRđi qua I chia đôiEF

(197)

3 Một số ứng dụng toán gốc và

các mở rộng

Bài tốn gốc tốn mở rộng có nhiều ứng dụng thú vị, hay tìm hiểu qua toán sau Bài toán sau đề xuất [3]

Bài toán 4. Cho tam giác ABC có đường cao AK, BE, CF

GọiDlà giao điểm củaAK và(O);LthuộcKE choBL⊥OA;

DE cắt(O) điểm G khácD Chứng minh AL BG cắt trung điểmEF

LL

G

F

E

K A

B C

O H

D K

Lời giải. Gọi H trực tâm tam giác ABC M trung điểm EF Theo toán thìBG qua M Mặt khác ta có EF ⊥OAnênBLkEF VìBG qua trung điểmM củaEF nên

B(EF, GL) = −1 Từ hàng điều hòa ta có

E(BA, F K) =−1

nên

B(EF, GL) = E(BA, F K)

Suy A, M, L thẳng hàng Vậy BL AG cắt trung điểmEF Ta có điều phải chứng minh

(198)

Bài toán 5. Cho tam giác nhọn, khơng cânABC nội tiếp đường trịn(O) Một đường trịn(K)đi quaB, C cắt đoạn thẳngCA, AB

lần lượt E, F khác C, B;BE giaoCF H;AH cắtBC D;

AD cắt (O) G khácA;GE cắt (O) N khác G; P thuộcDE

sao cho BP k EF Chứng minh BN AP cắt trung điểm EF

A

B C

K E

F

H

O

D

G

N

P I

Lời giải. Theo toán 4, ta chứng minh đượcBN qua trung điểmI củaEF DoBP kEF nên chùmB(EF, N P) =−1 Kết hợp với chùm

E(BA, F D) =−1 suy B(EF, N P) = −1 =E(BA, F D)

Suy giao điểm tương ứng A, I, P thẳng hàng Ta có điều phải chứng minh

Bài tốn 6. Cho tam giác nhọnABC có đường caoAD, BE, CF

đồng quy trực tâm H Gọi M, N trung điểm

DE DF; BM cắt CN P Chứng minh HP chia đôi

(199)

A

B C

O H

D F

E

M N

P L

K

S

T

Q

Lời giải. Gọi (O) đường tròn ngoại tiếp tam giác ABC Gọi K, L giao điểm củaBE CF với (O) Theo trước ta dễ thấy LE vàBM cắt tạiS thuộc(O) Tương tựKF CN cắt tạiT thuộc(O) Gọi Qlà giao điểm SL vàKT Áp dụng định lý Pascal cho

B T L C S K

ta suy P, H, Q thẳng hàng Ta dễ dàng chứng minh tứ giácEF LK hình thang nên theo bổ đề hình thangHQ chia đôiEF Suy raP H chia đôi EF Ta có điều phải chứng minh

Bài tốn 7. Cho tam giác nhọnABC có đường cao AD, BE, CF

đồng quy trực tâm H Trung trực HD cắt DF, DE Q, R;

BQ cắtCR P Chứng minh HP chia đôi EF

A

B C

O H

D F

E

M N

R P

Q

S I

(200)

Lời giải. Theo trước ta chứng minh BQ qua trung điểm M EF CR qua trung điểm N DF tốn chứng minh Thật vậy, gọiHQ, HRcắtBC tạiS, T Vì tam giác HSD vng D tam giác QHD cân H suy tam giácQSD cân Q Do

∠QSD =∠QDS =∠EDC

nên HQ k DE Mặt khác, dễ thấy Q trung điểm HS nên BQ qua trung điểm M DE Tương tự CR qua trung điểmN củaDF Theo trước ta có điều phải chứng minh

Nhận xét. Sử dụng phương pháp chiếu song song ta dễ dàng

suy toán với điểm P mặt phẳng thay cho H Bài toán tổng quát giải đơn giản biến đổi diện tích Các bạn quan tâm tới vấn đề

Bài toán 8. Cho tam giácABC nội tiếp đường tròn(O)và đường

caoAD, CF GọiK giao điểm củaAD và(O);KF cắt(O)tại L

khácK; đường thẳng quaA vng góc OC cắtCLtại N Chứng minh F N ⊥F O

A

B C

O H

D F

K N L

M P

Q

Lời giải. Ta chứng minh4F N A∼ 4F OC để suy ra4F N O ∼

4F AC Từ suy

khởi nguyên một bấtđẳng thức tổng quát chứng minh rằng câu hỏi triệu đô quyển sách ông Sputnik Education. http://sputnikedu.com/. Sput-nik Education tại đây), tại đây) https://damtson.wordpress.com John Nash, Andrew Wiles, Grothendieck, PaulErdă Alan Turing Richard K Guy Don-ald Knuth Robert P.Crease Thời báo New York Braum, Raymond Smullyan, Roger Penrose, Piet Hein Isaac Asimov Vladimir Nabokov, M.C Escher Salvador Dalí Lewis Carroll Arthur Harold Stone, John Tukey Bryant Tuckerman Richard Feynman Arthur Cayley Johannes Kepler tại đây. tại đây. William Newcomb tại õy. "Cỏ thỏng t" Wolfram. Măobius Guthrie Alfred Kempe, Peter Tait, Percy Heawood Julius Petersen Kenneth Appel Wolfgang116 Stan Wagon, Gottfried Leibniz Benoˆıt Mandelbrot Conway, "trò chơi đời" Ron Rivest, AdiShamir Leonard Adleman, ở định lý lớn Fermat, giả thuyết 3n +1 Fan Chung Ronald Graham Martin Gardner, puzzle master extraordinaire Martin Gardner – The Best Friend Mathe-matics Ever Had Magical Mathematics - A Tribute to Martin Gardner Martin Gardner Người gợi cảm hứng cho niềmsay mê toán học Martin Gardner, Puzzler and Polymath,Dies at 95 The Top 10 Martin GardnerScientific American Articles Trò chơi đời MartinGardner’s Mathemagical Life www.facebook.com/groups/Loicenter/ www.cut-the-knot.org/pythagoras/Carnot.shtml www.mathworld.wolfram.com/OrthologicTriangles.html www.mathworld.wolfram.com/Orthopole.html

Ngày đăng: 07/02/2021, 22:50

w